Vous êtes sur la page 1sur 389
McGraw Hill Education (We) Series IN Ll! Tata McGraw-Hill Published by Tata McGraw Hill Education Private Limited, 7 West Patel Nagar, New Delhi 110 008 ‘Numerical Chemistry Copyright © 2012, Tata McGraw Hill Education Private Limited, No part of this publication may be reproduced or distributed in any form or by any means, electronic, mechani- cal, photocopying, recording, or otherwise or stored in a database or retrieval system without the prior written permission of the publishers. The program listings (if any) may be entered, stored and executed in a computer system, but they may not be reproduced for publication. This edition can be exported from India only by the publishers, ‘Tata MeGraw Hill Education Private Limited. ISBN (13): 978-12-8-900122-2 ISBN (10): 1-25-900122-9 Vice President and Managing Director—MeGraw-Hill Education: India: Ajay Shukla Head—Test Prep and School: V. Biju Kumar Publishing Manager—Test Prep: K N Prakash Manager (Sponsoring): Abhishek Sharma Editorial Executive: Ravinder Singh Asst Manager (Developmental Editing): Anubha Srivastava ‘Asst Manager—Production: Medha Arora Production Executive: Dharmender Sharma Junior Product Specialist: Vikas Sharma General Manager—Production: Rajender P. Ghansela Manager—Production: Reji Kumar Information contained in this work has been obtained by Tata McGraw-Hill, from sources believed to be reliable, However, neither Tata McGraw-Hill nor its authors guarantee the accuracy or completeness of any information published herein, and neither Tata McGraw-Hill nor its authors shall be responsible for any errors, omissions, or damages arising out of use of this information. This work is published with the understanding, that Tata McGraw-Hill and its authors are supplying information but are not attempting to render engineering. or other professional services. If such services are required, the assistance of an appropriate professional should be sought. ‘Typeset at Script Makers, 19, A1-B, DDA Market, Paschim Vihar, New Delhi 110063, and text and cover printed at Cover Designer: K Anoop The McGraw-Hill Companies Contents Preface 1 Stoichiometry Basic Concepts in Chemistry Laws of Chemical Combinations Significant Figures Scientific Notations Mole Concepts Equivalent Mass Limiting Reagent Concentration Units Volumetric Analysis Concepts of H,O, Solution Concept of Oleum Hardness of Water JIT Window-1 JIT Window-H IIT Window-IIL IIT Window-1V. IIT Window-V LIT Window-VI UIT Window-VI Answer Keys Atomic Structure Fundamental Aspects Introduction of electron, Proton and Neutron 1-42 ARAM REKH KEKE Nl 27 28 32 37 39 40 43-69 43 44 Important Concepts 45 AIT Window-I 49 LIT Window-IT 53 AIT Window-HT 58 NT Window-1V 61 HIT Window-V 62 IT Window-VI 63 HIT Window-VIL 64 Answer Keys 68 Radioactivity 71-90 Radioactivity 1 Nuclear Transformation or Nuclear Transmutation 7 HIT Window-I 73 LIT Window-II sl AIT Window-T 84 LIT Window-IV 8s HIT Window-V 89 Answer Keys 90 Gaseous State 91-125 Gas ~ Laws 91 Kinetic Theory of Gases 93 Critical phenomenon 95 LIT Window-I 7 x Contents LIT Window-Il 103 LIT Window-Il 107 IIT WindowlV 12 IIT Window-V- 113 IIT Window-VI 116 JIT Window-VIT 7 HIT Window-VI U8 Answer Keys 123 Chemical Bonding 127-157 Ionic Bond or Electrovalent Bond 127 Lattice Energy 127 Characteristics of Electrovalentcompounds 128 Covalent Bond (By Mutual Sharing of Electrons) 129 Characteristics of Covalent Compounds 130 Fajan’s Rules 131 Hydrogen Bonding 131 Coordinate Bond 134 Valence Shell Electron Pair Repulsion (VSEPR) Theory 134 Valence Bond Theory 137 Sigma and Pi Bonds (and x Bonds) 138 Hybridisation 138 Molecular Orbital Theory 141 LIT Window-I 142 LIT Window-I 145 JIT Window-HI 147 IIT Window-IV- 148 IIT Window-V 152 IIT Window-VI 155 Answer Keys 156 6. Theory of dilute Solution 159-191 Colligative Properties Theory of Dilute Solutions 159 Solubility of Solids and Gases 160 Henry’s Law 160 Methods of Expressing the Concentration of a Solution 161 Ideal Solution 162 Raoult’s Law (when both Components are volatile) 163 Raoult’s Law (when only one Components is volatile) 163 Colligative Properties of Dilute Solutions 164 Van’t Hoff Theory of Dilute Solutions 167 Reverse Osmosis 167 ‘Van't Hoff factor 167 UIT Window-1 168 UIT Window-Il 174 MIT Window-IL 17 NIT Window-1V 180 HIT Window-V 184 MIT Window-V1 186 LUT Window-VIL 187 LT Window-VIL 188 MIT Window-1X 189 Answer Keys 190 Chemical Kinetics 193-234 Rate of Reaction 193 Factors affecting the Reaction Rate 194 Molecularity and order of.a reaction 194 Order of Reaction 195 Contents xd Rate law and Mechanism. 195 Radius Ratio 242 First order reaction 196 Structure of Simple lonic Examples of Ist order Reaction 197 Compounds 243 Second Order Reaction 198 Effects of Pressure and aes Grdee Reweaicn og ‘Temperature on Crystal Structure 245 Effect of Temperature on Bragg’s Law 246 Reaction Rate 199 Closed Packing 246 Arrhenius Equation 199 Properties of Solids 248 Activation Energy 199 Metal 249 Advanced Reaction Types 200 uT LIT Window-1 202 HT Window-IT IIT Window-IT 208 HIT Window-HI IT Window-Il 212 HIT Window-IV IIT Window-1V 221 LIT Window-V- IIT Window-V 222 HT Window-VI LIT Window-VI 224 AIT Window-VIT IIT Window-VIT 229 Answer Keys Answer Keys 232 9. Surface Chemistry 267-294 8. Solid State 235-265 Teeraustion 267 Introduction 235 Adsorption 267 Classification of Solids 235 Desorption 267 ‘Types of Symmetry in Crystals 236 Sorption 268 Crystal Lattice 237 ‘Types of Adsorption 269 Unit Cell 237 Adsorption Isotherms 269 Types of Unit Cells 237 Freundlich’s adsorption isotherm 269 Seven Crystal System 238 Langmuir Adsorption Isotherm 270 Bravais Lattices 238 Adsorption Isobars 271 Types of Lattice 239 Adsorption Isostere an Schottky Defect 241 Catalysis 21 Frankel Defect 24t Classification of Catalysis 21 Metal Excess, 241 Types of Catalysis 272 Metal Deficiency 242 Characteristics of Catalysis 272 H_Contents 10. Theories of catalysis 272 Enzyme Catalysis 273 Characteristics of Enzyme catalysts 273 Colloids 274 Classification of colloids 274 Purification of solutions or colloids 276 Properties of colloidal solution 217 Protection of colloid 218 AIT Window-1 279 HIT Window-H 283 AIT Window-U 288 HIT Window-1V 290 LIT Window-V 291 AIT Window-VI 292 HIT Window-VI 292 Answer Keys 293 Chemical Equilibrium 295-327 Some Irreversible Reactions 295 Some Reversible Reactions 295 State of Chemical Equilibrium 296 Characteristics of Equilibrium 296 Law of Mass Action 296 Characteristics of equilibrium constant 297 Examples for some Equilibriums 298 Le-Chatelier’s Principle 300 Degree of dissociation from density measurements 300 Le-chatelier’s principle and physical equilibrium 300 IIT Window=1 301 LIT Window-H 307 WW. NIT Window-UI1 3H UIT Window-1V 31S HIT Window-V 37 MIT Window-VI 37 NT Window-VIT 319 Answer Keys 326 tonic Equilibrium 329-359 Acid Base and Salts 329 Arrhenius Concept (Water-ion System) 329 Bronsted and Lowry Concept: (Proton donor & Acceptor System) 329 Lewis Concept (Electron Pair Donor and Acceptor System) 330 Strength of Acids 330 Strength of Base 331 SHAB Concept: (Soft & Hard Acid-base Concept) 331 Salts 331 ‘Types of Salt 331 Relative Strength of Acids and Base 332 Electrolytes 332 Tonisation of Water and p" 333 Ostwald Dilution Law 333 Hydrolysis of Salt 334 Buffer Solution 336 ‘Types of Buffer Solution 336 Solubility and Solubility Product 337 Ionic Product 337 Common Ion Effect on Solubility 337 Acid-Base Indicators 338 Acid Base Titration Curve 339 UIT Window-1 340 12. HIT Window-I 346 LIT Window-Il 349 HIT Window-1V 351 LIT Window-V 353 HIT WindowVI 353 LIT Window-VI 355 HIT WindowVUI 356 Answer Keys 358 Chemical Energetics 361-397 (Thermodynamics and Thermochemistry) Terminology 361 Zeroth Law of Thermodynamics 363 First Law of Thermodynamics 363 Enthalpy or Heat Content 365 Joule—Thomson Effect 366 Entropy 366 Second Law of Thermodynamics 367 Gibbs Free Energy (G) 368 Third Law of Thermodynamics 369 The Carnot Cycle 369 Thermochemistry 370 Hess Law of Constant Heat Summation 373 Kirchhoff’s Equation 373 LIT Window=1 373 HIT Window-I 379 LIT Window-I 386 HIT Window-1V 388 LIT Window-V 390 LIT Window-VI 392 LIT Window-VI 392 13, 14, UIT Window-VIT UT Window-IX Answer Keys Electrochemistry Electrolytes Electrolysis, Discharge Potential Theory Faraday’s Law of Electrolysis Electrolytic Conductance Kohlrausch’s Law Cell Standard Hydrogen Electrode Calomel Electrode Electrochemical Series Nernst Equation Ecell and p#® Concentration Cell Dry-Cell Lead Storage Cell AIT Window-I AIT Window-I HIT Window-HT LIT Window-IV- HIT Window-V AIT Window-VI IIT Window-VIL MIT Window-VIIL Answer Keys Redox Oxidation and Reduction 394 395 396 399-437 399 399 400 400 401 403 404 406 406 406 407 409 409 410 410 4il 418 427 429 431 432 433 434 436 439-459 439 iv Contents Valency, Oxidation Number and Formal Charge Redox tion Balancing of Redox Reaction LIT Window-1 440 442 443 445 AIT Window-IT LIT Window-HT AIT Window-IV- Answer Keys Appendix to Chapter 14 446 447 448 448 449 CHAPTER Stoichiometry Basic Concepts in Chemistry ‘The branch of chemistry which contains the funda- mental concepts and relationship between chemical identities is called as stoichiometry. In this unit fol- lowing facts are discussed. (A) Laws of Chemical Combinations (B) Significant Figures (C) Scientific Notations (D) Mole Concepts (B) Atomic and Molecular Mass (F) Equivalent Concepts (G) Limiting Reagent (H) Concentration U (1) Volumetric Analysis () Concept of HO, Solution (K) Concept of Oleum (L) Hardness of Water Laws of Chemical Combinations Law of Conservation of Mass (Lavoisier) It states that the mass of reactants before reaction and the mass of products and reactants (if left) are equal. ie. A+B Products It shows that mass of reactants A and B before reac- tion is equal to mass of products formed and mass of reactant if remain, after reaction Law of Constant Composition or Definite Proportions (Proust) Itstates that mass ratio of the elements in a compound is always fixed and it is independent of the method by which it is formed, €.g. Mass ratio of Hydrogen and Oxygen in H,0 is 2 : 16 or 1 : 8 (Itis fixed). Law of Multiple Proportions (Dalton) It states that weight of different elements which combine with fixed weight of other element, bear a simple ratio. e.q. The weight of oxygen that combine with 14 parts by weight of Nitrogen in NO, NO;, N03, N,O, and NO; are 8, 16, 24, 32 and 40 ie. in the ratio 1:2:3:4:5 Law of Reciprocal Proportion (Ritcher) It states that weights of two elements 4 and B which combine separately with a fixed weight of third ele- ment C, are cither the same or in simple ratio of A and B. “ c 2. Numerical Chemistry Mass ratio of C:S in CS, = 12: 64 Mass ratio of C&O in CO, = 12 : 32 © Mass ratio of $:0 in SO, = 32:32= 1:1 Law of gaseous volume (Gay-Lussac) It states that if gases reactants combined to form the ‘gaseous products their mole ratio is simply expressed in terms of volume ratio. e.g. H, +Cl, 2 HCI ® @& The ratio of the volume of H,, Cl, and HCI are 2 respectively. ( Significant Figures The Significant Figures are the digits which are known accurately and one extra digit which is known jin accurately e. 0012 has 2 significant figure 1002 has 4 significant figure 10.0 has 2 significant figure 7500 has 4 significant figure 6.023 x 10% has 4 significant figure Scientific Notations In Scientific Notation, digits are written in terms of digit having decimal after a digit and other terms are written in power of 10. ¢.g 1232 is written as 1.232 x 10° 6023 is written as 6.023 x 10° 0.001 is written as 1.0 x 10° (Cole Concepts “Mole” literary mean “Collection of identities”. It is the unit of amount of substance. One mole is that amount of substance which contains Avogadro’s ‘Number (6.02 x 10”) of its smallest part. Given Mass(in zm) Atomic Ma No.of mole (Forelemets) No. of atoms given Na Given Mass in gm. No.of mole = Molecular Mass. (For compounds) No. of molecules Na Conversion of Mole into Atoms, lons, Molecules, Electrons, Protons or Neutrons xAomiciy [Molecules or] Formula units % y ‘ alz 2 38 Q Mole ais = aie 7 Mole of any gas at STP = 224 litre Where SATP = 246 lite = Element C = Compound (ionic or covalent) Na = Avogadro's No. (6.02 x 103) @ = No. of electrons: P= No. of protons Gram-Atom: One gm-atom is equivalent to one mole of that element. e.4, 6gm of carbon = 6/12 gm-atom = 0.5 gm-atom. Loschmidt Number: It is the no. of particles in 1 cm’ of a gas at STP (2.688 x 10") Equivalent Mass It is that mass of an element which either combine or displace by 1 part by mass of Hydrogen, 8 parts by mass of oxygen, 35.5 parts by mass of chlorine, 80 parts by mass of bromine etc. Atom or Molecular Mass ent Mass = — Equi n= factor Atomic Ma: Valency Stoichiometry 3 Molecular Mass, Total charge on cation or anion part 100 £4. Eeaco,= J = 50 E, compound = lonic Mass 60 onic Mass © 50 coy vows” Charge _ Molecular Mass ising oredcing 900 =p — Facto E, 158 7 ©.4. Fraino, (HT medium) = (MnO; — Mn?*) Egstso, (OH medium) = (sino, + Mn0}>) 158 Exsino, (Neutral medium) = —~ = 52.66 (Mnoj > Mno,) . __ Molecular Mass Fp eleclae Mass: acid Basicity 98 eq. Enso," Molecular Mass ‘we Acidity 1 4 64. Exuo, ~ 3» Fearon, ‘Methods of Calculating Equivalent Mass (a) H, Displacement Method Ween gn EE 51 008, Wydcogen W, lerest in gm E, x12 4, gasat STP in ive (b) Oxide Formation Method wy, lee mg E, ment Woxygen required men in gt = Semen ne 5.6 ‘0, emat STP (©) Halide Formation Method Weserent in om E, 7 * Ehatee cen ried (A) Double Decomposition method Frwsocan Boost Woonpomi —Eeomponat Meaco, _ Fewoy eg. 20% = “CaCO Woo Fao Ean + Ey E..» +30 Eqn +8 (e) Law of Isomorphism Weert in coempound | Entin compound 1 Weta in compound It which reacts with same mass ofan ‘emasin compound! Ex in compound I (F) Atomic Mass It is the relative number which state how much times 1 an atom of that element is heavier than that of th part of an atom of C'*. (Either it is unit less or ex- pressed in a.m.u) 66054 x 10 gm 66054 x 10" kg. Methods of determining Atomic Mass (a) Dulong and Petit’s Method Atomic Mass (approx.) x Specific heat = 6.4. This law is only applicable for the solid elements which do not have allotropes. e.g. B, C, Be, Li, etc. Exact atomic Mass = Eq, Mass x valency. (b) Vapour Density Method ‘Atomic Mass = Eq. Mass x Valeney Molar Mass Valeney = aency “Equivalent Mass 4 Numerical Chemistry 2x Vapour density or Valency = ¥~ “Equivalent Mass (0) Law of Isomorphism For two isomorphous compounds: Wrsatincompounat Atomic Mass of metal in compound I © “Atomic Mass of metal in compound II ‘stain compound 1 ht resste wt sme mas bf anion asin compound | Molecular Mass: It is the relative mass of one molecule of a compound which state that a molecule 1 is how much times heavier than that of {th part of an atom of C?, Methods of determi: ing molecular Mass (a) Vapour Density Method (For volatile substance) Molecular Mass Where vapour density is determined by Duma’s ‘method or victor-Meyer’s method. (b) Diffusion Method According to Graham’s law % Vapour density Where rate of diffusion of gas having ‘molecular mass M, 1, = rate of diffusion of gas having molecular mass M, GMM (Gram Molecular Mass): Molecular mass ex- pressed in gm is called gram-molecular mass (No. of molecules in one GMM = 6.02 x 10") C Limiting Reagent ‘The reactant which consume or react completely dur- ing a chemical reaction is called Limiting Reagent, Any calculation for the product's must be on the ba- sis of Limiting Reagent. Let us consider a chemical reaction: Let us suppose the formation of H,O Ht@ + Lo, HO Cig Mole (B.R) 1 0.5 0 Mass (B.R) 2 16 0 It means 2 gm H, reacts with 16 gm O, to form 18 gm HO. If both H and O, are given in equal amount, it is oxygen which react completely and hence it is limiting reagent in that instance and H behavs as excess reagent. Concentration Units Molarity (M) Itis the number of moles of solute per litre of solution Weoture X 1000 Mi. x sone * Vecdation in ml Molality (m) Itis the number of moles of solute per kg. of solvent. Wesiwe % 1000 Mesure XM%s Solvent in em ite Relation between M and m a M sone 1000” m Ise = Molecular Mass of solute d= density of solvent Normality (N) It is the number of equivalents of solute per litre of solution. y= Movie 1000 E ote * Vso nl Relation between Normality and Molarity Normality = Molarity x (Molar Mass/Eq. Mass) N=nM Stoichiometry 5 Where 1 = factor or it is the number which di- vided by mol. mass to obtain equivalent mass. of that species Percentage Strength oie *100 Wctaion + % by mass (W! i.e. both solute and solution are expressed in mass. Veoiune 100 © % by volume (V/N) = Seite Sotto cc. both solute and solution are expressed in volume Weotune *100 © %by weight-volume (W/V) = “Sata 100 ice. solute is expressed in mass while solution is expressed in volume. Mole Fraction The fractional mole of either solute or solvent per mole of solution. Mole fraction of solute _ _Mole of Solute (Keone) Mole of Solution Mole fraction of solvent Mole of Solvent Mole of Solution (Korres) Fora binary system Xicjge+ Xen =! Xsotate Xx, Mole of Solute ‘Mole of Solvent Formality (F) The number of formula units per litre of solution. Wage % 1000 F- Formula Mass of solute X Veciion in mt Formula mass is only for ionic compound. Ionic Strength The strength of a solution when two or more ionic compounds are mixed together is called Tonic Strength. of that solution it is calculated as Tonic Strength = 5M (Where M=Molarity and Z (CHotumetric Analy It is the quantitative analysis by measuring volume of the reacting substance, on the basis of neutralisation (seen by using indicators), falency). Titration The process by which the strength of the unknown solution is determined with the help of a standard so- lution, is called Titration. (a) Acidimetry: To find out the strength of acid with the help of standard base is called acidimetry. (b) Alkalimetry: To find out the strength of alkali with the help of standard acid is called alkalimetry. ‘Types of Titration (a) Acid-Base Titration (i) Strong Acid Vs Strong Base (ii) Strong Acid Vs Weak Base (iii) Weak Acid Vs Strong Base (iv) Weak Acid Vs Weak base (b) Redox Titration (i) Permagnetometry (ii) Dichrometry (iii) lodometry (iv) Iodimetry (©) Precipitation Titration (d) Gravimetric Titration (e) Potentiometric Titration. Law of equivalence Whenever a substance react or to which extent it re- act, with another substance, the equivalent of both the substances are the same. ie. NV, = N,V. No. of Gram Equivalent = Nx Vin litre No. of milli equivalent (m.eq.) = Nx Vin em’, meg 100 W, sabtance in gm xE, substance 6 Numerical Chemistry Strength of solutions or Mixtures (a) When a number of acids or bases are added, NV = NiV, + NV+ NgVy + Where V,= Vi) (Total volume) Wat V+ Ng = Normality of mixture (b) When acids and bases are mixed together Nalin (NV, Naa +) = (NK EMH +) ‘Where N,N, --- are normalities of acid or base having volume Vy, V3 respectively ht Nj, Nj + are the normalities of base or acid having volume ¥,V3 --- respectively. Double Indicator Titration Whenever two alkali mixtures such as Na,CO, and NaHCO, or NaOH and Na,CO, or NaOH and NaHCO, are mixed and their composition is determined by using methylorangeorphenolphthalein indicator, the only fact you have to remember is that phenolphthalein changed its colour as soon as CO, or SO, starts forming during a reaction, But methyl orange shows complete neutralisation, c.g Neutralisation of NaOH and NaxCO, mixture by HC] is shown as follows (a) NaOH + HCl > NaCl + H,0 (b) NaC, + HCl - NaHCO, + NaCl (©) NaHCO, + HCI NaCl + H,0 + CO, Methyl orange colour change is due to complete Neutralisation by a, 6 and c. But phenolphthalein colour change is due to only @ and b (Concepts of H,0, Solution HO, decomposes to give O, gas as 2H,0, + 2H,0 +0, The part of O, gas at STP is produced by the de- composition of one part of H,0, in called volume strength of H,O,. It is represented by *V” in inverted comma such as *SV°, *LOV’, etc. 7 Percent strength of HO, = = x Volume strength, re 20r= 56 gt Volume Strength = 5.6 x Normality Volume Strength = 11.2 x Molarity Concept of Oleum Oleum is H,S,0, ie. H,SO, + SO, ie. when SO, gas is saturated in H,SO, solution, it is called oleum. Its percentage in terms of H,SO, is always more than 100 %. The extra % is the mass of HO needed in or- der to react with SO, to form H,SO, by the reaction so, + HO + H,S0, Mole> 1 1 1 Mass > 80 gm 18 gm 98 gm For example: 109 % oleum means 9 gm H,0 is re- quired to react with 40 gm SO, to form H,SO,, Hence % of free SO, = 40 and H,SO, (%) in oleum is 60. Hardness of Water # If water does not produce or produce a little-bit froth, with soap, then water is called hard. ‘* Hardness of water are two types. Temporary and Permanent. Temporary hardness is due to presence of bicarbonates of Ca and Mg while permanent hardness is due to presence of solute chlorides and sulphates of Ca ang Mg. ‘© Hardness is expressed in term of CaCO, i.e. no. of parts of CaCO, present per million parts by weight ‘of HO (10° parts) Mass of CaCO. 10° gm of 1,0 I IT WINDOW-) l > SE [> Bxampte 1 10 gm of a metal form 12 gm of its oxide on heating in air. Calculate the equivalent mass of that metal.” Hardness in ppm = > Solution = Mose W x8 Stoichiometry 7 _ 10x8 _ 10x8 (2-10) 2 > Example 1 How much volume of water is required to dilute =40 Ans. N 500 ml. of 5- HCI into decinormal? > Solution For dilution of any solution m.eq. of the solution in both the cases are the same. i.e NV = Nab 1 Lai = ¥. 3 7% 500 = 55a = y= 2500 em Hence volume of water added (2500 ~ 500) = 2000 ml or em’ Ans. > Example 1 Calculate the no. of mole, molecules, atoms and electrons in 3.0 gm of C,H,.? > Solution No. of mole of GH, = 5 = 0.1 mole No. of molecules. = 0.1 x 6.02 x 10” 02 x 10 molecules, 1x 6.02 x 103 x8 = 4.816 x 10 atoms. 1x 6.02 x10" x 18 = 10.836 x 10” electrons. [> Example 14 How many atoms, ions and electrons are present in 1.06 gm of NaCO,? > Solution Mass of Na,CO, given 06 gm Molecular Mass of Na,CO, 23x2+12+16x3 = 106 No. of atoms No. of electrons 1.06 No, of mole of Na,CO,= +57 = 0.01 mole Na,CO, ionises as Na,CO; =2 Na’ + CO} No. of formula units of Na,CO, = 0.01 x 6,02 x 10° = 6.02 x 10" No. of atoms (Total) = 6.02 x10" x 6 =3.612 x 10? No. of total ions = 6.02 x 10"! x 3 (cations & anions) = 18.069 x 10" Ans, [> Example 1 From 200 mg of CO,, 10°! molecules of CO, are removed. Calculatetheno.ofmoleofCO, lefibehind.? > Solution = 4.545 x10° 5 107! Mole of CO, left = 4.545 x 10 - ———- ° 6.02 x 107 = 4.545 x 10 ~ 1.66 x 10 = 2.885 x 107 mole Ans. [> Bamptet Calculate the equivalent Mass of FeS in the following reaction.? FeS +0, —*5 Fe,0; + SO, » Solution In order to calculate equivalent mass of a substance, Let us calculate the no. of overall electrons required as Fe > Fe* +e SoS" +607 S—Fe +84 470 Mol. Mass Thus, Eye = > Ans. 8 Numerical Chemistry > Example 1.7 Calculate the equivalent Mass of KMnO, if following change is observed.? 3. MnO; — Mn?" +2 MnO, > Solution It is a disproportion reaction hence equivalent mass is calculated by calculating the overall change in oxi- dation state per molecule ‘Change in Oxidation State [Lx2-1x7|+[2x4—2%7] -7|+|8-14|=5+6=11 1 Change in O.S per molecule = Exo, i this reactio 1 3 (Where M= Mol. Mass of KMnO,) > Example 1.8 Calculate the strenght of 20 °V" HO, in gm/litre. > Solution 20°V"H,O, means ‘I” part HO, produce 20 parts ‘Oy at STP. 34 AS Exo, => 717 Using equation, volume strength =5.6xN strength in gm /litre Eq. Ma +. strength in gm/itre = 60.86 Ans, [> fampets — —————C—S— 18.0 gm glucose present in 198 gm of its syrup hav- ing density 1.05 gm/m’, Calculate molarity (M) mo- lality (m) and mole fraction of solute.? > Solution Glucose (C,H,0¢ 80 2 _ —Wemcose ¥ 1000 ee aac ™ Vain 181000 «1.08 Mu 180 x 198 =0.53M Weotacose 1000 i eT _ 18x 1000 180 x (198-18) =0.55m ‘Mole Fraction of Glucose Memsose Metucose * M0 18 - —1s0__ _ _ 0.1 18, 180 ~ 0.1410 180" 18, = 0.00990 Ans. [> Example 1.10 ‘What is the molarity of water at 4°C? (LIT, 2003) > Solution As we know water has density 1 gm ml’ (Maxi- mum) at 4°C. Motarity (4 = se 21000 ony Msotute * Vsotation = = 10001000 55 56 mole"! Ans. 18x 1000 [> Bxampte 1.17 ‘A plant virus is found to consists of uniform cylindrical particles of 150 A° in diameter and 5000 A® long. The specific volume of the virus is 0.75 cm) gm. If the virus is considered to be a single particle, find its molor mass. (IIT, 1999) > Solution Since the virus is cylindrical its volume = 7777) =3.14x (Bato) x 5000 x 10-*em* Stoichiometry 9 = 8.8312 x 107 em’ 8.831071 x1 Mass of one virus = “T= gm = 118 x 10° gm + Molecular Mass of Virus = 1.18 «10° x 6.023 x 105 107 x 10" gm mole “' Ans. > Example 1.12 If 200 gm of 20 % and 300 gm of 25 % solution of HCI are mixed together. Calculate the composition of mixture in terms of mass.? > Solution Mass percent means mass of solute per 100 gm of solution. 20x 200 in 20% = =40 gn Mics in 20 % solution = —T— = 40 gm 25x 300 in 25 % solution = - Wey in 25 % solution = gm Total Wye) = 40 + 75 = 115 gm Total Mass of Solution = 200 + 300 = 500. Mass % in the mixture = > x 100 500 = 23% Ans. > Exam, 3 Calculate the percentage of Fe” and Fe’ in Fey 9,0. (Anon-stoichiometric oxide). > Solution Let overall oxidation state of Fe in Fe, .,0 is.x. Then 0.94 xx-2=0 x= —— * * 994 Again suppose the % of Fe” is x’ and hence % of Fe’ is 100-2" Thus, aug 00 ¥x2+(100-x)x3 [> Example 1.14 20 gm of a mixture of Na,CO, and CaCO, is heated to aconstant mass 1.5 gm. Calculate the composition of mixture? > Solution Since Na,CO, does not decompose on heating, it is only CaCO, that decomposes as CaCO, (3) +> Ca0(s) + CO,(g) T And loss of mass is due to loss of CO, gas. caco, > Ca0 + CO, 100) So” as Massof¢,co,=1-136gmand Massofy,,co,=0.864gm I Example 1.15 6.0 gmsample of CaCO, reacts with 20 gm solution of HCI having 20% by mass of HCI (density = 1.10 gm/ ml). Calculate percentage purity of CaCO, sample.? > Solution 20 ; Viict stuion = 1 = 18.18 em’ Nigga OM Ae ad Mycy XViyc) 36.5 x 18.18 (Since 20 % of 20 gm=4 gm.) Meg. of HCl = M.eq. of CaCO, 4x 1000 = AX1000 18.18 = 109.6 36.5 18.18 * M.cq. of CaCO 1000 109.6 100 _ = 6 0s 1000 “2 $48 em c3C0; ‘eco, 10 Numerical Chemistry % Purity of sample = 8, 100 = 91.33 % > Example 1.16 7M +M oM 100 cm’ 2 HCI, 200 cm’ 3 H,SO,, 200 cm’ Ti M ‘NaOH and 100 cm? “y Ca(OH), are mixed together. Calculate the normality of mixture and its nature? > Solution HCI (N = MM), HySO, (V=2 M) ‘NaOH (= M) and Ca(OH), (N= 2M), VigNgg= (VN, + VaNg + >) ~ (WINE +VENL +) Where Fy = (V+ Fy ++ IN} +) Acid 600N, (100 x 3 + 2002) Ym 2 3) Base = (2005-4 +1001 10 = (50 + 80) (20+ 100) = 130-120 =10 104 600 60 Since M.eq. of Acid is more, the nature of solution is acidic. Ans, > Example 1.17 2 ; 3M 105 em® FeSO, solution reacts with 150 em? of — KMnO, solution in acidic medium, Calculate the strength of FeSO, in gm litre”. [Fe = 56, S = 32, O= 16] > Solution For KMnO, (acid medium), M.eq. of FeSO, is equal to M.eq. of KMnO, Meg, of KMn0,, vx Vin ml Meg, of FeSO, Meq = OE x Exeso, FeS0, = Yqqq * EFS 15. 152 2 Weeso,= Toa XSi 2 vo 80. Topp * 7 (Sinee Fe" is converted into Fe’, hence s) Weeso, = 2.28 gm Strength in gm litre” [> Example 1.78 M 110 cm’ of K,Cr,0, solution having molarity 21.71 Ans. reacts with sufficient amount of HCI solution to liberate Cl, gas along with other products. Calculate the volume of Cl; gas at STP liberates.? > Solution Ex.cxp, in acidic mediun Hence N= 6M M.eq, of K,Cr,0, = M.eq, of Cl, 1 7p *110 = Meg, of Cl Volume of Cl, gas at STP _ 10x 35.5% 22.4 12x 1000 x71 [> Baample 1.79 50 ml of a mixture of NaOH and Na,CO, diluted to = 0.1026 litre Ans. 200 ml, 25 ml of this solution require 12 ml x HCL a You have either reached 2 page thts unevalale fer vowing or reached your ievina tit for his book. 12 Numerical Chemistry 1 10. 1B. 14. (a) 2 mole (b) I mole (©) 0.5 mole (4) 0.25 mole An ionic compound is represented by the formula M,N,, the total no, of ions formed is equal to (a) Me (b) (c) Me (@) xty . A sentence contains 35 alphabets and written ina line, One page contain 35 lines. The mass of carbon needed to write this page if one al- phabet contains 10'° carbon atoms is approxi- mately (a) 2.4410? gm —(b) 2.44 x 107 gm (c) 244x107 gm (d) 2.44 10 gm A sample of lime stone have 75 % purity. If 5 gm sample is used, the no. of Ca formula units present is: (a) 6.02 x 10% (bd) 6.02 x 10 (c) 4x 10 (@) 4x 10" If 10” molecules of CH, is effused out from a vessel in which 16.0 gm CH, is taken, the no. of molecules of CH, in the vessel is: (a) 6.02 x 10! (b) 60.2 x 10! (©) 59.2 107 (@) 59.2 If a person inhale 10°° oxygen atom per sec. The volume of oxygen gas (O,) inhaled by the person in a day at STP is: (a) 16.06 litre (b) 16.06 x 10° litre (©) 16.06 x 107 litre (d) 16.06 x 10' litre The density of CCl, vapour in gm/L at STP will be (a) 3.425 (b) 6.875 (©) 10.252 (@) 4.575 Ifair contains 21 % 0, by volume. The num- ber of atoms of oxygen per litre of air at STP is: (a) 3.810" (b) 0.38 x 10" (c) 38 x10 (d) None of these How many moles of electrons weigh one kilo- gram (a) 6.02 «10 Lag 6) x10 ©) 9.108 6.02 9.108 ——| ny 5108 x 6.022 x10 © (da) 10° {HIT, 2002] 15, The number of atoms of Na in 5 a.m.u of it is? [Na=23] (a) 23 (b) 5 (c) 5/23 (d) 23/5 16. The number of electrons present in 5.6 litre of H, gas at S.P. is? (a) 6.02 «10° (b) 3.012 x 10° (c) 12.04 x 107 (@) 3.0x10"" 17. The total cationic charge in coulomb present in 46 gm of Al,O, is? (a) 26.07 x 10°C (b) 2.607 x 10°C (©) 260.7x10°C — (d) 26x 108 18. The number of ©, molecule and its volume at S.TP in 50.6 gm of hydrated sodium carbon- ate ore: (a) 6.926 x 10", 22.4 litre (b) 5.92 x 1071, 25.76 litre (c) 6.926 x 10, 25.76 litre (d) $.92 x 10", 22.4 litre 19. The volume of CO, contains the same no. of molecules as in 5 gm of CH, is (a) Slitre of CO, (b) 7 litre of CO, (©) Mitre of CO, (a) 3 litre of CO, 10.0 gm of CO on buring in air gives the prod- uct which have the number of nucleons (a) 9.47 x 10° (b) 94.7 x 10° (c) 947 x 10°? (a) None 20. HINTS AND SOLUTION: 1IT WINDOW-I > Dress (with one correct option) 1. No. of atoms in 10 gm CaCO, = to 6.0210" x5 100 = 3.01 x 10 atoms Stoichiometry 13 | No. of molecules in 1.6 gm CH, = No. of atoms in 2.24 litre of CO, = 224 6.02108 x3 24 1.86 x 10° atoms No. of atoms in 2 mole of Hy = 2x 6.02 x10" x2 = 12.08 x 10 atoms No. of atoms in 5 gm-atom of Na,CO, = 5 x 6.02 x 10° x 6 = 18.07 x 10 atoms No. of gm-atom (mole) of Na= 2x10 6.02 x 10™ = 033 No. of molecules in 0.1 mole of CO, =0.1 XN. IN, No. of molecules in 11.2 litre of CO, at STP 2 = SSK, = 22.4 O5Na No. of molecules in 22 gm CO. =0.5 Ny No. of molecules in 22.4 x 10° ml CO, at STP. _ 24x10 2.24 x 10° Lo TaXNa =O.1Ny, No. of molecules in 22.4 litre of CH, at S.T.P _ 2A “224 =N No. of molecules in 2-gm-mole of CH, =2N, No. of molecules in 22400 em? CH, at S.TP. 22400 ~ 224008 "8 Mass of H,O = Volume x density = 0.06 x 1 = 0.06 gm/nl No, of molecules of H. oe xNq = 3.33 x 107 x 6.02 x 10” =2x 10" Is 10. u. 12. 13. 2AL + 3/20, + — ALO, I mole 2x27+16x3 5 mole of AL,O, MN, =xM" + yN™ Total no. of ions = x+y, Total No. of alphabets in a page = 35 x 35 225 Total No. of carbon atoms = 1225 x 10'° atoms, 1225 x 10'° 6.02 10 = 2.44 107 gm Mass of carbon = x12 gm ‘ 5x15 Mass of pure CaO in the sample = 100 =3.75 gm 3.15 No, of formula units = => xNq 14 x 10 16.0 gm CH, = 6.02 x 10” molecules of CH, No. of CH, molecules left = 6.02 x 10°? — 10 Y (6.0210 |) = 59.2 x 10? No. of oxygen atom inhaled per day = 10" x 60 x 60 x 24 atoms No of molecules of O, gas a 1 = 10" x 60 x 60 x 24 x > = 4.32 x 10 molecules Volume of ©, gas at STP 22.4 litre = 16.04 x 10! litre. density ~ _MASs__ 1243554 mnsty™ Volume 2. = 6.875 gmilitre Volume of O, per litre of air = 0.71 litre No. of “0” atom per litre = O71 6.02 x10 x2 24 = 0.38 x 10" 14_ Numerical Chemistry 14. 16. 17. 18. 19. Mass of 1 electron = 9.108 x 10°! kg. i => 9.108 x 10°! kg = ———- mole of 6.023 x10" electron 1 1 kg = ————_________ mole 6.023 X 9.108 x10" x10 ofe = _ seer 6.023 9.108 Oe OF* | ama = one twelfth the mass of one atom of c Na has atomic mass 23 i.e. one atom of Na weighs 23 amu. No. of atoms in 5 amu = 5/23 atom No. of electrons in 5.6 litre of H, at S.T.P 56 2 = =" x 6.02310" x2 24 (Since 1 molecule of H = Ze") = 3.012 x10" 1,0, =27x2+ 163 = 102 Total cationic charge = 8 5.6.02 10 «23x 1.6 x10" 102 = 26.07 x 10% coulomb. Hydrated Sodium Carbonate=Na,CO,-10H,0 Its molecular mass = 106 + 180 = 286 50.6 No. of mole= 55> No. of ©} molecule in the above mole 50.6 2 13 = Sx 6.02 x10" x= molecules Tyg 002 x10" x molecule = 6.926 x 10” molecules ‘Volume of O, gas of S.T.P 22.4 litre = 25.76 litre No. of molecules in 5 gm CH, = =x 6.02 x10 16 20. : ax6 0210 molecules of CO, 5x 6.02 x10? canine 8224 lite of CO, 6 x 6.02 x 10" =7 litre of CO, co+ 1, 4 00, I mole I mole 28 gm 44am 44 10 gm CO= x10 gmco, gm 3g * 10 gmco, 44x10 28x44 mole CO 44x10 23 = x 6.02 10" 2 ones molecules of CO, = BMS HIT 4 wactcons 28x44 94,647 x 10° nucleons: (Total and p which are present in the nucleus are called nucleons.) [> @ oN EQUIVALENT CONCEPT L 2. 2 4 ‘quivalent mass of a metal, x gm of which re- acts with I-gm-equivalent of an acid is? (a) 1 (b) x2 (©) 2 @x ‘An element (x) having equivalent mass *E” forms a general oxide X,, O,,. The atomic mass of an element should be? (a) 74 () 2m, ME (©) Bin Oo, 5.0 gm of a metal reacts with oxygen to form 5.6 gm of its oxide. The eq. Mass of metal is? (a) 33.33 (b) 66.66 (©) 99.99 (@) 44.44 8.6 gm ofan element when treated with excess of acid produce 1900 ml of H, gas at $.T.P. ‘The equivalent mass of metal is? (a) 48.69 (b) 58.69 (©) 50.69 (@) 56.69 Stoichiometry 15 6 € 10. i. 12. . For the reaction N, + 3H = 2NH,, if molee- ular mass of NH; and N, are M, and M,, their equivalent masses are E, and E, respectively, then E, ~ Eis: 2M, —M, fa) 3 (b) M,-M, (©) 3M,-_M, (@) M-3M, In the reaction FeS, + KMnO, +H" — Fe''+ SO, + Mn™* + H,0 The equivalent mass of FeS, would be equal eS Molar Mass (a) Molar Mass oo Molar Mass Molar Mass oy OS Equivalent mass of S in SCI, is 16. What is the equivalent weight of S in S,Cl, (S = 32, C1= 35.5) (a) 16 (b) 32 (©) 8 (a) 64 The equivalent weight of MnSO, is half of its molecular weight when it is converted to (a) Mn,0, (b) MnO, (©) MnO, (@) Mno [1IT, 1988] 14.2 gm of M,CO, on heating produce 8.6 gm of MO. The equivalent mass of M is (a) 25.78 (b) 21.23 (©) 27.9 (a) 32.5 0.65 gm Zn treated with H,SO, to form its sulphate with the liberation of H, gas. The ZnSO, on strong heating produce 0.81 gm of its oxide. The equivalent mass of Zn is: (a) 65 (b) 32.5 (©) 16.25 (@) 36 The ratio of the equivalent mass of KMnO, in acidic and alkaline medium is fa) 1 (b) uo (d) © © ‘The equivalent Mass of MnO, in the follow- ing reaction is: 3MnO, - 2Mn’* + MnO, 3M 3M oa Oa 13M M o> @ 13, The equivalent mass of HBO, in its with NaOH to from Na,B,0, is equal to (a) Mia (b) M3 (©) M2 (@M 14, 6.35 gm of Cu is displaced by a metal (2.9 gm) when added in its sulphate solution. If equivalent mass of Cu is 31.5, The equivalent mass of metal is: (a) 19.5 (b) 144 (c) 248 (@) 205 15, 2 gm metal \X° on reaction with sufficient Acid produce 1120 cm’ H, gas at STP. Its equiva- Tent mass is: (a) 10 (b) 20 (©) 30 @) 40 HINTS AND SOLUTION: ON EQUIVALENT CONCEPT 1, That mass of metal is called equivalent mass which reacts with 1-gm-equivalent of an acid. 2. As we know, ‘Atomic M Equivalent Mass of x= “tome Mass Valeney Atomic Mass = B= => Atomic Mass = En. Since oxygen have valency 2, hence m= 2 2En m = En 3. Equivalent Mass Mass of metal = see x8 Mass of oxygen > x8 = 66.66 06 4. Equivalent Mass 16 Numerical Chemistry 6 i i. M M 5. ENE) = EQN.) = Mass of metal = ess of met x12 Vol. of H gas at STP.(litre) 86 = 2 x112 = 50.69 19 2 3 2M, Fey+ Fe +e (S,)? > 28* +1007 FeS, > Fe** +28 +1le™ Eres a (= Molar Mass of FeS,) Inscl,, One mole of S combine with 2 mole of Cand 1 equivalent of S = 1 mole of CL oe Eg =1x355=35.5 InS.Cl, 2 mole of C1 =2 mole of § 1 equivalent of Cl= | mole of S E,=1x32=32 MnSO}* + MnO3* Change in oxidation state of Mn = 2 Exo, (acidic) ‘xiao, (Alkaline) 5 12. 3MnO; 2 Mn** +MnO, Change in 0.8, =|2 x2-2%7|+|1 x4—1 x7] =|4-14)+/4-71 =10+3=13 Change in O.S per molecule = — 3M ‘Mio, = 73 13. 4H}°BO, + 2NaOH —> Na,Bj°0, +7H,0, Change in 0.$ of Beron is not observed hence M En.p05= 15. Enea _ Mass of metal iT800 Vol of H, gas at STP(in ml) 2 —— x 11200 =2 1120 . E, mM. Mos, = “2 Wt.of M,CO, — Eu.co, Wtof MO Eyio 142 _ By +30 86 Ey t8 0.65 O81 Ex, Eq, +8 = nS iy Exaao, (acidic) = SO Mol. Mass Exx4yo, (alkaline) = > (C) ON Atomic and Molecular Mass 1. 30 gm of an oxide of divalent metal is con- verted to its chloride completely. The mass of chloride is found to be 5.0 gm. The atomic ‘mass of metal is? (a) 33.25 (b) 66.5 (©) 99.75 (@) 40.25 2, 2.492 em of ZnO is produced when 2 gm pure Zn metal is oxidised in air. If the atomic mass of oxygen is 16, What will the atomic mass of mm? (@) 63.0 (©) 64.0 (b) 65.0 (@) 66.0 Stoichiometry 17 1 . 5 gm of a pure metal produce 5.8 g1 Chlorine exist in two isotopic forms CI’ and CL”. The ratio of the percentage of Cl and C1” is? [If average atomic Mass is given as 35.5 amu] fa) Vel (b) 2:1 (3:1 () 4:5 |. 216.0 gm of metal oxide when reduced by carbon produce metal along with a poisone- ‘ous gas which on passing through ammonical cuprous chloride, the rise in mass observed is. 28.0 gm. What is the atomic mass of metal? (a) 64.0 (b) 200.0 (c) 216.0 (d) 244.0 2.6 gm of metal when treated with 1120 cm’ ©, at S.T.P. and the product when passed through alkaline pyragallol solution, its mass increases by 0.5 gm. The Atomic Mass of ‘metal is? (a) 41.6 (b) 36.5 (c) 35.5 (d) 40.5 5.6 gm of iron is oxidised by 4.4 gm of CO, in aclosed retort at 450°C into Fe” and a poiso- neous gas is released. If atomic mass of oxy- gen is 16 the atomic mass of Fe is: (a) 28 (b) 32 (c) 56 (d) 58 ‘Vapour density of a chloride of non metal is 104.25. The non metal is a member of 15th ‘group. The non metal is? @N ) P (©) As (a) Sb MS and Hg,S are isomorphous with each other and the percentage of S in both the com- pounds are respectively 20 and 8 % respec- tively. If atomic Mass of Hg is 200.0, atomic mass of *M’ is: (a) 72.52 (c) 168.6 (b) 61.28 (d) 69.56 of its oxide on heating in air. If the specific heat of metal is 0.06 cal/gm. Its exact atomic mass would be: (a) 106.66 (b) 50 (©) 100 (d) 90 Two elements A and B combine to form two compounds A,B, and AB;, If 0.2 mole of both 12. 13. 15. the compound weighs 20 gm and 15.0 gm re- spectively. The atomic mass of A and B are: (a) 40. & 25 (b) 25 & 16.66 (©) 25.2.&16.0 (A) 20.22. & 18.0 0.5 gm of a gas have volume 250 cm’ at 600 mm of Hg at 300 k. Its molecular mass would be: (a) 48.2 (b) 62.4 (©) 56.28 @) 72.2 Duma’s bulb is filled with a compound at 27°C and 720 mm of Hg weighs 0.5 gm. The molecular mass of compound is (a) 92.0 (b) 108 (©) 118 (@) 120 0.607 gm of a silver salt of tribasic acid is re- duced to 0.370 gm pure silver. The molecular mass of acid would be: (Ag = 108) [AIEEE *08] (a) 210.0 (b) 180.0 (©) 192.0 (@) 160.0 0.15 gm of @ volatile compound taken in a Victor-Meyer apparatus and it displaces 40 cm’ of air measured at 27°C and 720 mm of Hg. if aq. tension at 27° C is 10 mm Hg, the molecular mass of compound is: (a) 198.8 (b) 160.2 (©) 140.6 (@) 98.8 Chlorophyll contains 0,025 % Mg by mass. Then the probable molecular mass of chloro- phyll is: (If one molecule of chlorophyll con- tain one Mg atom). (a) 96000 (©) 25000 (b) 56000 (d) 12696 HINTS AND SOLUTION: ON ATOMIC AND MOLECULAR MASS Woo. Eu tEox Wwe, 3.0 > 30 ‘Atomic Mass = Eq. Mass x Valeney = 33.25 x2= 66.5 18 Numerical Chemistry Wr Atomic Mass of Zn Woryen Atomic Mass of oxygen 20 _ x ~ jas 7 1g 778504 Atomic Mass of Zn = 65.04 3. Let % of CI =x, -. % of CP” = 100—x Then 100 x 35.5 =x x 35 + (100 —x) 37 = x= 15% % of CL = 75, & % of CP” = 25 wer _ 75 wer” 25 4. CO is absorbed in ammonical CuCl, solution and increase in mass is the mass of CO. Mass of CO = 28 gm Mass of oxygen in it = 16 gm Wyreatoxide _ Atomic Mass of Metal Woxyyen Atomic Mass of oxygen 216-16 _ Atomic Mass of Metal 16 16 =} Atomic Mass of metal = 200.0 5. The increase of mass of alkaline pyragallol is, due to absorption of on reacted O, gas. 32x 1120 22400 Total Mass of O, 5 gm Mass of , reacted with metal = 1.5 — 0.5 = 1.0 gm Atomic Mass of metal ‘Atomic Mass of O, Atomie Mass of metal Given 5.6 0 44 12 Eq. Mass of Fe= 7 x8 =28 Atomic Mass of Fe = 2 x 28 = 56 [Since change in O.S of Fe =2] 7. Since non metal is a member of 15th group, it ‘means its valency is either 3 or S and Molecular Mass is twice of V.D. so, it must be 208.5 gm/ mole. It is possible only when valency is 5. MCI, = Atomic Mass of M+ 5x 35.5 = 208.5 = Atomic Mass of M+ 535.5 Atomic Mass = 31, (It must be P). 8. For Isomorphous compounds Wt. of Metal in compound Ist Wt. of mental in compound IInd that reacts with same mass of anion as in Ist © At Mass of metal I 2 200 > 32 ~ Atomic Mass of metal "M* 200 x 32 92 = 69.56 64 64 sp.heat 0.06 10668 => Atomic Mass of metal ‘M’ = 9, Approx. at mass = x8 = 50. 106.66 10. According to question, 205 =25 and) When x= Atomic Mass of 4, of B. 11 use PV= © rr. M = M-“ er PV _ 05% 00821300 “600 250 760 ~ 1000 = 62.4. Ans. Stoichiometry 19 12. Aswe know PY _ Pe TT :. Volume of vapour at S.T.P 720 x 120 x 273 300 x 760 = 103.45 em? = 108.26 gnvmole i Majer 3% Savers Atomic Massof Ag Mol. Mass of salt 037 30.607 ‘108 Mol. Mass of Ag salt => Mol. Mass of silver salt Mol. Mass of tribasic aci 31.0 531 -3244+3 =210. 14, Press of dry gas = (720~10)=710 m.m. of Hg Volume of air at STP is calculated by PY _ PVs TT PV XT; _ 710%40%273 = vy-32 - meee? T)xR 300x760 = 34cm’ 34 cm? = 0.15 gm volalite substance rT 22400 em| otsxz2400 = 98.8 Ans. 15, Chlorophyll contain 0.025 % Mg by Mass 0.025 gm Mg = 100 gm chlorophyll 100 x 24 24 gm Mg= —75-~ = 96000 gm 96000 gm/mole 1. The volume of H,O added in order to convert 50 30N em’ of > H,SO, solution into decinormal i 2. 5. 6. 7. (a) 200 em’ (b) 250.cm* (©) 100 cm* (@) 50cm’ Fuming HNO, have specific gravity 1.90 and it is 92 % pure by mass. The volume of this sample HNO, required to prepare 250 ml zg solution is: (a) 2.5em* (b) 2.0 em? (c) 1.8em* (@) 13cm? Molarity of 560 gm pure H,O at 4° Cis: (a) 40 (4 (©) 55.5 @ 445 ‘The mole fraction of solute in 1 molal aqueous solution (a) 0.0176 (b) 18 (©) 0.05 (@) 0.92 0.7 gof Na,CO,:xH,0 is dissolved in 100 em’ of HO, 5.0 em’ of which required 19.8 ml of 0.1 N HCL. The value of x is? (@4 (b) 3 (2 @1 M 100 em’ M/2 H,SO,, 200 ml = HINO, and 100 cm’ of 4 HCI are mixed together. The normality of mixture is: (a) 05 (c) 0.3 () 0.2 (@ 04 M M 200 ml [> HINOs, 100 ml [ NaOH, 200 mt M “Z_HCI are mixed together. The normality and the nature of mixture is: (a) 0.28, basic (b) 0.28, acidic (c) 0.32, acidic (d) 0.32, basie ‘The mass of 60% HCI required for the neu- tralisation of 10L of 0.IMKOH is: (@) 60.82 (b) 21.9 gi (©) 100 gu (@) 219 gn 20 Numerical Chemisty 9. 50 ml of 10 N H,SO,, 25 ml of 12 N HCl and 40 ml of SN HNO, were mixed together and the volume of mixture was made up to 1000 ml by adding water. The normality of result- ing solution is: (a) IN () 2N (c) 3N (d) 4N 10. The normality of a solution of a mixture con- taining HCI and H,SO, is N/5. 20 ml of this solution reacts with excess of AgNO; solution to give 0.287 gm of AgCl. The normality of HCI solution is: (a) 0.10 (b) 0.20 (©) 01S (a) 0.50 11. A solution mixture containing 0.5 M CaSO, 0.2 M MgCl, and 0.4 M Al,(SO,)s. The total ionic strength is’ (a) 83 (b) 9.3 (©) 103 (d) None of these 12, An aqueous solution of 6.3 g. Oxallic acid di- hydrate is made up to 250 ml, The volume of 0.1. N NaOH required to completely to neutra- lise 10 ml of this solution IT, 2001) (a) 40 ml (b) 20 ml (c) 10 ml (@ 4mi 13. How many moles of electron weighs 1 kg (a) 6.023 x 10° 6.023 x10 (b) re 9.108 x10" 9.108 1 8 d) ———— x 10" © Siosx 6023 14. 100 ml 0.1 M KMnO, solution neutralises 50 ml FeSO, solution in acidie medium. The mass of FeSO, is: [Fe = 56, $ =32, 0 = 16] (a) 5 gm (b) 6.2 gm (©) 7.6 gm (6) 8.6 gm 15, One gm Zn sample is dissolved in 250 ml 0.1 NH,SO, solution and the excess of H,SO is N neutralised by 100 ml <> NaOH solution, The % purity of sample is: (a) 25% (b) 50% (©) 5% (@) 90% 16. 100 ml a dibasic acid contains 2.94 gm acid 25 ml of this solution requires 15 ml normal NaOH for neutralisation using methyl orange as an indicator. The acid would be: (a) HCOOH (b) CH,COOH (CHjCOOH (©) COOH (@ CH—COOH COOH CH—COOH 17, How many litre of cone, HCI of specific grav- ity 1.2 gnv/ml containing 40% by strength HCL will be required to prepare 2 lite of ® solu- tion? 0 (a) 0.125 litre (b) 0.1825 litre (©) 0.525 litre (@) 0.25 litre 18. 1.25 gm of a sample of bleaching powder is dissolved in 250 cm’ of H,0 and 25 ml of this solution reacts with KI solution. The Io- aoe N dine liberated requires 12.5 em’ of 57> hypo solution. The % of available chlorine in the sample of bleaching powder is: (a) 5.68% (b) 10.25 (©) 14.2% (@) 16.8% 19, The no, of moles of KMnO, that will needed, to react completely with one mole of ferrous oxalate in acidic solution is? [IIT, 1997] (a) 3/5 (b) 2/5 (c) 4/5 @t 20. The number of moles of KMnO, that will be needed to react with one mole of sulphite ions in acidie solution is (UIT, 1997] (a) 2/5 (b) 3/5 (©) 4/5 @1 HINTS AND SOLUTION: ON VOLUMETRIC ANALYSIS 1, From the law of equivalence We have V,N, = VaNy Stoichiometry 21 1 Ll 50% 5 =x 75 V, =250.em* Volume of H,O added = 250 ~ 50 = 200 em’ 2. HINO, sample is 92% pure «. 100 gm sample contain 92 gm HNO, 100 ie Toy cm’ sample contain 92 gm HNO, 92x 1000 SE 51.90 = 27 Nuwos smale = “G3 5c100 * 27.95 From law of equivalence, We have NV, = NV, 1 50 n-o 280 Vim 395 = 1.80 em? 27.75 x¥, _ Wx 1000 Mol. Mass x Volume _ 560.x 1000 18x 560 360 cm’ I gm/em)’, (Since $60 gm because density = 55.5 4. 1 molal means 1 mole solute in 1 kg of solvent 1000 18 1000 + 18 = 0.9823 Mole fraction of solute = 1 —0.9823 = 0.0176 5, M.eq of HCI (100 ml) =5 x 19.8 x 0.1 = 9.9 99 = x53 =0: Waescos= Jog * 53 = 0.5247 gm => 0.5247 gm Na,CO, = 0.1753 gm H,0 => 106 gm Na,CO, =35.4 gm H,0. = 2 mole x2, 6. Meg, of the mixture is the sum total of the meq, of all the acids. VigNig = ViN + VaN2 + VaNy ‘Mole fraction of solvent (Xpjsan0) = 400 Ny.~ 100% 1 +200 % + 100% 160 Ng 7 aor Ny 7 04 10. = 240 100= 140 Since M.eq. of Acid is more than M.eq, of base. Thus, solution is acidic M.eq. of KOH = 10,000 x 0.1 = 1,000 = meq. of HCl 1000 2O™ 36.5 = 36.5 gm pu 1000 365 gm pure Ww 60 gm pure sample = 100 gm sample HCl 36.5 gm pure sample = Pe «36 5 em sample HCI = 60.8 gm Nogheg = iV, + NgV2 + NV Ng X15 = 50% 10+ 25 x 12+ 405 Ng = 8.70 ‘Again this solution vol. made up to 1000 ml. NVA NV; 8.7 x 115=N, x 1000 =, = 87115 “ e100 | meg. of acid = 20% = = 4 0.287 1000 _ eq, of AgCl= meq 1435 => 2 meq. of acid is not of nutralised ie. HCI. => 2meq. of HCl = Volume of HCI x N =20xN 22 Numerical Chemisty 11. Ionic Strength (u) = ; Bcz? ‘Where C is constant and Z is valency. u $(05x2 405%2*)+ (02x27 +02xP)+ (1.2 x3? +1.2 x2?) = 4(441+108+4.8)= 103 12. Mol. for of oxalic acid dihydrate COOH + 24,0} ~ 126 coon Eoratie acid = 63 Meg. of oxalic acid = M.eq. of NaOH 3 MXM AN XV 6.31000 Sant x10 =0. saxas0 <1 “O1XTa = V,= 40m. 13. Metecioa = 9-108 x 107! kg 1 kg electron = —! _- eteetron 9.108 x10 1 = 5 X mole electron 9.108 x 10™" 6.023 x 10° -_ le electr ~ 7108x6023 Me eeeon feaci Molecular Mass 14. Exajao, it acid medium = 3 N=sM Nyigo,= 5X 0-1 = 0.5 M.q. of KMnO, = M.eq. of FeSO, = 1000.5 =50 meq. 50 = Mreso,* Vreso, = 50 Neeso.= 59, =1 15. 16. 17. 18. — -50_,, Mol. Mass (FeSO,) Peso 1000 I (Since Fe** changes into Fe"). 50 = 2) 152 = i090“! = 768m Meg. of H,SO, = 250 x0. 1 = 100x— = M.egq. of NaOH ag 72 Meg, of H,SO, reacted with Zine = 25 -2=23 mea E, 29 Topo “ "8 2365 soap 70-7475 gm 0.7475 % Purity = 100 = 74.75 % M.cq. of NaOH (100 ml) = 4x 15 x 1 = 60 Meg. of NaOH = M.cq, of oxalic acid m, Moraiescid~ 1999 60 Moxaticacia 294- F294 T99%— 9 2.94 x2 x 1000 > Moaie wis” pq 7 98 For dilution N,V, = 40 L a xy - 4x2 gst = 0.1825 litre Mceq. of all the reacting components are the same, Thus, meq. of available chlorin iodine = meq, of hypo solution meq. of =x125 -05 25 m.cq, of available Cl, in 250 ml _ 0.5250 5 =5 Stoichiometry 23 Wor, Topp % 35-5 = 01775 gm o.1775 % ot available Cl, = —5—= x 100 = 142% 19. 6 KMnO, +24 HSO, + 10 FeC,0, — 3 K,SO,+ 6 MnSO, +5 Fe,[SO,} + 24 H,0 + 10CO,. +. No. of mole of KMnO, per mole of FeSO, =315, 20. 2 KMnO, + SH,SO, — K,SO, + 2MnSO, + 3H,0 + 2H,S0, +. No. of mole of KMnO, per mole of H,SO, 215, > (E) ON Chemical Calculations . The volume of air (21 % O, by volume) re- quired to burn completely 2.5 gm of acetylene gas is: (a) 25.6 litre (b) 42.5 litre (©) 51.4 litre (¢) 60.2 litre 2, 5 gm each of H, and Cl, reacts together to form maximum HCI of mass (a) SgmHCl (b) 5.14 gm HCL (©) 10 gm HCl (6) 25 gm HCI 3. The mass of HNO, needed to convert 5 gm of odine into lodic acid according to the reac- tion I, + HNO, > HIO, + NO, + H,0 (a) 12.4 gm (b) 24.8 gm (©) 0.248 gm (a) 4.96 gm 4. NH, gas is passed through 50 ml of HO at 4°C till. saturation. The density of solution now decreases to 0.8 gm/ml and if NH, is 40 % by mass in solution. Its volume will be: (a) 104 ml (b) 102 ml (©) 50m (d) 90 mt 5. Assuming that petrol is octane having density 0.8 g/m! 1.425 litre of petrol on combustion will consume: (a) 0.50 mole of 0, (b) 0.1 mole of O, (©) 0.125 mole of 0, (d) 0.2 mole of Oy 6. The percentage by volume of C,H, in the mix- ture of C,Hg, CH, and CO is 36.5. The volume 7. 10. re 12. 13. of CO, produced when 100 ml of the mixture is burnt in excess of oxygen is: (a) 17.3 ml (b) 173 ml (©) 346 ml (@) 86.5 ml The mixture of N, and H, is caused to react in a closed container to form NH,. If two mole of each components are persent after reaction, The mole of N, and H, originally present is respectively are? (a) 3 and4 (b) 3.and 5 (©) 3 and 6 (@) 4and5 Equal weights of Zn and lodine are mixed to- gether and Iodine is completely converted to Zine iodide. The fraction of the original Zine remains unreacted is: [Zn = 65] (a) 0.746 (b) 0.256 (©) 0.529 (@) 0.488 If the yield of chloroform obtainable from ac- etone and bleaching powder is 75 %. What is the weight of acetone required for producing 30 gin chloroform? (a) 40. gm (b) 19.4. gm (c) 10.92 gm (@) 145 gm 1f 0.5 mole of BaCl, is mixed with 0.2 mole of Na;PO, solution, The maximum mole (Ba,(PO,)), that can be formed is (a) 0.7 (b) 05 (©) 03 @ OL The ratio of amounts of H,S needed to pre- cipitate all the metal ions from 100 ml of | M NO, Agand 100 ml 1 M CuSO, (@) 1:2 (b) 2:1 (©) Zero (@) Infinity A partially dried clay mineral contains 8% H,O. The original sample contained 12% H,O and 45% silica. The% of silica in the partially dried sample is nearly: (a) 50% (b) 49% (©) 55% (@) 47% “7 gm Ag was dissolved in HNO, and the solution was treated with excess of NaCl, 2.87 gm of AgCl was precipitated. The value of. (a) 1.08 gm (©) 2.70 gm (b) 2.16 gm (@) 1.62 gm a You have either reached 2 page thts unevalale fer vowing or reached your ievina tit for his book. Stoichiometry 25 = 119.5 gm chloroform = 58 gm CH,COCH, 75 % product formed = 30 gm CHCI, Since 75% CHC1, = 30 gm +. 100% CHCl, = 40 gm Since 119.5 gm CHCI, =58 gm acetone 58x40 119.5 10. 3BaCl, + 2Na,PO, = 6NaCl + Ba(PO,), Initially 3 2 6 ot Given mole 05 02 ‘Thus Na,PO, is limiting reagent Hence Mole of Ba,(PO,), formed = 0.1 11, 2Ag NO, + H,S ~ Ag, + 2HNO, CuSO, + H,S > Cus + H,S0, Thus ratio of precipitate formed by same mole of Ag NO, and CuSO, is 1: 2 12, Clay contains silica, HO and impurities 40 gm CHCl, 194 gm ot %of — %of silica HO Impurities dried clay x 8% = 92-x original clay 45% 12% 43% The mass ratio of silica and impurities remain constant throughout, henee, ao 4 x =47% 13, During the reaction mass of Ag remain con- stant throughout 108 s in AgCl= (F) ON HA, of Crystallisation, Har c . 0.2 gm sample of H,O, required 10 ml of (N) KMn0, in a titration in presence of H,SO,, ‘The purity of HO, is: (a) 25% (b) 85% (©) 65% (6) 95% 2. ‘The volume strength of 1.5 N H,O, solution is: (a) 48 (b) 84 (©) 30 (a) 80 2 5. 10. ‘A Commercial bottle of H,0, is labelled as “20 V’. Its Molarity is: (a) 1.785 (b) 1.587 (©) 1.23 (@) 1.03 The strength of HO, solution which as “10 V" in the bottle is: (a) 15.18 (b) 30.36 (c) 15.28 (@) 10.28 The ratio of H,SO, prepared by free SO, and H,SO, present initially in 109 % oleum is’ (a) 0.726 (b) 0.816 (©) 0.521 (@) 0.92 If litre of H,O, produces 50 litres of O, at labelled S.TP. HO, is? (a) ‘Sov? (b) “10V" (©) sv" (@) “250V" A sample of hard water contains 96 ppm of SO} and 183 ppm of HCO} with Ca” as the only cation, How many moles of CaO will be required to remove HCO} from 1000 kg of this water. If 1000 kg of this water is treated with the amount of CaO. (Calculated above), ‘What will be the concentration above (in ppm) of residual Ca”” ion, (HIT, 1997] (a) 20 ppm (b) 40 ppm (©) 60 ppm (@) 80 ppm 10 ml of hydrocarbon gas burnt in air and on cooling volume, is reduced by 30 ml. When passed through KOH, a further 30 ml volume. decrease is observed. The molecular formula of hydrocarbon is: (@) GH, (b) CH (©) GH, (@) CH, ‘The amount of CO needed per kg of Ni in ‘Mond’s process given below is Ni+4CONi(CO), — [Ni=58] (a) 1.93 ke (b) 152 kg (©) 1.22 kg (@) LOkg CS, and Cl, in the mass ratio 1 : 2 are allowed to react according to CS, +3 Ch > CCl, +8,Ch The fraction of it left behind is? (a) 0.358 (b) 0.287 (©) 0.208 () 0.576 a You have either reached 2 page thts unevalale fer vowing or reached your ievina tit for his book. Stoichiometry 27 Fraction of CS; left = 1 ~ 0.713 = 0.287 I IIT WINDOW—II ] - 1, 44 gm of CO, contains: (a) I mole of CO, (b) 0.1 mole of CO, (©) 6.02 x 10” molecules of CO, (@) 0.1 gm-mole of CO, 2. Inthe reaction: FeS, + 0, + Fe,0, +S0, (a) Equivalent Mass of Fe’ Mol. Mass of FeS,) (b) Molar ratio of FeS, FeO, and SO, is 4 2: 8 respectively. (©) Oxidation state of iron in FeS, is +2. (d) Equivalent Mass of SO, is M3 (M = Molar Mass of SO,). 3.5 gm each of H, and O, combine to form HO. (a) 5, gm H,O (maximum) will be formed (b) 5.6 gm HO (maximum) will be formed (©) 0, behave as limiting reagent (d) H, behave as limiting reagent 4. Loschmidt number is equal to: (a) the number of particles in 1 cm’ of a gas at STP (b) the number of particles in 22.4 litre of a gas at STP. (c) 6.02 x 10" particles (d) 2.688 x 10" particles M/LI (M = -N ~N 5. 100 ml of 7 HCI, 200 ml of <> HNO, and 300 ml of = ‘NaOH are mixed together. The resulting mixture has (a) normality equal to 1/30 (b) Acidic in nature (©) 30 m.eq.of alkali neutralises 30 meg. of acid (4) 30 meq, of acid remain unreacted. 6. 1 gm Mg sample is treated with 125 ml 0.1.N HCI and the excess of HCI is neutralised by 50 ml 0.5 N NaOH completely. The correct statement is/are: (a) Mass of Mg present in the sample is 0.12 am (b) Mass of Mg sample unreacted is 0.88 em (c) % of Mg present in the sample is 12% (4) Mass of impurities present in the sample is 0.88 gm. 7. 1 gNa,CO, x H,O is dissolved in 50 ml of IM HCL The excess acid required for neutralisa- tion is 86 cm* N/2 Ca(OH),. The correct state- ment is/are: (a) Water of crystallisation of washing soda is 10. (b) M.eq, of HC! used for washing soda =7 (©) Meg, of HCI used for Ca(OH), = 43 (@) Total M.eq. of HCI = 50. 8. x gm of Na,CO, and NaOH mixture is dis- solved in 100 ml H,0. 25 mil of this solu- tion requires 20 ml of 0.05 N HCI for colour change. If methyl orange is taken 25 ml mix- ture requires 25 ml 0.05 N HCI, The correct statement is/are: (a) 0.02 gm NaOH in the mixture. (b) 0.02 gm Na,CO, is the mixture, (©) 0.0265 gm Na,CO, is the mixture. (@) 0.0565 gm Mixture is given. 20 ml ofa mixture of C,H, and CO when burnt in oxygen produce 40 ml gas. The correct statement is/are (@) 15 ml CO in the mixture (b) 10 ml C,H, in the mixture (©) 10 ml CO in the mixture (@) 5 ml CO in the mixture 10. 5 gm of a mixture of Ca and Mg reacts with sufficient N, to form compound which on hydrolysis produce 0.1444 mole of NH, gas. Choose the correct statement/statements, (a) Mass of Ca in the mixture is 2 gm (b) Mass of Al in the mixture is 3 gm (©) Mole of Ca in the mixture is 10 gm (@) Mole of Al in the mixture is 0.111. ° a You have either reached 2 page thts unevalale fer vowing or reached your ievina tit for his book. Stoichiometry 29 aim pressure. Calculate the composition of the alloy (Al=27, Mg = 24)? Dry hydrogen was passed over 1.58 gm of red hot copper oxide till it is completely reduced to 1.26 gm copper. If in this process 0.36 gm of H,0 is formed. What will be the equivalent weight of Cu and 0.? Calculate the equivalent Mass of the com- pounds underlined in the following equations. FeS, + KMnO, + H* > Fe + SO, + Mn?* +H,0. (b) No +3 Hy 22. NH, > (D) ON VOLUMETRIC ANALYSIS 16. Calculate the strength in gmlitre of a solutior of H,SO,, 12 cm? of which neutralises 15 em? of decinormal NaOH solution.? 100 gm ofa sample of HCI solution of relative density 1.17 contains 33.4 gm of HCI. What volume of this HCI solution will be required to neutralise exactly 5 litres of decinormal NaOH solution.? ‘One gm-atom of Ca was burnt in excess of oxygen and the oxide was dissolved in water to make a I-litre solution, Calculate normality of alkaline solution. A metal weighing 0.43 gm was dissolved in 50 ml of (N) HSO,. The unreacted H,SO, re- quired 14.2 ml of (N) NaOH for neutralisation, Find out the equivalent weight of the metal.? 4.35 g of a mixture of NaCl and Na,CO, was dissolved in 100 cm’ of HO, 20 ml of which, 15, (a) 17. 18. 19, 20. N ‘was neutralised by 75.5 ml of +> solution of H,SO,, Calculate the percentage of NaCl and Na,CO, in the mixture.? ‘You are given a 2.198 gm sample containing a mixture of XO and X,0,, It requires 0.015 mole K,Cr,0, to oxidise the sample com- pletely to form XOj and Cr**. If 0.0187 mole a. of XO; is formed what is the atomic mass of x? 22, 3.68 gm of a mixture of CaCO, and Na,CO, on reaction with 1000 ml of x HCI solution m4. 25. produces 1.76 gm of CO,. Calculate the % of each components in the mixture.? 100 ml of solution of 0.1 N HCI was titrated with 0.2 N NaOH solution. The titration was discontinued after adding 30 ml of NaOH so- lution. The remaining titration was completed by adding 0.25 N KOH solution. Calculate the volume of KOH required.? 100 ml of 0.6 N. H,SO, and 200 ml of 0.3 N HCI were mixed together. What will be the normality of the resulting solution? Calculate the normality of a mixture obtained by mixture 100 ml of 0.2 M H,SO, with 100 ml of 0.2 M NaOH.? | > (E) ON WATER OF CRYSTALLISATION AND DOUBLE INDICATORS 26. 27. 28. 29. 30. 500 ml of 2 M HCI, 100 ml of 2 M H,SO, and one-gm-equivalent of a monoacidic alkali are mixed together, 30 ml of this solution required 20 ml of 143 gm Na,CO,:xH,0 in one litre solution. Calculate water of Crystallisation of Na;CO,xH,0? Ina mixture of NaHCO, and Na,CO,, volume of a given HCI required is x ml with phenol- phthalein indicator and y ml is required with ‘methyl orange indicator. Calculate volume of HCI for complete neutralisation of NaHCO, in the mixture.? A solution contains Na,CO, and NaOH. 25 ml of this mixture require 19.5 ml of 0.005 N HCI for the end point, when phenolphthalein is used as indicator if methyl orange is used as indi- cator, then 25 ml of solution mixture requires 25.9 ml of the same HCI for end point. Calcu- late the strength of Na,CO, is the mixture.? Amixture of Na;CO, and NaHCO, (8 gm) dis- solved in I litre water, 25 ml of this solution require 32 ml of decinormal HCI using methyl orange as an indicator and 12 ml decinormal HCI when phenolphthalein is used as an indi- cator. Calculate the strength of NaHCO, and 1Na,CO, in the mixture.? 50 ml of @ mixture solution containing NaOH and Na,CO, is taken. 5 ml of this solution 30 Numerical Chemisty when titrated with x HCI require 12 ml for neutralisation when phenolphthalein is taken. A further drop of methyl orange is added in the same solution and titrated with the same HCl requires 4 ml. Calculate the % strength of NaOH and Na,CO, in the mixture solution.? HINTS AND SOLUTION: 1T-WINDOW IV . No. of mole of N, Wt of metal 135 Fog XB = x B= 203 Fee Wi of oxygen 0.53 <8" 2038 Atomic Mass = 2 x 20.38 = 40.75 Ans. Ans. 24 0.165 = *35.5 =7.0 “0.835 Molecular Mass = 85 x 2 = 170 According to question MClx = 170 7Txx+35.5 x= 170 = 425x=170=5 x=4 Valency = 4 Atomic Mass of Metal = 7 x 4= 28 Mass of Cu in Cu,S ‘Mass of Ag that reacts with same mass of Sas in Cu __ Atomie Mass of Cu Atomic Mass of Ag TOSS _ 35 x= 107.768 13553 x Be ANS: . mole of 4,8, = = = 106 23° 015 1 mole of 4B, = 2% = 62 mole of AB, = 575 We have 2x+3y=106=9 x= 26 and x +2y = 62 No. of mole of gas is also 3. 8. % 10. uw. 12. 13. 96 Henee its molecular mass = =~ In Fe,O,, mole of iron = 3. 1.5 mole Fe = 116 gm Fe,0, 3 mole Fe = 232 gm Fe,O, Mol. Mass of Fe,0,, = 232 gm. 0.497 gm Ag=0.701 gm of it 071 0.497 Hence Mol. Mass of Acid = 304.65 — 216 + 2(dibasic) salt 216 gmAg= x 216 = 304.65 gm = 90.65 Ans. Mote of pt = 2222 oe 195 = 2262 mole t= 0.80 gm base 195 . 1 mote pt 2802195 mole Pr= 77 gm base = 595.4 Ans. By using PV = nRT (m=88)(#4) HI reat 760 )\i000) ~ = M= 86.113 Ans, 32 Mass ratio is equal to equivalent ratio if va- lency is the same, Mes _ Eee Mz Exe => x= 20.06 Ans, Let Mass of Al =.x gm Mass of Mg = 1 gm 3 AL + 3HCl > AIC + SH, I mole 1.5 mole 27 gm Al = 1.5 x 22.4 litre of H, at STP 15x224xx x gm Al= 3 litre of H, Similarly Mg +2 HCI MgCl, + H. a You have either reached 2 page thts unevalale fer vowing or reached your ievina tit for his book. a You have either reached 2 page thts unevalale fer vowing or reached your ievina tit for his book. Stoichiometry 33 together and made up to 2 litre, 30 ml of this acid mixture exactly neutralises 42.9 ml of Na,CO, solution containing 1 gm Na;CO, 10 H,0 in 100 ml of H,O. Calculate the amount of sulphate ion in gm present in solution.” HIT, 1985] . The reaction 2C +0, 2 CO is carried out by taking 24 g of carbon and 96 g O, find out: (a) Which reactant is left in excess? (b) How much of itis left? (c) How many mole of CO are formed? () How many ‘g’ of other reactant should be taken so that nothing is left at the end of the reaction.? [Roorkee, 1986] }. What is the strength in gm per litre of a solu- tion of H,SO,, 12 ml of which neutralised 15 mlof x NaOH solution? n-butane is produced by the monobromination of ethane followed by wurtz reaction. Caleu- late the volume of ethane at NTP to produce 55 gm n-butane if the bromination takes place with 90 % yield and the wurtz reaction with 85% yield.? IHIT, 1989] tur, 1987) . A solid mixture 5 gm consists of lead nitrate and sodium nitrate was heated below 600° CC until weight of residuce was constant. If the loss in weight is 28 %. Find the amount ‘of Pb (NO,), and NaNO, in the mixture.? (HIT, 1990] - Calculate Molality of 1 litre solution of 93 % H,SO, by volume. The density of solution is 1.84. gmt? IIIT, 1990] - Upon mixing 45 ml of 0.25 M lead nitrate solution with 25 ml 0.10 M chromic sulphate precipitate of lead sulphate takes place. How ‘many moles of lead sulphate are formed? Also calculate the molar concentration of species left behind in final solution. Assume that lead sulphate is completely insoluble.? (IIT, 1993] 8.0575 x 10° kg of Glauber’s salt is dissolved in water to obtained. 1 dm’ of a solution of density 1077.2 kg m, Calculate molarity, molality and mole fraction of Na,SO, in solu- tion.? [IT, 1994) 9. Amixture of ethane (C,H,) and ethene (C;H,) occupies 40 litre at 1.00 atm and at 400 K. ‘The mixture reacts completely with 130 gm of , to produce CO, and H,O. Assuming ideal behavior. Calculate the mole fraction of C3H, and C,H, in the mixture.? IIIT, 1995] 10. 1.20 gm sample of Na,CO; and K,CO; was dissolved in water to form 100 ml of solution. 20 ml of this solution required 40 ml of 0.1 N HCI for complete neutralisation. Calculate the weight of Na,CO, in mixture. If another 20 ml of thin solution is treated with excess of BaCl. What will be the weight of precipitate? [Roorkee, 1997] 11, For the reaction Nj0,(g)=2 NOg)+ 0.5 O.(g) Calculate the mole fraction of N,O,(g) decom- posed at constant volume and temperature., if the initial pressure is 600 mm Hg and pressure at any time is 960 mm of Hg, (Assume ideal gas behaviour)? [1IT, 1998} 12. A plant virus is found to consists of uniform cylindrical particles of 150 A in diameter and 5000 A long. The specific volume of the virus is 0.75 cm*/g. If the virus is considered to be single particle. Find its molecular weight.? IIT, 1999] 13, How many millilitre of 0.5 M H,SO, are needed to dissolve 0.5 g of copper I carbon- ate? [HIT, 1999] 14, A mixture in which the mole H, and O, is 2 1 is used to prepare water by the reaction. 2 H,(g) + 0x2) > 2 H,0(2) The total pressure in this container is 0.8 atm at 20° C before the reaction. Determine the fi- nal pressure at 120° C after reaction assuming 80% yield of H,0.” [Roorkee, 1999] 15. The formula weight of an acid is 82.0. 100 cm? of a solution of this acid containing 39.0 g of the acid per litre were completely neutralised by 95.0 cm’ of ag. NaOH containing 40 g of NaOH per litre, What is the basicity of the acid? [Roorkee, 2000] 16. 1 gm charcoal is placed in 100 ml of 0.5 M CH, COOH to form an adsorbed monolayer of acetic acid molecules and thereby the molarity of CH,COOH reduces to 0.49. Calculate a You have either reached 2 page thts unevalale fer vowing or reached your ievina tit for his book. a You have either reached 2 page thts unevalale fer vowing or reached your ievina tit for his book. a You have either reached 2 page thts unevalale fer vowing or reached your ievina tit for his book. Stoichiometry 37 17. 18. 19. Density of water = 1000 kg/m* (000 x10? 1000 Molarity = 100076101000 _ 18100010 6 CaO + P,O,p=2 Ca, (PO), P,O,)= 31x44 16x10 Since 284 gm P,O,) = 6 x 56 gm CaO 656x852 284 = 1008 gm CaO According to question Py, = 10" atm, T= 298 K, Vy, =2.46 em? 10° X2.46 x 10 = ny, % 0.082 x 298, = ny, = 1.0% 107 ° No. of molecules of N, in the sites = 6.02 x 10" x 10-7 = 6.02 x 10'° Total surface area = density x area = 6.02 x 10" 10° cm? 6.02 x 10"7 Since only 20 % of this site is occupied by Ny Total area occupied by 0.2 x 6.02 x 10" 5.5 Ans. 852 gm P,O\y = gm CaO No. of surface site occupied per molecule 12.04 x10'° 2 6.02x10"" I IIT WINDOW-VI l > [ERI Match the Ser A with Set B SetA SetB (@) Expo, is (p) Mol, Mass/1 (0) Eu.s0, 18 (q) Mol.Mass/I1 (©) Exes, in the reaction (FeS, + 0, > Fe,0, + 80,) is (@) Exo in the reaction (5) 3 x Mol (0) Mol. Mass/2 (3Mn0;=2MnO,+ — Mass/13 MnO; ) is (©) Exyo3 im the reaction (1) 3 x Mol. (3 MnO, = 2 Mn* Mass/7 + Mn0,) is a Set Set B (a) 20 ml (N) HCI reacts. (p) No. of N molecules of with S0ml = NaOH. HCI eh 0 N (q) No. of (6) 10 ml 5 HCI reacts molecules of N HCl left with 50 ml 75 NaOH. 6 99 492! N () No. of mol- (6) $0 ml 55 HCIreacts ecules of HCL . N left= 2.71 x with 100ml <> NaOH. 922 N (8) No. of mol- (@) 100ml > HCIreaets ecules of HCI N left = 1.8 x with 50ml {5 NaOH. jot a, Seta Set B (a) 4 gm Ca & 2. gm Hy reacts together to form Cal. (b) 2. gm Mgand 2 gm H, reacts together to form Mg H,, (©) 5 gm Na and 6 gm H, reacts to form Nal, () 2.7 gm Aland | gm H, reacts to from AlHy, (p) 5.783 gm H, left unreacted (q) 1.8 gm Hy left unreaeti (®) 0.7 gm H, Left unreacted, (5) 1.83 gm H, left unreacted, 4 Seta SerB (a) No. of molellitre of — (p) Normality solution is (b)No. of mole/kg of (q) Formality solution is () No. of formula unit’ (r) Molarity litre of solution is, (d) No. of equivalent) (s) Motality litre of solution is 38 Numerical Chemisty 5 Set A Set B (a) Minimum molecular mass of Heamoglobin if it contain 0.056 % iron is [Fe = 56] (b) Minimum molecular mass of chlorophyl if it contain 0,025 % Mg is (Mg =24) (©) Minimum mol. mass of anickel dimethyl glyox- ime if it contain 0.025 ‘Gis [Ni= 58] (@) Minimum mol. Mass of chromianotrin if it contain 0.025 % Cr is [Cr=52] 6 Seta (p) 96000 (q) 2,08,000 (8) 1,00,000 (8) 2,32,000 Set B (a) Titration of NH,OH (p) Phenolphthalein with HCI solution (b) Titration of K,Cr,0, with Na,S,0, solution (©) Titration of H,SO, with NaOH solution (4) Titration of K,Cr,0, with 1, solution in acid ‘medium. (q) Methyl orange (1) No need of indicator (8) Alizarin bem Matrix-Matching Type Matrix — 1 (a) | mole SO, at STP (p) 24.7 litre (b) I-mole O, gas at (q) 22.4 litre SATP. (©) 32 gm O, gas. (d) 64 gm SO, gas (8) 1.204 x 10 atoms (5) 1.809 x 10 atoms Matrix - 2 Molar Mass (@) Eu,n0,= J (p) 42 (©) Excoon = MOMEM®SS—(q) One H-atom \ directly attached with central atom, _ Molar Mass 1 (1) 46 (€) Exaygo, = MOEMESS (5) H-attached with 5 central atom have shorter bond length, () 31.6 Matrix - 3 (a) VN, (p) Applicable for acid & base (b) VM, (q) Application for base. (6) VN =ViN, + VNy (©) Law of equivalence +N (4) Milli equivalent of —_ (s) Applicable for dilu- the two reacting tion, substance must be the same. Matrix — 4 (a) Alkaline solution (p)CO (b) Alkaline Pyragallol (q) CO, (c) Ammonical Cu,Cl,(r) C,H, (d) Charcoal (s) SO, (0, Matrix -5 (a) ‘SV"H,O, sample (p) H,O, has open book structure (b) 109 % oleum (@) 40% SO, a You have either reached 2 page thts unevalale fer vowing or reached your ievina tit for his book. 40 Numerical Chemisry 3, 8 gm methane is burnt with 4.48 litre of O, at STP. Find out the volume of CO, gas produced at STP and also the weight of CO, formed. (a) 22.4 litre, 4.4 gm_ (b) 2.24 litre, 44 gm (c) 2.24 litre, 4.4 gm (d) 224 litre, 44 gm [D] All such titration which involves the direct titra- tion of iodine with a reducing agent are grouped under iodimetry. lodimetry is employed to de- termine the strength of reducing agent such as sodium thiosulphate 1, +$,0? 31° +8,02 If liberated Iodine involves oxidation of an iodide ion by a strong oxidising agent in neutral or acidic medium the liberated iodine is then litrated with reducing agent. lodometry is used to estimate the strength of oxidising angent e.q Cu + F > Cu,F, +F, 1, + 8,0} > 8,0; +1 Starch used as indicator near the end point which form blue colour complex with f;. The blue colour disappears when there is no more or free I) > 1. 100 ml of 0.1 N Na,$,0, decoloutises iodine by the addition of “X" gm of crystalline blue vitriol to excess of KI. The volue of is (a) Sem (b) 2.5 gm (©) 10.gm (@) 1.25 gm 2. In the reaction 2 CuSO, + 4 KI > Cu,l, + K,SO, + 1,, the ratio of equivalent weight of CuSO, to its molar weight is: (a) 18 (by 14 (©) 12 @1 3. In iodine titration, Iodine remains in solution in the form of @ 1 ©) 1 Oh @I [E] Methyl orange and phenolphthalein are general- ly used as an indicators in acid range base titra- tions. Methyl orange acts in acid range having P" range 3.1 — 7.0 while phenolphthalein acts in the P* range of 7.1 to 10. Phenolphthalein changes its colour as soon as CO, and SO, starts, forming in a reaction, although reaction is not, complete; Methyl orange changes its colour af- ter completion of the overall neutralisation, > 1, 25 ml of Na,CO, solution requires 100 ml of 0.1 N HCI to reach end point with phenol- phthalein as indicator, Molarity of resulting solution with respect to HCO, ion is: (a) 0.008 M (b) 0.004 M (©) 0.16M (d) 0.08 M 2. 0.01 Mole of H;PO, and 0.01 mole of HPO, reacts with ‘17 ml of standard NaOH in pres- ence of phenolphthalein indicator, volume ‘Vis: (a) 500 ml (b) 400 mL (©) 300 ml (4) 200 mi 3. 20 ml of “X" M HCI neutralises completely, 10 ml of 0.1 M NaHCO, solution and a further 5 ml of 0.2 M Na,CO, solution to methyl or- ange end point. The value of X is (a) 0.167 M (b) 0.133 M (©) 0.15M (@) 02M I ANSWER KEYS: IIT WINDOW-II (A) l LL) 2@ 3@ 40 5) 6@ 74) 8@ 9) 10.) 1.@ 12.) 13.6) 14.@ 15.) 16.(b) 17.(@) 18.(@)_—19.(6) 20. (b) I ANSWER KEYS: IIT WINDOW-II (B) ] 1.@ 2a) 3.4) 4.0) 5.(a) 6(c) 7.(b) 8b) 9. (@)_—-10.() 11.(b) 12.@) 13.4) 14.) 15. (b) I ANSWER KEYS: IIT WINDOW-II (C) ] Lb) 2.6) 3.@) 4.00) 5.@) 6(c) 7.(b) 8). @)_—10.() 1.0) 12.0) 13.@) 14. @)—15.(@) Stoichiometry 41 [answer keys: irr winow-(o) J 2 12,20. 06 13. Al; 0. 55 gm 14.8 L@ 26 30 4@ se@ Mg: 0.45 gm 6d) 7.(b) 8a) (a) 10. (@)_— 1S. (@) M/I1.(6) i316, 6. 125 IL (c) 12a) 13.) 14 (©) 1S.(@) 17. 46. 72 mp 18.2 16.(c) 17-(b)18.(@)—19.(a)-20.(@) 19, 12.01 20. 45. 99% Na,CO, I ANSWER KEYS: IIT WINDOW-II (E) l 21. 100. 04 22. 23.16 ml 24, 1@ 20) 3. 4@ 5 95 70 36.10 6.0) 7.06) 8) 9.06) 10.4) 97 yy 28,0,1356 1L@ 12.@) 13.0) 29. 5.8 gm/l, 2.69 gm1 30. 11. 68 (NaOH) I ANSWER KEYS: IIT WINDOW-II (F) l I ANSWER KEYS: IIT WINDOW-V l 1.) 2.(b)3.(@)4.(b) 5.0) 6.(b) 7.(b) 8) (a) 10-6) 1.6, 528 em 2.6. 125 gmvlitr 4,35. 55 lite 5.3.32 & 1.68. gm I ANSWER KEYS: IIT WINDOW-III l 6.10.42 8.4.3x107 1. (b), (c), (d) 2. (a), (b), (c) 9.0, 66,0, 34 10. 0.394 3. (b), (¢) 4. (a), (d) 11.0.4 12. >. 095 x 107 5. (a), (b), (¢) 6. (a), (b), (c), (d) 13. 8.09 ml 14. 0. 787 atm 7. (a), (b). (6) (@) 8.) () 15.2 16.5x 10m? 9..(b), (©) 10. (a), (b) (@) 17.55.5 18, 1008 IL. (a), (d) 12. (a), (¢), (d) 19.2 13.@.(0) 14. (@), (6) 15. (a), (b), (¢) I ANSWER KEYS: IIT WINDOW-VI ] I ANSWER KEYS: IIT WINDOW-IV. l a-r,b-pye-q.d-Te-s 1 2. a-qb-p,e-s,d-r 1.40.75, 2.24 3. .c-p.d=r 3.28 4.107. 76 4 c-qd-p 5.26, 18 6.32 5. a—r,b-p,c-s,d—q 7.232 gm 8, 90. 65 6, a-gb-re B+ Diameter of the atom _ 10° _ 49s Be’ + He’ > “cts ou Diameter of the nucleus. 10 2. Mass on neutron is found to be 1.675 * 10° 7. Density of the hydrogen nucleus containing kg, only one proton. 3. Charge on neutron is zero. 5 Mass _ [A 1.66% 10g] (Important Concepts Volume 44 x mom? 1. "itis Pah om cootenn mn sect Sli d= 1.685 10" prem? asi path ofan electton ina magnetic field is The Relation between Volume ofthe atom and circular with radius ‘r’ given as =!" sngeleih a Bibione ‘Where ei e Volume of the atom oa pain Volume of the nucleus charge on electron B= Intensity of applied magnetic field 3. Mass of moving electron = rest 9. Number of revolution per second made by an 1 electron in a shell Velocity 2 4. Rutherford concluded that the number of par- Gammon der a ticles (N) scattered at an angle @ is such that Y Where £, = Energy of first shell va 10, Maximum number of lines produced when an sin* ) electron jumps from nth level to ground level 5, The graph between angle of scattering and the (n=l) number of orparticles scattered in correspond- 2 ing directions is shown as 11. When an electron returns from n, to m, state, the number of lines in the spectrum 2 (=m)(ng~n +1) 5 2 i 12, Line of longest wavelength or line of shortest 3 energy in any series of hydrogen spectrum = a = s = (n+) 2 13, Line of shortest wavelength or line of high- est energy (series limit) in any seri hydrogen spectrum 46 Numerical Chemisty 14. 15. 16. 11. 18. 20. Fae oh mre Excitation potential for 1 — ny En En Electronic charge lonisation potential m, —+ 20 Separation Energy: (fan electron is already present in the excited state, then the energy re- quired to remove that electron is called sepa- ration energy. It is shown as Ereparation B..— Forces ah Aofan electron = 4 ? h V2eVm Ap Ax= AE At a AE Ar> 4 (for energy and time) i Ao Ao> 4 (for energy and motion) it ‘The number of waves made by a Bohr elec- 2ar tron in one complete revolutior _2ar ar For hydrogen wave function, number of nodes can be calculated as: (@) Number of radial nodes = (n= 1 1), (ii) number of angular nodes = / (iii) Total number of nodes = (r ~ 1) (iv) Number of nodes planes 2. 2. 23. 25. 26. 27. 28. Orbital angular momentum of an electron is calculated using the following expression aD = Vigan nek la The region of maximum electron density i called antinode There are seven f— orbitals designated as Soe SoS, Frfar-sry Soler-srp Sa ie) pin angular momentum = 2x Where s= 4 1 Total spin of an atom = n x + Where m= no. of unpaired electrons [s(s+1 Spin magnetic moment= /5(6+0) = ch mc Planck's Quantum Theory: Energy is emit ted or absorbed only in discrete units or pack- ets of energy called photon (quantum). The energy associated with a quantum is given by he E=hv where f = 6.625 x 10 J. see = x 1077 erg — sec. ©=3 x 108 m/sec Bohr’s Model Energy released or absorbed (AE) in given by AE It ifn, > n, emission spectrum 1n, > ny absorption spectrum Angular momentum is an integral multiple of hn _ nh eemur= 2 2 where n= 1,23 son 9, Bohr's Radius Radius of nth orbit (r,) is given by eh 6.625 3x 10" em / see Tan = a You have either reached 2 page thts unevalale fer vowing or reached your ievina tit for his book. a You have either reached 2 page thts unevalale fer vowing or reached your ievina tit for his book. Atomic Structure 49 + The solution of Schrodinger’s equation is a= —+_, [2m(E =v)" i.e. greater in the value of (E - V), shorter is the wavelength of wave function. Cartersion and polor coordinates ., Electron (yz) eos @ rsin 8..cos & Fig 2.4 X=rsin.6. cos Y=rsin 6, sin g Z=rcos } vere + The energy of an orbit is given by mes Benn? I IIT WINDOW-1 l > BO > Example 2.1 Caleulate the force of attraction between an electron and a body having two protonic charge when they are 0.529 x 10°* cm apart. [e==16x10¢,P'=+ 16% 10] > Solution Force of attraction (F) = = K= 9X10? No? CP F= 165x107 N, Ans. > Beampte 22 Calculate the velocity of an electron in the Ist Boht’s orbit of hydrogen.” > Solution For H-atom, Z = 1, n = 1 (ground state) _ dase? _ 23.14 x (48x10 nh 1% 6.67 x 1077 = 2.19 x 10° m/sec Ans. P> Brampie 23 AIR Services on FM is transmitted 97.2 m band My What is its frequency in Hertz ? > Solution ve cia= 3210" _ 3.086 10° Hertz 97.2 > Beampte 24 The dissociation energy of H, is 430.53 kJ mole". If H, is exposed to radiation energy of 4 = 253.7 nm. What % of radiant energy will be converted into K.E, > Solution _ 430,53 x 10° = 0" joule/motecule 6.02 10" = 7.15 x 10°” Jimolecule _ he _ 6.625 x10 x3 x 108 Enpoton = ME = SO25 103 X10" a 253.7 x10 = 7.83% 10 J. Energy converted to K.E = (7.83 — 7.15) « 10 "joule = 0.68 x 10" J. % of energy converted into K.E. 0.68 x 10°" 7.83 x10 Eun x 100 = 8.68% Ans. 7 Bampte2 I, molecule dissociates into atoms after absorbing light of 4500 A°. If one quantum of radiation is ab- a You have either reached 2 page thts unevalale fer vowing or reached your ievina tit for his book. a You have either reached 2 page thts unevalale fer vowing or reached your ievina tit for his book. a You have either reached 2 page thts unevalale fer vowing or reached your ievina tit for his book. Atomic Structure 53 (©) orbital angular momentum of an orbital is given by = V+) Won ‘Thus for 2p orbital orbital momentum = ViG+1) wan = 2 hin = V2h Ans. > Example 2.20 Calculate the momentum of electron moving with 1/3rd velocity of light. > Solution mass of & in motion = Rest mass Since v= ¢/3 ‘momentum = 9.69 x 10" Ans. > Example 2.21 Anelectron beam can undergo diffraction by erystals. ‘Through what potential should a beam of e be accel- erated so that its wavelength became equal to 1.54A° IIIT, 1997] gems > Solution Aswe Know, (6.62510) 2%9.108%10" x(1.54%107)' x1,602%10-" =63.3 volt. Ans. > Exam, 2 ‘The dust particle having mass equal to 10"' g diam- eter = 10 cm and velocity 10 cm sec”, The error in measurement of velocity is 0.1%. Calculate uncer- tainty in its position. Comment on the result > Solution Aw = 10x = 1x 107 em see 100 os Au de = 4am f= ——_ Sa =5.27%10 "em © F314 x10 xT ‘The uncertainty in its position Ax _ 5.27107" 1o* The factor being small and almost being negligible for microscopic particles, [> Beampre 2.23 ‘What is the maximum precision with which the mo- ‘mentum of the electron ean be known if the uncer- tainty in the position of electron in +0.001 A°. Will there be any problem in decreasing the momentum if it has a value of h/27 90, Where 7, = Bohr’s radius, =5.27x 10% cm. diameter > Solution Av=0.001A = 0.001 x 10°! m= 10-7 m. = £05107 597 10 Ns 4x3.14x10 h 6.625 x10 if momentum = 5— = 8.02510 __ 2mm, 2x3.14x0.529x10" =2x104Ns, uncentainty in momentum is about 527x107? 210° ie 263.5 times as large as the momentum itself is. I IT WINDOW l am Multiple Choice Questions Pe 1. The maximum number of electrons with clockwise spin that can be accommodate in af subshell are (@) 14 (b) 7 (5 (@) 10 a You have either reached 2 page thts unevalale fer vowing or reached your ievina tit for his book. Atomic Structure 55 2 23, 24, 28, 26. 27: 28, 29, 30. (b) 5 7 (@ 9 If ris the radius of first orbit, the radius of nth orbit of the hydrogen atom will be (a) rn? (b) en (©) rin (@ Pr The ratio of radius of 2", 4" and 6" orbit of hydrogen atom is: (a) 2:4:6 (b) 1:49 (©) 1:46 (a) 1:23 The distance between 3" and 2™ orbits in the hydrogen atom is (a) 2.646 x 10° em (b) 2.116 x 10 “em (©) 1.058 x 10 em (d) 0.529 x 10° em E, = -313 6/n* keal/mole. If the value of E = 34.84 kcal/mole, to which value does ‘n’ cor- respond? (a) 4 (b) 3 (©) 2 @1 The ratio of the difference between 1" and 2" Bohr’s orbits energy to that between 2" and 3 orbits energy is: (a) 12 ) 13 (c) 27/5 (@) 527 The ionization energy of the electron in the 1s orbital of the hydrogen atom is — 13.6 eV. The energy of the electron after promotion to 2s otbi (a) -3.4eV (b) ~13.6eV (c) -27.2eV (4) 0.0 eV ‘The speed of the electron in the 1” orbit of the hydrogen atom in the ground state is [e is the velocity of light] se ) © 137 © 1370 eo a) © 13.7 © 137 ‘The wave number of the line of Balmer series ‘of hydrogen is 15200 cm ', The wave number of the first Balmer line of Li ion is: (a) 15,200 em (b) 60,800 em! (c) 76,000 em * (@) 136,800 em! Which one of the following orbitals has the shape of baby-soother type? 3. 32. 33. 35. 36. 37. 39. (a) d, (b) dey (© a: @ p, ‘An ion which has 18 electrons in the outer- most shell is (a) K (b) Cu (c) Cs” (@ Th In Millikan’s oil drop experiment, we make use of: (a) Ohm’s Law (b) Ampere’s Law (©) Stoke's Law (4) Faraday's Law As the speed of the electrons increases, the measured value of charge to mass ratio (in the relativistic units): (a) increases (b) remains unchanged (©) decreases (@) first increases and then decreases The number of waves made by a Bohr elec- tron in an orbit of maximum magnetic quan- tum number +2: (a) 3 (b) 4 (2 @i An electron of mass m and charge e, is accel- erated from rest through a potential difference V in vacuum. Its final speed will be: (a) \eVim) (b) 2eV/m (©) JeV2m (@ ¥@eV7m) The difference in angular momentum associ- ated with the electron in the two successive orbits of hydrogen atom is: (a) hin (b) hia (©) Wid @) (DA Xn (a) are deflected in « magnetic field (b) are deflected in an electric field (c) remain undeflected by both the fields (@) are deflected in both the fields ‘The energy of an electron in the first Bohr or- bit of H atom is ~ 13.6 eV. the possible energy value (8) of the excited state (s) for electrons in Bohr orbits of hydrogen is (are): (a) -3.4eV (b) 4.2eV (b) -6.8eV (@) +6.8eV The energy of the electron in the first orbit of He’ is871.6x 10 "J. The energy of the elec- tron in the first orbit of hydrogen would be: 56 Numerical Chemisty (a) -871x 10°F (b) -435x 107 {c) -217.9x 10" J (d) 108.9 x 107 J 40. The wavelength associated with a golf ball weighing 200 g and moving with a speed of $ mh is of the order of (a) 10m (b) 107m (©) 10° m @ 10° m 41. The Bohr orbit radius for the hydrogen atom (n= 1) is approximately 0.530 4°, The radius for the first excited state (11 = 2) orbits is, (a) 013.4 (b) 10.6.4” (©) 4.774" (a) 2.12.4" 42. The angular momentum (L) ofan electron in a Bohr’s orbital is given as: b= fh (a) in A b) L= pa 0) b= SD b= me Lm @ ie 4a 43. The quantum numbers + 4 and — 1 for the electron spin represents: 2 (a) rotation of the electron in clockwise and anticlockwise direction respectively (b) rotation of the electron in anticlockwise and clockwise direction respectively (€) magnetic moment of electron pointing up and down respectively (@) two quantum mechanical spin states which have no classical analogues 44. How many moles of electrons weigh one kilo- gram? (a) 6.023 x 10° 1 © S08 6.023 192 9.108 1 —!_ x40 9.108 x 6.023 45. If the electronic configuration of nitrogen has 1s’, it would have energy lower than that of the normal ground state configuration 1572s? x10 © 2p* because the electrons would be close to the nucleus. Yet 1s” is not observed because it vio~ lates: (a) Heisenburg uncertinity principle (b) Huné’s rule (c) Pauli’s exclusion principle (d) Bohr postulates of stationary orbits 46. No. of visible lines when an electron returns from 5" orbit to ground state in H spectrum : (a) $ () 4 (©) 3 (@ 10 47. Consider the following statements: (A) Electron density in the X¥ plane in 3dy_ orbital is zero (B) Electron density in the XY plane in 3d. orbital is zero (C) 2s orbital has one nodal surface (D) For2p_orbital ¥Zis the nodal plane, which are the correct statements (a) (A) and (C) (b) (B) and (C) (c) only (B) (d) (A), (B), (C), (D) 48. The first emission line in the H-atom spectrum in the Balmer series appears at: @ 2 om 32 em TR 1 OR 1 IR a SR © Te em @ om 49. Radial probability distribution curve is shown for s orbital in figure below. The curve is: Time —> (a) Is (b) 2s (©) 3s (@) 4s orbital has (a) A lobe along Z-axis and a ring along XY plane (b) A lobe along Z-axis and a lobe along X¥ plane (©) A lobe along Z-axis and a ring along YZ plane (@) A lobe and a ring along Z-axis Atomic Structure 57 51. Which of the following statement is wrong for an electron that has n= 4 and m =—2 (a) The eleciton may be in a d-orbital (b) The electron is in the fourth principal electronic shell (©) The electron may be in a p-orbital (@) The electron must have the spin quantum number = + % Which of the following statement is/are wrong (a) Ifthe value of I= 0, electron distribution is spherical (b) The shape of the orbital is given by mag- netic quantum no. (©) Angular moment of 1s, 2s, 3s electrons are equal (d) In an atom, all electrons travel with the same velocity 53. In Hydrogen atom the energy of an electrons, determined by quantum numbers (a) n (b) and! (c) n, Land m (4) m1, mands 54, ‘The first line in the Balmer series in the hydro- gen spectrum will have the frequency (a) 4.57x10'4s' (by 3.29x 10%! (c) 822x105! — (d) 8.05 x10" s* 55. The maximum value of / for an electron in fifth energy level is (a) 5 (b) 4 {c) 3 (d@) 2 56. The minimum value of n for which g ~ sub- shell is possible is (a) 6 (b) 5 4 (3 Ifthe value of prineipal quantum number is 3, the total possible values for magnetic quan- tum number will be fa) 1 (b) 4 (©) 9 @ 12 58. The uncertainty in the position of an electron (mass 9.1 x 107*g) moving with a velocity of 3.0.x 10% cm s ' accurate up to 0.011% will be (a) 1.92 em (b) 7.68 em (©) 0.175 em (d) 3.84em ‘The momentum of a photon having frequency 1.0.x 10" ms lis about (a) 2.21 11 kg ms" (b) 2.21 x 1077 kg ms! 52, 57, 59, 60. 61. 62. 63. 65. 66. (c) 2.21 x 10° kg ms (d) 6.63 x 10° kg ms The work function of a metal is 4.0 eV, If the metal is irradiated with radiation of wave- length 200 nm, then the maximum kinetic en- ergy of the photo electrons would be about (a) 64x 10°F (b) 35x10) () 10x 10°F (d) 20x10" 5 The maximum kinetic energy of the photo- clectrons is found to be 6.6 x 10” J when the metal is irradiated with a radiation of fre- quency 3 x 10'* Hz. The threshold frequency of the metal is, (a) 1x10" Hz (b) 3x 105 Hz (c) 2x 10'S Hz (@) 2x10 Hz If threshold wavelength (4,) for ejection of electron from metal is 300 nm, then work function for the photo electric emission is (a) 6x 10°F (b) 1.21085 (©) 3x10 PF @ 6x10? 5 Radial part of the wave function depends on quantum numbers, (a) mands (b) Land m (©) Lands (@) mand! If E is the kinetic energy of the particle then which of the following expressions is correct for the de-Broglie wavelength of the particles? A A fa) A= by A |—_ eae « Imi h Jim 5 tie é at - an At 200°C hydrogen molecules have velocity 10° cm sec”'. The de-Broglie wave length in this case is approximately (a) 2A (b) 1000 A (c) 100 A (d) 10 A The electrons idemtified by quantum numbers nand [(i)n=4, [= 1 (ii) n=4, 1= 0 (ii) n= 3,1 =2 (iv) n=3,/= 1 can be placed in order of increasing energy, as (a) (iv) < Gil) < Gill) < (b) (ii) < (iv) < (i) < Git) (c) (i) < Gili) < Git) < (iv) (@) Gi) < @ < (iv) < Gi) a You have either reached 2 page thts unevalale fer vowing or reached your ievina tit for his book. a You have either reached 2 page thts unevalale fer vowing or reached your ievina tit for his book. a You have either reached 2 page thts unevalale fer vowing or reached your ievina tit for his book. a You have either reached 2 page thts unevalale fer vowing or reached your ievina tit for his book. a You have either reached 2 page thts unevalale fer vowing or reached your ievina tit for his book. a You have either reached 2 page thts unevalale fer vowing or reached your ievina tit for his book. a You have either reached 2 page thts unevalale fer vowing or reached your ievina tit for his book. a You have either reached 2 page thts unevalale fer vowing or reached your ievina tit for his book. a You have either reached 2 page thts unevalale fer vowing or reached your ievina tit for his book. a You have either reached 2 page thts unevalale fer vowing or reached your ievina tit for his book. a You have either reached 2 page thts unevalale fer vowing or reached your ievina tit for his book. a You have either reached 2 page thts unevalale fer vowing or reached your ievina tit for his book. a You have either reached 2 page thts unevalale fer vowing or reached your ievina tit for his book. a You have either reached 2 page thts unevalale fer vowing or reached your ievina tit for his book. a You have either reached 2 page thts unevalale fer vowing or reached your ievina tit for his book. a You have either reached 2 page thts unevalale fer vowing or reached your ievina tit for his book. a You have either reached 2 page thts unevalale fer vowing or reached your ievina tit for his book. a You have either reached 2 page thts unevalale fer vowing or reached your ievina tit for his book. a You have either reached 2 page thts unevalale fer vowing or reached your ievina tit for his book. Radioactivity 77 > Solution At radioactive equilibrium A —> B Na he (ads Ne da (ads 31x10? _ 2x10" 1 (hide (hyady = 6.45 year Nuclear Transformation or Nuclear Transmutation The conversion of one clement into another by atti- ficial means, i.e. by means of bombarding with some fundamental particles, is known as artificial trans- mutation. The phenomenon was first applied by Ru- therford on nitrogen whose nucleus was bombarded with cparticles to produce oxygen. HY + Het OT HT Nitrogen eePanticle Oxygen Proton ‘nolope isotope In case, the element so produced shows radioactivity, the phenomenon is known as induced radioactivity. Following important fundamental particles have been used in the bombardment of different elements. (i) ocParticle: Helium nucleus represented as (Hey (ii) Proton: Hydrogen nucleus represented as |H' (ii) Deutron: Deuterium nucleus represented as iH? or ,D? (iv) Neutron: A particle of mass number 1 but no charge represented as gn! + Since c-particles, protons and deutrons carry positive charge, they are repelled by the posi- tively charged nucleus and hence are not good projectiles, Neutrons, which carry no charge at all, are the best particles. + Further among o-particles, protons and deu- terons; the latter two carrying single positive charge are better projectiles than the o-parti- cles, + However, the positively charged a-particles, protons and deuterons can be made much more effective if they are imparted with high velocity. + Cyclotron is the most commonly used instru- ‘ment for accelerating these particles, the par- ticles leave the instrument with a velocity of about 25,000 miles per second, + A more recent accelerating instrument, called. the synchotron or bevatron makes use of the induced current or force exerted on the elec- tron in a changing magnetic field. + Neutrons, being neutral, cannot be accelerated by particle accelerators. Parallel Path Decay A radioactive element A decay to B and C in two parallel paths as: Say emission of a Say emission of B | (, The average decay constant for the element A can be expressed as Raverge = Pepa + AB pth Aa pair ™ [Fractional yield of B] x ,, Ay paty= [Fractional yield of C] x Ay, Maximum yield of Daughter Element A radioactive element A decays to give a daughter clement B which further decays to another daughter element C and so on till a stable element is formed (AB ©). Also if number of daughter atoms at ¢ is zero and parent atom is much more lived than daughter (i.e. Ay < Ay) Where, 2, and 4, are decay constant of A and B respectively, then number of at- oms of daughter element B after time sis Moka [pons ght yak ee] Maximum activity of daughter element can be ex- pressed at yu de logy, | => me [i] ln a You have either reached 2 page thts unevalale fer vowing or reached your ievina tit for his book. a You have either reached 2 page thts unevalale fer vowing or reached your ievina tit for his book. a You have either reached 2 page thts unevalale fer vowing or reached your ievina tit for his book. Radioactivity 81 involved in many reactions of industrial importance such as alkylation, polymerisation, catalytic synthe- sis ete, (iv) Analytical studies Several analytical proce- dures can be used employing radioisotopes as tracers. (a) Adsorption and occlusion studies A small amount of radioactive isotope is mixed with the inac- tive substance and the activity is studied before and after adsorption. Fall in activity gives the amount of substance adsorbed. (b) Solubility of sparingly soluble salts The solu- bility of lead sulphate in Water may be estimated by mixing a known amount of radioactive lead with or- dinary lead. This is dissolved in nitric acid and pre- cipitated as lead sulphate by adding sulphuric acid. Insoluble lead sulphate is filtered and the activity of the water is measured. From this, the amount of PbSO, still present in water can be estimated. (0) Ion-exchange technique Ion exchange process of separation is readily followed by measuring activ- ity of successive fractions eluted from the column. (d) Reaction mechanism By labelling oxygen of the water, mechanism of ester hydrolysis has been studied yo. yo RoC + HOI» R-c® | +R! ON ‘or’ ‘on (c) Study ofefficiency ofanalytical separations The efficiency of analytical procedures may be measured by adding a known amount of radioisotope to the sample before analysis begins. After the completion, the activity is again determined. The comparison of activity tells about the efficiency of separation. I IT WINDOW-1I ] > Te CEU eae 1. Acertain nuclide has a half life of 30 minutes. If one starts with 48 g of it, amount left at the end of 30 minutes will be (a) 24g (b) 16g (© 8g @) 6g 2. ‘The sample of radioactive isotope with a half life of 20 days weighs 1.0 g. After 40 days, the weight of the remaining element is (@) 05 (b) Zero (©) 0.258 (@) 16g 3. 1f5.0 g of a radioactive substance has f, hours, 20 g of the same sample will have a f,5 of? (a) 56 hr (b) 3.5 br () 14 br (@) 28 hr 4. The half life of a radioactive substance is 100 days, After 400 days, one gm of element will be reduced to (a) 05g (b) 0.25 g (©) 0.0625 g (d) 0.125 g 8.0 g of a radioactive substance is reduced to 0.5 g. after one hour. The half life period of the radioactive substance is? (a) 15 min (b) 30 min (©) 45 min (@) 10min 6. Mass number of a nuclide is 216. Its approxi- ‘mate radius in Fermi units is (@) 6.0 (b) 7.0 (c) 8.0 (d) 7.8 7. A sample of radioactive element with t,, of 11.2 years weighs 2.0 g. After 33.6 years how much amount of that element is left? (a) 0.258 (b) 05g (0) 0.75 g @ 10g 8. The radioactivity due to C-14 isotope (half life = 6000 years) ofa sample of wood from an ancient tomb was found to be nearly half that of fresh wood. The tomb is there for about? (a) 3000 yr old (b) 6000 yr old (©) 9000 yr old (@) 12000 yr old 9. A substance is kept for 2 hours an three fourth disintegrates during this period. The half life of the substance? (a) 2 hours (b) 1 hours (©) 30 minutes (@) 4 hours 10. After three half lives, the percentage of frac- tion of amount left is? (a) 635 (b) 12.5 (c) 50 (d) 75 11, The reactor moderated by heavy water is called a You have either reached 2 page thts unevalale fer vowing or reached your ievina tit for his book. a You have either reached 2 page thts unevalale fer vowing or reached your ievina tit for his book. a You have either reached 2 page thts unevalale fer vowing or reached your ievina tit for his book. Radioactivity 85 15. 16. 17. 18. 19. 20, 21. 22, 23. m4, Fertile nuclides are (a) Isotopes (b) Fissionable (©) Not fissionable (4) None of these If 1/16" of a substance is left after 40 days then the half-life is ICPMT, 2000 [CPMT, 2000) (a) 40 days (b) 30 days (©) 20 days (@) 10 days The nuclei which can undergo fission reae- tions are called [PMT (DELHI), 2000] (a) Fertile nuclide (b) Infertile nuclides (©) Fissile nuclides (4) Infissile nuclides Initially 300 g of a substance has half-life of 30 minutes. The amount left after 3 hr is CBSE, 2000) (@ 3.698 (b) 4.698 (©) 6.66 ¢ (4) 5.69¢ ‘The carbon dating is based on IPMT (MP), 2001]] @ cl ) cl (©) Cc @ Cc" If the binding energy of an atom is 128 MeV and binding energy per nucleon is 8 MeV. Then the number of nucleons in the atom (nu- cleus) is ICMPT, 2001] (a) 8 (b) 16 (©) 18 (6) 28 ‘The half life of ,C"*, if decay constant is 6.31 x 104is ICBSE, 2001) (a) 1098 years (b) 109.8 years (©) 10.98 years (@) 1.098 years The first artificial disintegration of an atomic nucleus was achieved by [Kerala (E), 2002] (a) Geiger (b) Wilson (©) Madam Curie (@) Rutherford To differentiate between C-12, C-13 and C-14, the instrument that you would use is [Kerala (E), 2002} (a) Infrared spectrometer (b) Atomic absorption spectrometer (©) Mass spectrometer (@) Ultraviolet spectrometer Which one of the following statements is cor- rect IEAMCET (E), 2002] (a) Theradius (R) ofa nuclide of mass number Ais given by R=R,A'” (Ry is constant) (b) jN'° and ,O"* are isobars (©) The end nuclide formed in thorium series (An) is .;Bi2” (d) Ca“ has magic number of protons and ‘magic number of neutrons 25. In a radioisotope, the value of f,, and A are identical in magnitude. The value of 2 is [Karnataka (M), 2002] fa) (0,693) (b) 0.693 (c) (0.693)'7 (d) 1/0693 26. The elements ,K" and ,Ca" are {CMPT, 2002] (a) Isobaric (b) Isotopic (0) Isotonie (@) Isoelectronic 27. According to the nuclear reaction Be + JHe* — ,C'? + gn! the mass number M of Be is IAFMC, 2002] (a) 4 (b) 6 (c) 7 @9o 28. Half life period of ,,1'* is 60 days. Percentage of radioactivity present after 180 days is ICBSE (AIEEE), 2002] (a) 50% (b) 75% (©) 36% (@) 12.5% 29, If ,U** nucleus absorbs a neutron and disin- tegrates in ,,Xe""*, ,,Sr°° and X. Then what will be the product X. ICBSE, 2002] (a) o-particle (b) B-particle (c) 2-neutrons (@) 3-neutrons I 1IT WINDOW-IV l > SMI 1, Complete the following reactions: (th ot 7 Sle +68" (HIT, 2004] GBS + on’! —s + Yle + en IHIT, 2003] Gil) Ws~e — Z e+ {LIT, 2005] 2. “Cu (Ty = 12.8 year) decays 6° emission (38%), B° decay products and calculate partial half-lives for each of the decay processes. [HIT, 2002] a You have either reached 2 page thts unevalale fer vowing or reached your ievina tit for his book. a You have either reached 2 page thts unevalale fer vowing or reached your ievina tit for his book. a You have either reached 2 page thts unevalale fer vowing or reached your ievina tit for his book. Radioactivity 89 10. 1, log —4 6.909 0.693 _ 9 9349 a-143430 2.303 66.6 or log = 0.0312 a-2,3814 x10 Taking antilogarithm ——4 __.. 54 7 4~2.3814 x10" a= 1.07 (a~2.3814 x 10) 0.07a = 2.518 x 10" mole a= 3.597 10 “mole At. wt. of Mo = 99 Amount of Mo = 3.597 x 10* x99 =3.56x 10g aot —> Pb change in Mass No 4 No. of aparticles 28 4 No. of feparticle = (Atomic No. of Product) ~ (Atomic No. of reactant — Z x No of a) =82-90+2x7= ty of carbon = $770 years. 11540 3770 | $770 1 12 4 It means "4 remain, Wo. = 7mg= 7x 10" gm. 7x10" 14 .05 x 10.* x 6.02 x 10°* 01x10” =05x107 Mole of carbon ~ 1 Atoms of C" om m8 +. No. of ‘nin C= 3.01 x 10° x8 40x 10"! Total Mass of Neutron _ 01x10 x8 x 1.00 602x108 =4x 107 gm. I 1T WINDOW-V l [> Waten the column with column 1 Matrix Matching 4 Columat A, Nuclear Fission B. Nuclear Fusion C. Bleeder Reactor D, Atomic Reactor Column I A. 4n Series B. (4n+ 1) Series C. n+ 2) Series D. (4n+3) Series Column t A. Halt ite B. Average life ©. Complete life D. Decay life Column t A. Isosters B. Isotopes C. Isobars D, Isotones Column I A. Isoelectronie B. Isodiaphers C.Isotones D. Isosters| Column 11 pu QU or Pu! R. Steller Energy S. Atom bomb, Column I P. Thorium series Q Actinium Series R. Uranium Series 8, Naptunium Series Column Tt P.Dependentofinitialcone. Q. Infinity R. 0.693/av. Life. 8. 1.044 X ty Column I P. HP and ,He? Q.€O, and N,0 R. ,H*and H! 8. .yCa"® and Kk” Column 11 P. He’ and ,He*, Q.€0, and N,0. RX? and , X74 S.Na’ , F, 0 T. HP and He’ a You have either reached 2 page thts unevalale fer vowing or reached your ievina tit for his book. a You have either reached 2 page thts unevalale fer vowing or reached your ievina tit for his book. a You have either reached 2 page thts unevalale fer vowing or reached your ievina tit for his book. Gareous Slate 93. RP snd Py=n, TE Pryai = P; + P> [for two components only] RT P, (nem) = |R=—"_»,| and |p, =p, ny tn nny =+Partial pressure = Mole fraction Total pressure. =» Partial pressure = Volume fraction x Total pressure (vii) Graham’s law of diffusion Diffusion ‘The process of intermixing or intermi ling of two or more gases against the law of gravity due to movement of molecule of gases by random ‘motion is called diffusion Effusion The process by which a gas comes out from a higher pressure region to lower pressure re- gion through a small orifice or hole is called effusion. Graham's law Under similar condition of tempera- ture and pressure. The rates of diffusion of different gases are inversely proportional to the square roots of their molecular mass or density. Rate of diffusion is always taken as moles of a gas diffused per unit time or volume of a gas diffused per unit time or distance travelled per unit time. In totality, the rate of diffusion is also proportional to the pressure of the gas at a given temperature. Thus, poi fd _ om [Me rep at [ea Vd Vd VM In case of effusion. a V20RTM. Where P= Partial pressure of gas R= Gas constant A= Area of orifice. Other terms have their usual meaning, ( Kinetic Theory of Gases Basic Assumptions: of kinetic theory (i) A gas consists of extremely small discrete par- ticles called molecules. (ii) The molecules of a gas are similar in shape and size but different from other gases. (ii) The gas molecules more randomly in to and fro direction, colliding with one another and with the walls of container. (iv) The collision between molecules are supposed to be perfectly elastic (v) The force of attraction between the gas mol- ecules are supposed to be negligible. (vi) The average kinetic Energy per molecule of ‘gas is proportional to the absolute tempera- ture. (vii) ‘The pressure exerted by a gas is due to colli- sion of the gas. (viii) The volume of a molecule is so small that it may be neglected in comparison to total vol- ume of the gas. BRT M On this basis, Ls D PV = dn v2, = Vane = 22 3 d [where m=mass of gas molecules no of gas molecule 1 > 5 MVing 2 z 5 * Kinetic Enerey. (Exige) z = 3 Frise . 3 = Exsetic= SRT (for one mole) 3 = |Exccic=—NRT| [for n-mole of gas] Velocities of Gas Molecules Gas molecules have tremendous velocities in to and. fro direction. Due to which molecules colloid with each other and their velocities changes. (a) Root Mean Square Velocity The square root of average of the square velocity is called root mean square velocity (V,,,) a You have either reached 2 page thts unevalale fer vowing or reached your ievina tit for his book. a You have either reached 2 page thts unevalale fer vowing or reached your ievina tit for his book. a You have either reached 2 page thts unevalale fer vowing or reached your ievina tit for his book. Gaseous Stale 97 gases we assume planes at a distance of mean free path, Coefficient of viscosity (7)) is given by 1 mKT } t 160" where K is the Boltzmann constant equal to R/N o= diameter of molecule m= Mass of molecule. ‘Molar Heat Capacity ‘The amount of heat energy required to raise the tem- perature of | gm of substance through 1°C is called specific heat capacity. (Its unit is eal.) If the heat energy required to raise the temperature of I mole of substance by 1°C, it is called molar heat capacity. Molar heat capacity = sp. Heat X Molecular mass of gas. Molar heat at constant volume (Cv) and molar heat capacity at constant pressure (Cp) > Cp-C 2eal. For Monoatomic gas, Cp = 5 cal C,=3 cal 1.67. For Diatomic gas, Cp= 7 eal. C Cp 7 Gs Seal Y 1.40 For Triatomic gas C, = 8 cal. and C,-= 6 cal. (Non polar) 8 yes =133. I 1IT WINDOW-1 l > BO > Example 4.1 A gas occupying a volume of 725 ml at a pressure of 0.970 atm is allowed to expand at constant tempera- ture until its pressure becomes 0.541 atm, What is its final volume? > Solution P,=0.970 atm. ¥,=725 ml P,=0.541 atm. Vy Given Since Temperature remain constant, PW = PV, => 0.970 X725= 0.541 x V, => V,= 1299.9 ml Ans, FF Brampte 42 Cyanogen is a compound of carbon and nitrogen. On combustion in excess oxygen 500 ml of cyanogen give 500 ml of N, and 1000 ml, of CO, measured at same temperature and pressure, What is the formula of Cyanogen? > Solution Let the formula of eyanogen be C.Ny, Ly, 2 CN, 0, —> x CO, + 1 mole xmole ~ mole, According to question 500 ml cyanogen give 1000 ml CO, it means x=2 and 500 ml cyanogen give 500 ml N, it means y = 2 ©. the formula of cyanogen = (CN), Ans. PS Baampte a, A gas filled balloon having a volume of 2.5 Lat 1.2 atm and 25°C is allowed to rise to the stratosphere (about 30 km above the surface of the earth) where the temperature and pressure are — 23°C and 3.00 x 10° atm, respectively. Calculate the final volume of the balloon.? > Solution P,=1.2atm, P,=3.00x 10° atm. -23°C Given VF Bamps Dry ice is solid CO; A 0.050 gm sample of dry ice is placed in an evacuated vessel of volume 4.6 L at 30°C. Calculate the pressure inside the vessel after all the dry ice has been converted into CO; (gas).? a You have either reached 2 page thts unevalale fer vowing or reached your ievina tit for his book. a You have either reached 2 page thts unevalale fer vowing or reached your ievina tit for his book. a You have either reached 2 page thts unevalale fer vowing or reached your ievina tit for his book. Goseous Stale 101 Mass of air displaced by balloon _ 12% 4190x108 10° Pay load ~ Mass of air displaced ~ Total mass of bal- loon. = 5028 x 10* gm. 78.13 x 10* gm. Ans. Ans. P> Breample 4.15 A vertical hallow cylinder of hight 152 om is fitted with a movable piston of negligible mass and thick- ness. The lower half of the cylinder contains an ideal gas and upper half is filled with Hg. The cylinder is, initially as shown in figure. If temperature is raised half of the mercury comes out of the cylinder. Find the temperature assuming the thermal expansion of mercury to be negligible.” [Roorkee, 1993] > Solution CS | 76 om Hg 1520m wea” —— P= free em Temp. rised * (0-2 en) LJ Before heating P, =(16 + 76) = 152 em of Hg. T,=300K V, = (of air) = V2. After heating 76 + 38) = 114 em of Hg. P> Bampte 476 20% N,O, molecules are dissociated in a sample of gas at 27°C and 760 torr. Calculate the density of equilibrium mixture? [Roorkee , 1996] > Solution Initially N,0, (g) = 2NO, (g) (mole) 1 9 Ateqlm la 2a Here a=02 ++ Mole of NO, = 0.8 Mole of NO; = 0.4 0.8 x 92-+0.4 x 46 ‘Molecular Mass of Mixture = 16.66 Density = = Pmt. 76.661 Vo RT 0.082130 =3.117 gmL* P> Beampte 4.77 One way of writing the equation of state for real gases is Where B is constant. Derive an approximate expres- sion for B in terms of the Vander Waal constants “a” and *b".2 IIIT, 1997] > Solution Vander-Waal equation is written as RT (For | mole) a You have either reached 2 page thts unevalale fer vowing or reached your ievina tit for his book. a You have either reached 2 page thts unevalale fer vowing or reached your ievina tit for his book. a You have either reached 2 page thts unevalale fer vowing or reached your ievina tit for his book. Goseous State 105 17. A mixture of methane and ethene in the mole ratio X : Y has a mean molecular weight = 20. What would be the mean molecular weight if the same gases are mixed in the ratio Y : X (a) 22 (b) 24 (©) 208 (@ 19 18. If the most probable velocity of hydrogen molecules at a temperature °C is Vo, Suppose all the molecules dissociate into atoms when temperature is raised to (21 + 273)°C then the new rms velocity is? (a) bn () 30427379", © 2/v3K, @) Vor, 19. An open vessel at 27°C is heated until 3/8" of the air in it has been expelled. Assuming that the volume remains constant, The temperature at which the vessel was heated. is? (a) 307°C (b) 107°C. (c) 480°C (a) 207°C 20, The vapour pressure of water at 80°C is 355 mm of Hg. A 100 mL vessel contains water saturated with O, at 80°C, the total pressure being 760 mm of Hg. The contents of the ves- sel were pumped into a 50 mL vessel at the same temperature. What is the partial pressure. of 0,2 (a) 1115 mm (6) 810mm (©) 405 mm (6) 355 mm 21, At STP, 0.48 g of O, diffused through a po- rous partition in 1200 seconds. The volume of CO; diffused under same condition is? (a) 286.5 ml (b) 346.7 mi (©) 112.2 ml (6) 224.8 ml A bubble of gas released at the bottom of a lake increases to eight times its original vol- ume when it reaches the surface. Assuming that atmospheric pressure is equivalent to the pressure exerted by a column of water 10 m. high, what is the depth of the lake? (a) 90m (b) 10m (©) 70m (@) 80m 23. A gas can be easily liquefied. (a) when its inversion temperature equals the Boyle temperature 22, (b) under reversible adiabatic expansion (©) under pressure when it is cooled to be- low the critical temperature (@) at low pressure and above the critical temperature 24, Which of the following is correct for critical temperature? (a) Itis the highest temperature at which lig- uid and vapour can coexist. (b) Beyond this temperature, there is no dis- tinction between the two phases and a ‘gas cannot be liquefied by compression. (©) At this temperature, the surface tension of the system is zero. (@) At this temperature, the gas and the lig- uid phases have difficult critical densi- ties. 25. 40.0 mL of'a gaseous mixture of CO and C,H, is mixed with 100 mL of O, and burnt, The volume of the gas after the combustion is 55 mL, Calculate the composition of the original mixture (a) 25 mL of CO and 15 mL of C,H, (b) 15 mL of CO and 25 mL of C,H (©) 10 mL of CO and 30 mL of CH, () 20 mL of CO and 20 mL of C,H 26. 80 mL of oxygen is added to 50 mL of a mix- ture of Hy, CoH, and CO, after which the total mixture is bumt. The volume of the cooled mixture after combustion measures 65 mL. This is reduced to 15 mL by treatment with a KOH solution. Calculate the volume of each gas in the original mixture. (@) 20 mL of H,, 20 mL of C,H, 10 mL of co (b) 10 mL of H,, 20 mL of C, co (©) 15 mL of Hy, 15 mL of C, co (@) 20 mL of H, 25 mL mL of C,H, 5 mL. of CO 27, 50 mL of a mixture of C,H,, and C,H; was mixed with 150 mL of O, and burnt. The vol- ume of the cooled mixture of gases after the combustion is 112.5 mL. The percentage of CH, in the original mixture is, |, 20 mL of, 20 mL of a You have either reached 2 page thts unevalale fer vowing or reached your ievina tit for his book. a You have either reached 2 page thts unevalale fer vowing or reached your ievina tit for his book. a You have either reached 2 page thts unevalale fer vowing or reached your ievina tit for his book. Goseous Stale 109 23. 24, 25, 26. 2. Calculate, (a) what volume of gas would re- main and (b) what weight of iodine would be liberated assuming the volume to have been measured at NTP? A certain container has 5 x 10° molecules of a gas A at a pressure of 900 torr. By a photo- chemical combination allowing light to fall on the molecules, the pressure drops to 500 due to dimerisation. If the temperature and vol- lume remain the same, how many monomer, dimmer and total molecules are present after exposure to light? Acar tyre has a volume of 10 litre when inflat- ed. The tyre is inflated to a pressure of 3 atm at 17°C with air. Due to driving the temperature of tyre increases to 47°C. (a) What would be the pressure at this tem- perature? (b) How many litre of air measured at 47°C and pressure of 1 atm should be let out restore the tyre to 3 atm at 47°C? Amixture of N, and water vapours is admitted to a flask which contains a solid drying agent. Immediately after admission, the pressure of the flaks is 760 mm. After standing some hours, the pressure reached a steady value of 745 mm, (a) Calculate the composition in mole % of original mixture. (b) Ifthe experiment is done at 20°C and the drying agent increases in weight by 0.15 what is the volume of flask? In a space shuttle, the CO, output per astro- naut has been estimated as 44g per hour. cata- lytic converter reduces CO, into H,O at a rate of 600 ml. (at STP) per min. What fraction of the time would such a converter be operated in order to keep up the CO, out of one astro- naut.? A container holds 3 litre of Ni, and HO, at 29°C. The pressure is found to be 1 atm. The water in container is instantaneously electrol- ysed to give H, and O, following the reaction, 1 HO —> Hay + 5 Ong) At the end of electrolysis the pressure was found to be 1.86 28. 29, 3. 32. 33. atm, Calculate the amount of water present in the container if the aqueous tension of water at 29°C is 0.04 atm.? A closed bulb contains 0.01 mole of inert he- lium gas and a sample of solid white NH,Cl The pressure of the He is measured at 27°C and is found to be 114 mm Hg. The bulb then heated to 327°C. All the NH,CI decomposes according to the equation. NH,Cl, —> NHyq +HCly) The final total pressure in the bulb after com- plete decomposition of solid is 908 mm Hg. ‘Assume all the gases are ideal (a) What is the partial pressure of HCl.pin the bulb at 327°C when reaction is eom- plete? (6) How many grams of NH,Cl the bulb at 27°C? An under water bubble with a radius of 0.5 em at the bottom of tank, where temperature is 5°C and pressure is 3 atm rises to the surface where temperature is 25°C and pressure is 1 atm, What will be the radius of bubble when it reaches to surface.? An open vessel at 27°C is heated until three- fifths of the air in it has been expelled. Assum- ing that the volume of the vessel remains con- stant, find the temperature to which the vessel has been heated.? Aspherical balloon of 21 cm diameter is to be filled with hydrogen at N.T.P. from a cylinder containing the gas at 20 atm at 27°C. If the cylinder can hold 2.82 litres of water, calculate the number of balloons that can be filled up. Calculate the relative rates of diffusion for SUF, and UF, in gaseous form, Also if naturally occurring uranium ore having U?*> and U* as 0.72 and 99.28 percent by moles. and if it is desired to enrich the U"™ to 10% of the sample making use of relative rates of dif- fusion of UF, having U* and U™* isotopes, how many diffusion stages are required.? A thin tube of uniform cross-section is sealed at both ends. It lies horizontally. The middle 5 cm containing Hg and the two equal ends horizontally. The middle 5 cm containing Hg |) were in a You have either reached 2 page thts unevalale fer vowing or reached your ievina tit for his book. a You have either reached 2 page thts unevalale fer vowing or reached your ievina tit for his book. a You have either reached 2 page thts unevalale fer vowing or reached your ievina tit for his book. Goseous Stale 113 10. 20 dm’ of SO, diffuses through a porous pa- tition in 60 second. What volume be O, will diffuse under similar condition in 30 second? [Roorkee, 1996] 11, One mole of nitrogen gas at 0.8 atm takes 38, seconds to diffuse through a pinhole whereas ‘one mole of an unknown compound of xenon with fluorine at 1.6 atm takes 5 seconds to dif- fuse through the same hole. Calculate the mo- lecular formula of the compound, IIIT, 1999] 12, The composition of the equilibrium mixture (Ch = 2C)), which is attained at 1200°C is determined by measuring the rate of effusion through a pinhole, It is observed that a 1.80 mm Hg pressure, the mixture effuses 1.16 times as fast a krypton effuses under the same conditions. Calculate the fraction of chlorine molecules dissociated into atoms (Atomic wt of Kr=84). IIIT, 1995} 13. A mixture containing 1.12 litre D, and 2.24 litre of H, at NTP is taken inside a bulb con- nected to another bulb through a stop cock with a small opening. The second bulb is ful- ly evacuated. The stop cock is opened for a certain time and then closed. The first bulb is now found to contain 0.10 g of D,. Determine the % by weight of the gases in second bulb. [Roorkee, 1998] 14, At27°C, H, is leaked through a tiny hole into a vessel for 20 minute. Another unknown gas at the same T and P as of the H, is leaked through the same hole for 20 minute. After the effusion of the gases, the mixture exerts a pressure of 6 atm. The H, content of the mi ture is 0.7 mole. If volume of container is 3 litre, what is mol. wt. of unknown gas? HIT, 1992] 15. The average speed at 7, K and the most prob- able speed at T, K of CO, gas is 9 x 10° cm. sec”, Calculate the value of 7, and T; IIT, 1990] 16. The average speed of an ideal gas molecule at 27°C is 0.3 m sec ', Calculate average speed at 927°C IHIT, 1986] 17. Aglass bulb of | litre capacity contains 2x 10”! molecules of nitrogen exerting pressure of 7.57 x 10° Nm”. Calculate the root mean square speed and the temperature of gas molecules. If the ratio Of Clay, 10 ting i8 0.82, calculate for these molecules at this temperature. (UIT, 1993] 18, The average velocity of gas molecules is 400 m/sec. Calculate its rms velocity at the same temperature. [HIT, 2003] 19. (a) Calculate the pressure exerted by 5 mole of CO, in one litre vessel at 47°C using van der Waal’s equation, Also report the pressure of gas if it behaves ideally in nature. Given that a = 3.592 atm litre mol *, b = 0.0427 litre mol *. (b) If volume occupied by CO, molecules is negligible, then calculate the pressure exerted by one mole of CO, gas at 273 K. a= 3.592.dm’ atm mol? IIT, 2000] 20. Using van der Waal’s equation, calculate the constant, ‘a’ when two mole of a gas confined in a four litre flask exerts a pressure of 11.0 atmospheres at a temperature of 300 K. The value of “bis 0.05 lit. mol’. [IIT, 1998] 21. One way of writing the equation of state for Clip, > is constant, Derive an approximate expression for B in terms of van der Waal’s constants a and b. IIT, May 1997] I (IT WINDOW-V l > Ltr ews 1. The value of the molar gas constant is, (a) 8.3145 x 10° J (Kg mol) 'K* (b) 1.987 cal mol K" (©) 0.083145 10° dm’ bar mol ' K (d) 0.083145 dm’ bar mol ' K a You have either reached 2 page thts unevalale fer vowing or reached your ievina tit for his book. a You have either reached 2 page thts unevalale fer vowing or reached your ievina tit for his book. a You have either reached 2 page thts unevalale fer vowing or reached your ievina tit for his book. Goseous State 17 Vi. Match the compression factor under different condition (in list 1) with its value (in list 11) List ‘Compression factor (2) for ideal gas Z for real gas at low P Z for real gas at high P Z for ertical state pope Match the different temperatures (in list 1) with its value (in list 11) List List poe Critical temperature Boyle temperature Inversion temperature Reduced temperature 4, aiRb 2aiRb Tie 84/27 Rb vu. Match the factor p D. in column I with column IL Column t Column I Diffusion of gas 1. Temperature Density 2. Pressure Critical 3. Attractive forces temperature Kinetic Energy 4. Molecular Mass. VIII. Match the column I & column II. Ix. A Column I Column It Hydrogen gas 1, Compressibility fac- (P= 200 atm, toré1 1=273k) Hydrogen gas 2. Attractive forces are (P=0,7=273k) dominant CO, (P= Latm T 3. PY =nRT =273K) Real gases with 4, P(V—n)=nkT. very large molar Mass Column I Column 11 Boyle's temperature. (Tp) 1. a/b Inversion temperature. (Ti) 2. 3a/Rb Critical temperature. (Te) 3. 2a/Rb List 318 (1 +pbiRT) 1 (alk) Column Column 1 D._ Kraft temperature 80 Rb T. Minimum temperature for micell formation, I IIT WINDOW-VII l > ESE 1, Statement-I: The ideal gas could be easily liq- uefied. Statement-II: The ideal gas has no attractive or repulsive forces. Statement-1 ‘The experimently determined value with critical parameters of 'R’ is much different from 8,314 JK‘ mole! Statement-Il: The simple Vander Waal’s at- traction doesn’t exist at the cr al state, Statement-I : Compressibility factor (z) for a non-ideal gas could be greater than 1 Statement-II: Non-ideal gas molecule may have significant intermolecular attraction. ‘Statement : In the plot of PV Vs P. the devia- tion dip from ideal state for N,,CH, and CO, gradually increases Statement-II: The deviation from ideality de- pends on the nature of gas. Statement-I: The rate of diffusion or effusion can be assumed to be directly proportional to the root mean square speed or any other aver- age speed. a You have either reached 2 page thts unevalale fer vowing or reached your ievina tit for his book. a You have either reached 2 page thts unevalale fer vowing or reached your ievina tit for his book. a You have either reached 2 page thts unevalale fer vowing or reached your ievina tit for his book. Goseous Stale 121 where 1 is number of moles of gases ‘a’ and *b’ are Vander Waals constants both being characteristic of a gas. (1) Ifthe observed molar volume of a gas at 25°C is 20 litre, The gas (a) do not deviate from its ideal behav- jour (b) deviates from its ideal behaviour and become more compressible (6) deviates from its ideal behaviour and become less compressible (A) none of these. (2) The dip of the curve shown above, shows that (a) Repulsive force between gaseous particles dominates (b) Attractive force between gaseous particles dominates (©) The magnitude of repulsive force decrease with increase in pressure (@) The magnitude of attractive force i creases with increase in pressure. (3) The unit of Vander Waal constant ‘a’ is (a) atm litre mole! (b) atm litre” mole (c) atm mole” litre (d) atm mole litre! 5. When the pressure is considerably high, /, will be quite small. The pressure correction {may become negligible and Van der ‘Waal’s equation reduces to P (Vq—nb)=nRT This explains why the PY, after reacting a minimum value, increases with increase in pressure. For a real gas, PV versus P plot is not linear. Hence Pip or m/PV is not indepen- dent of P for calculation of p and molecular mass M we must obtain P/p at P= 0. As P—> 0, P (V— nb) =aRT SPV =n (B+ pb)= (RT+ Pd) _m (2 2) (eet) Pv ara) Pw 6. PM Mb p RT (rty (1) The nature of graph plotted for P on x- axis versus bp. on y-axis will be: (a) Hyperbola (b) Straight line (©) Parallel to x-axis (@) None of these (2) Calculate the density of N, gas at room temperature of 25°C and atmospheric pressure, given that b= 0.0694 dm’ mol! (@) 136 gL (b) 0.568 gL (©) 227gL —@) 136gL (3) The slope of the graph is M P Le 2 © ar > o “4 (@) None of these (ary th No of molecules Molecular speed Graph shows typical Maxwell speed distri- bution curves for N, gas at three different temperatures 7, 7, and T,, The peak of each curve represents the most probable speed. On increasing temperature, curve begins to flat- ten out and shift towards right, But at constant temperature T, same nature of graph is ob- tained for three gases. A, B and C correspond- ing to T, T, and T;, In an apparatus a beam of atoms (or molecules) exits from an oven at a known temperature and passes through a pin- hole. Two circular plates mounted on the same shaft is rotated by a motor. The first plate is called chopper which allows small bursts of atoms and second plate is detector. Eventually a You have either reached 2 page thts unevalale fer vowing or reached your ievina tit for his book. a You have either reached 2 page thts unevalale fer vowing or reached your ievina tit for his book. a You have either reached 2 page thts unevalale fer vowing or reached your ievina tit for his book. Gaseous Stale 125 13.(0)(€).(0) 14.(@.(0).(15.@).(6) ANSWER KEYS: IIT WINDOW-VIL l 16. (a), (d) 17. (b), (c), (4) 18. (a), (b), (©) 1, dora? 2. Lor‘a’ 3. 2or‘b? 4. 3or'e’ I ANSWER KEYS: IIT WINDOW-VI l 5. Lor‘a” 6. Lor‘a’ 7. Lor‘a’ 8. Lor‘a’ 1A-3 W Ad 9. Lora’ 10. 3 orc’ B-4 B-3 cos cl I ANSWER KEYS: IIT WINDOW-VIII l D-2 D-2 E-1 IV A-3 0) 1.@ 2] .@ 2.(b) 2.(d) ml A-4 B-4 3.(a) 3. (a) B-3 c-1 4.0) 4.(b) Get D-5 GB] 1.@) (41 1. D-2 E-2 2@ 2.(b) Vv A-3 VI A-4 3.(@) 3.(b) Bo4 Bol 4.0) c-2 c-2 [5] 1.@) [6] 1.) D-1 D-3 2.(a) 2.) VIL A=1,2 Vill A-P,S 3.0 3.0) B-2,3,4 B-R (7) 1.@ c-1 c-R9 2.00) D-1,4 D-R 3d) IX A-1 B-3 c-4 D-s a You have either reached 2 page thts unevalale fer vowing or reached your ievina tit for his book. a You have either reached 2 page thts unevalale fer vowing or reached your ievina tit for his book. a You have either reached 2 page thts unevalale fer vowing or reached your ievina tit for his book. Chemical Bonding 129 Non-polar solvents like benzene and carbon tet- rachloride do not solvate the ions as their dielectric constants are low. Ionic compounds are, therefore insoluble in non-polar solvents. Ionic compounds like sulphates and phosphates of barium and strontium are insoluble in water (because lattice energy is greater than hydration energy). This can be attributed to the high lattice energies of these compounds due to polyvalent nature of both the cat- jon and the anion. In these cases, hydration of ions fails to liberate sufficient energy to offset the lattice energy. Covalent Bond (By Mutual Sharing of Electrons) ‘The covalent bond is formed when two atoms achieve stability by the sharing of an electron pair, each con- tributing one electron to the electron pair. The arrangement of electrons in a covalent mol- ecule is often shown by a Lewis structure in which only valency shells (outer shells) are depicted. For sake of clarity, the electrons on different atoms are denoted dots and crosses. Polarity of Bonds A covalent bond is set up by sharing of electrons be- tween two atoms, It is further classified as polar or non-polar depending upon the fact whether the elec- tron pair is shared unequally between the atoms or shared equally. For example, the covalent bonds in H, and Cl, are called non-polar as the electron pair is equally shared between the two atoms. In the case of hydrogen fluoride the bond is polar as the electron pair is unequally shared. Fluorine has a greater attraction for electrons or has higher elec- H:H Cl Hydrogen molecule (Both formed by equal sharing of electrons between the atoms, i, by non-polar bonds) Chlorine molecule tronegativity than hydrogen and the shared pair of electrons is nearer to the fluorine atom than hydrogen atom. The hydrogen end of the molecule, therefore, appears positive with respect to fluorine. Bond polarities affect both physical and chemi- cal properties of compounds containing polar bond, The polarity of a bond determines the kind of reac- tion that can take place at that bond and even affects the reactivity at nearby bonds. The polarity of bonds can lead to polarity of molecules and affect melting point, boiling point and solubility. Dipole Moment It is vector quantity and is defined as the product of the magnitude of charge on any of the atom and the distance between the atoms. It is represented by 4 Magnitude of dipole moment | 4 |= — (hanes inewy * TBisanss AY The unit = 10°" (esu) cm (D) is used in practice. In SI units charge q is measured in coulombs (C) and the distance, - in metre, m 1 C= 2.998 x 10” esu and 1 m= 10? Cm + 1Cm = 2,998 x 10° x 10° = 2.998 x 10!" (esu) em ‘Therefore in SI system, the unit of dipole moment is coulomb metre 1 bem = 2:998%10 > = 2.998 x10" D or w=! -3336x10 cm 2.99810" Dipole moment is a vector quantity and is often in- dicated an arrow parallel to the line joining the point of charge and pointing towards the negative end e.g., H-F % lonic character of a covalent bond +6 bd Fig. 5.1 a You have either reached 2 page thts unevalale fer vowing or reached your ievina tit for his book. a You have either reached 2 page thts unevalale fer vowing or reached your ievina tit for his book. a You have either reached 2 page thts unevalale fer vowing or reached your ievina tit for his book. Chemical Bonding 133 100. 100 3 H.0 § of o g g |Me 2 2 a 2-100 f -100-4 = ASH; 3S 2 Phy ° Sak 3 ca chy Sol eo LT | wo Li — Molecular weight increasing > — Molecular woight increasing —> (a) (b) Fig. 5.3 Having no power to form H-bonds, the simple carbon family hydrides (SnH,, GeH,, SiH, and CH,) show a decrease in their bp’s and mp’s with the de- crease in their molecular weights. (ii) Ice has less density than water. ‘The explanation of this fact is as follows: In the crystal structure of ice, the O-atom is surrounded by four H-atoms, ‘Two H-atoms are linked to O-atom by covalent |» 0.964 MH. }-——> Water molecule bonds as shown (by normal covalent bond) and the remaining two H-atoms are linked to O-atom by two H-bonds shown by dotted lines. Thus in ice every water molecule is associated with four oth- er water molecules by H-bonding in a tetrahedral fashion. Ice has an open cage like structure with a large empty space due to the existence of H-bonds. As ice melts at 0°C, a number of H-bonds are ‘Open cage-like tetrahedral crystal structure of ice. Circles indicate oxygen atoms Bonds represented by solid line are normal covalent bonds while those represented by dotted lines are hydrogen bonds. Fig. 5.4 a You have either reached 2 page thts unevalale fer vowing or reached your ievina tit for his book. a You have either reached 2 page thts unevalale fer vowing or reached your ievina tit for his book. a You have either reached 2 page thts unevalale fer vowing or reached your ievina tit for his book. Chemical Bonding 137 pair repulsions are weakest. Groups at 90° repel each other strongly, while groups 120° apart repel each other much less. Structure I is the most symmetrical, but has six 90° repulsions between lone pairs and atoms. Structure II has one 90° repulsion between two lone pairs, plus three 90° repulsions between lone pairs and atoms. These factors indicate that structure I] is the most probable, The observed bond angles are 80°40’, which is close to the theoretical 90°. This confirms that the correct structure is III, and the slight distortion from 90° is caused by the presence of the two lone pairs, Asa general rule, if lone pairs occur in a trigonal bipyramid they will be located in the equatorial posi- tion (round the middle) rather than the axial positions (top and bottom), since this arrangement minimizes repulsive forces. phur hexafluoride (SF,): The electronic struc- ture of S is Is"2s* ‘2s°2p°3s"3p”. Alll six of the outer elec- trons are used to from bonds with the F atom. Thus in SF,, the $ has six electron pairs in the outer shell: hence the structure is octahedral. There are no lone pairs, so the structure is completely regular with bond angles of 90°. F F Fig5.7 (CYalence Bond Theory This theory was produced by Linus Pauling, who was awarded the Noble Prize for Chemistry 1954 Atoms with unpaired electrons tend to combine with other atoms which also have unpaired electrons. In this way the unpaired electron are paired up, and the atoms involved, & all attain a stable electronic arrangement. This is usually a full shell of electrons (ie, anoble gas configuration). Two electrons shared. between two atoms constitute a bond. The number of bonds formed by an atom is usually the same as the number of unpaired electrons in the ground state, ic. the lowest energy state. However, in some cases the atom may form more bonds than this. This occurs by excitation of the atom (i.e. providing it with energy) when electrons which were paired in the ground state are unpaired and promoted into suitable empty orbit- als. This increases the number of unpaired electrons, and hence it increases number of bond which can be formed. A covalent bond results from the pairing of elec- trons (one from each atom), The spins of the two elec- trons must be opposite (antiparallel) because of the Pauli exclusion principle that no two electrons in one atom can have all four quantum numbers the same. 1, In HF, H has a singly occupied s-orbital that overlaps with a singly filled 2p orbital on F. 2. In H,O, the O atom has two singly filled 2p orbitals, each of which overlaps with a single occupied s-orbital from two H atom: 3. In NH,, there are three singly occupied p or- bitals on N which overlap with s orbitals from three H atoms. 4. In CH,, the C atom in its ground state has the electronic configuration Is, 2s”, 2p\, 2p, and only has two unpaired electrons, and so can form only two bonds. If the C atom is excited, then the 2s electrons may be unpaired, giv- ing Is’, 2s”, 2p,, 2p), 2p,. There are now four unpaired electrons which overlap with singly occupied s orbitals on four H atoms. 2p 2s 2px 2Py 2, Hleowonie structure A) FA) Fa Tg of carbon atom- ground state Is 2s 2p Carbon atom- excited state 4) 4 1/4\4 Carbon atom having gained four electrons | 4h] | th from H atoms in CHy molecule A sp° hybridisation a You have either reached 2 page thts unevalale fer vowing or reached your ievina tit for his book. a You have either reached 2 page thts unevalale fer vowing or reached your ievina tit for his book. a You have either reached 2 page thts unevalale fer vowing or reached your ievina tit for his book. Chemical Bonding 141 Pr, NH; C10; X=L[5+5] Hybrid state of P is X=! 5=1) X=Y0+7+ 1) Hybrid state of N is Hybrid state of Clis sp'd 5 9° NOy XeFy X=}[0+5+1] x se 7 pe Pcl PH SF SF, X=H6+541] X=13+5] X=113+6-1] X=h14+6] =4 =4 Hybrid state Hybrid state Hybrid state Hybrid state spe 5 s spd Molecular Orbital Theory Why He, molecule does not exist and why O, is paramagnetic? These questions cannot be explained by valence bond theory. In 1932 F, Hund and R.S. Mulliken put forward a theory known as Molecular Orbital Theory to explain above questions and many others. According to this theory, as the electrons of an atom are present in various atomic orbitals, elec- trons of a molecule are present in various molecular orbitals, Molecular orbitals are formed by the com- bination of atomic orbitals of comparable energy and proportional symmetry. While an electron in atomic orbital is influenced by one nucleus, in a molecular orbital, itis influenced by two or more nuclei depend- ing upon the number of atoms in the molecule. Thus, an atomic orbital is monocentric while a molecular orbital is polycentric. The number of molecular or- bitals formed is equal to the number of combining atomic orbitals. When two atomic orbitals combine, two molecular orbitals are formed . One is known as bonding molecular orbital (BMO) whereas other is anti-bonding molecular orbital (ABMO). BMO has lower energy and hence greater stability than the corresponding ABMO, First BMO are filled, then ABMO starts filling because BMO has lower energy than that of ABMO. Molecular orbitals like the atomic orbitals are filled in accordance with the Aufbau Principle obey- ing the Pauli’s Principle and the Hund’s rule, Order of energy of various molecular orbitals is as follows: For O, and higher molecules Gs, ols, 62s, 6*2s, 6 2po[n2p, = 2p.) [n*2py n*2pz], o*2px For N, and Lower molecules Ols, o*ls, 02s, 0*2s, (2p, = 1 2p,], 62p,, [R*2p, = n*2p,], o*2p, Bond order It may be defined as the half the difference between the number of electrons present in the boding orbitals and the anti-bonding orbitals i.e. Bond order (B.O) __ No.of e“in BMO- No. of e"in ABMO 2 A positive bond order suggest stable molecule while zeto or —ve B.O suggest unstability of the species. a You have either reached 2 page thts unevalale fer vowing or reached your ievina tit for his book. a You have either reached 2 page thts unevalale fer vowing or reached your ievina tit for his book. a You have either reached 2 page thts unevalale fer vowing or reached your ievina tit for his book. Chemical Bonding 145 > Solution Since both the groups attached on benzene ring are the same, their dipole moment is decided by the for- mula sa abt pt 2 con High the value of @, lesser is the value of 4. Thus cl SO © I IIT WINDOW—II l > (MST 1. Which of the following is required for the for- mation of an ionic bond? (a) An electron from the more clectronega- tive element should be transferred to the less electronegative element. (b) The total energy of the resulting molecule should be less than the total energy of the reactants, (©) The lattice energy of the resultant mol- ecule should be as low as possible. (d) The ionic potentials of the reactants should be identical. 2. Which ofthe following statementsisineorrect? (a) NH, is more basic than PH, (b) NH, has a higher boiling point than that of HF (©) Nj is less reactive than P., () The dipole moment of NH, is less than that of SO,. AICI, is covalent while AIF, is ionic. This can be justified on the basis of (a) the valence-bond theory (b) Fajans’ rules (©) the molecular-orbital theory (4) hydration energy 4. Which of the following pairs have nearly identical values of bond energy ? (a) O, and H, (b) N, and CO (©) Fyand 1, (d) O, and Cl, 5. Which of the following is the most ionic ? (a) PLO (b) MnO (©) C10; (@) Mn,0, 6. Among LiCl, BeCh, BCI, and CCl, the cova- lent bond character varies a (a) LiCl < BeCl, > BCI, > CCl, (b) LiCl > BeCl, < BCl, < CCl, (©) LiCl < BeCl, < BCI, BeCl, > BCI, > CCl, 7. Ina metallic erystal the (a) valence electrons remain within the fields of influence of their own kernels (b) valence electrons constitute a sea of mo- bile electrons (©) valence electrons are localized between the two kemels (d) Kernels as well as the electrons move rap- idly Polarization involves the distortion of the shape of an anion by an adjacently placed cation, In this context, which of the following statements is correct ? (a) Maximum polarization is brought about by a cation of high charge. (6) Maximum polarization is brought about by a cation of low radius (©) A large cation is likely to bring about a high degree of polarization (d) The polarizing power of a cation is less than that of an anion, 9. Which of the following oxyacids of phospho- us are monoprotic (monobasic) ? (a) HPO, (b) H:PO, (c) H,PO, (d) H,P,0, 10. Which of the following has greater bond length ? (a) P—O (b) S—O (©) C—-0 (@) 0-0 11. Which of the following has been arranged in order of increasing covalent character? (a) KCl 3>2 (©) 1>2>3 @ 2>1>3 3. Considering a diatomic molecule HCI, its dipole moment and bond length determined experimentally are sHCI = 1.03 Debye, and 1.275 A respectively. What is the charge con- tained by each dipole (2) 8.0710" esu — (b) 4.03 x 10 "es (©) 16.15 x10" esu (a) 2.01 x10" esu 4. Among LiCl, BeCl,, BCl, CCl, covalent bond character follows the order: (a) LiCl > BeCl, > BCI, > CCI, (b) LiCl < BeCl, < BC, < CCl, (©) LiCl > BeCl, > CCI, > BCI, (@) LiCl < BeCI, < BCI, > CCI, (ID) Dimethyl glyoxime is an important reagent, which is used in the detection of Ni** ion in fourth group of qualitative analysis and also in its gravimetric estimation. The compound formed by Ni** ions with dimethyl glyoxime is diamagnetic with the structure as given below ‘a’ and ‘b’ are C ~ C bond length” 1. Which of the following is correct regarding, the bond length ‘a’ and *h” (a) a=b (b) a>b (©) b>a (d) none of the above 2. How many sigma bond are associated with carbon atom in the given structure are: (@ 5 (b) 10 (© Il @) 22 The atomic orbitals which the central atom is using in hybridisation are: (a) 4s. 4p,, 4,, p, and 4p, (b) 3dz*, 4s, p, and 4p, (©) 3d.3_. 48, 4p, and 4p, 3. (A) 3d.2_2. 48, 4p, and 4p, 4, The necessary conditions for the formation of intramolecular hydrogen bonding is (a) The ring formed as a result of hydrogen bond should be planner (b) A five or six membered ring should be formed (©) Interacting atoms should be placed in such a way that there is minimum strain during the ring formation (d) Allof these (UID) The degree of polarity of a covalent bond is measured by the dipole moment (4 bond) of the bond defined as bond = charge on one of the poles x bond length j bond is a vec- tor quantity. The dipole moment of a molecule is the vector addition of all the bond moment present in it. For a triatomic molecule contain- ing two bonds like H,O, molecule is given by HF molecule = Hoga * Hons *2ithona H bond cos @ = bond angle # molecule is an experimental property so is also the bond length is measured by mi- crowave spectroscopy which is based on the principle that absorption of microwave radia- tion by a molecule causes rotational transition of molecules. However, a molecule with per- ‘manent dipole moment equal to zero is micro- wave inactive, a You have either reached 2 page thts unevalale fer vowing or reached your ievina tit for his book. a You have either reached 2 page thts unevalale fer vowing or reached your ievina tit for his book. a You have either reached 2 page thts unevalale fer vowing or reached your ievina tit for his book. Chemical Bonding 157 [answer keys: rt winnow-v lcs c—Qs D—T D—P,Q,R,T M Le) db Lib) (Ib Lee) 3.A—T 4.A—P,R 2.) 2.(d) 2.(d) B—Q B—P,R 3.) 3.(0) 3.(a) cos c-a@ 4. (b) 4.4) 4. (b) D—Ss D—Ss dy) 1b) (V) L@ (VD Lo) 5.A—P 6.A—P 2. (b) 2. (b) 2. (d) =-é BRS 3.(c) 3.(a) 3.(@) c—Q cP 4.(0) 4© D—R D—P.Q 5. (b) TA—QR 8.A—QS B—Q B—P,R,S [answer Kes: 1rt winnow-vi | es CPR D—QS D—R,S LA—s 2A—P.Q,R B—S B—P,Q a You have either reached 2 page thts unevalale fer vowing or reached your ievina tit for his book. a You have either reached 2 page thts unevalale fer vowing or reached your ievina tit for his book. a You have either reached 2 page thts unevalale fer vowing or reached your ievina tit for his book. Theory of dilute Solution 161 3 Xe Hever g oo 0010 oda0 Mole fraction of HCI gas (in cyclohexane) Partial pressure of Fig 6.1 solution. Or Mole fraction of the gas in solution is proportional to the partial pressure of the gas. Mathematically, Henry's Law can be expressed as mo PorpsX-X uP or p= Ky X where X is the mole fraction of gas. ‘mis the mass of gas dissolved per unit volume. P is the pressure of the gas in equilibrium with the solution. K,, is proportionality constant. Value of Ky, depends upon nature of gas and tem- perature. Higher the value of K;, at a given pressure, lower is the solubility of the gas in the liquid. Ky value for O, and Ny increases with increase in tem- perature indicating that solubility of gases decreases with increase of temperature. Note: Aquatic species are more comfortable in cold water rather than warm water because solubility of gases decreases with increase of temperature. Henry’s Law is applicable only if 1. The pressure is not too high. 2. The temperature is not too low: 3. The gas is not highly soluble i.e. the gas does not react with solvent. When a mixture of gases is brought in contact with a given solvent then the solubility of the gas is propor- tional to its partial pressure. Nature of Solute Easily liquefiable gases are more soluble in water and the gases that can form ions in aqueous solutions are more soluble. hods of Expressing th Concentration of a Solution (i) Mass percentage It is defined as the amount of solute in grams present in 100 grams of the solution Mass of solute Mass percentage of solut 100) ‘Mass of solution Mass of solu Mass of solute + Mass of solvent 100 Mass of solute = or —___x100 Volume of solution x Density of solution Mass of solute jis termed as mass Mass of solution The ratio fraction, Mass pe Thus, i Mass fraction of solute x 100) olu ntage of (ii) Percent by Volume Percent of solute by volume Volume of sol _ Volume of solute 99 Volume of soution (ii) Percent mass by Volume Percent of solute Mai Volume of solution of solute mass by volume = 100 (iv) Strength (Grams per litre) Concentration of solution = ——M2s8 of solute in grams Volume of the solution in litres Mass of solute in grams ooo 1000 ‘Volume of the solution in mL (v) Parts per Million (ppm) __Massof solute gs Ppm ‘Mass of solution (vi) Mole fraction It is defined as the ratio number of moles of one component to the total number of moles of the solution (i.e., all the components). Let n moles of solute (4) and N moles of solvent (B) be present in a solution. Mole fraction of solute = a You have either reached 2 page thts unevalale fer vowing or reached your ievina tit for his book. a You have either reached 2 page thts unevalale fer vowing or reached your ievina tit for his book. a You have either reached 2 page thts unevalale fer vowing or reached your ievina tit for his book. Theory of dilute Solution 165 Lowering in vapour pressure = py ~ p, a Po According to Raoult’s Law: “Relative lowering in vapour pressure of a dilute solution is equal to mole fraction of the solute present in the solution.” If n moles of solute be dissolved in N moles of Relative lowering in vapour pressure = the solvent, the mole fraction of the solute will be nN According to Raoult’s law, Wy/My Ws, Wa Ma My = WM Po Ma Ws Sincen +N=N This is the mathematical expression for Raoult’s law. Modified form of Raoult’s Law: The above rela- Po _ntN | N ip can be written as, 1s Po-Py ” N or at. n or PoP rN Note: Equation (2) provides more accurate result during ‘mathematical calculation, Determination of Lowering of vapour pressure Lowering vapour pressure is determined by Ostwaid and Walker dynamic method. It is based on the prin- ciple when air is allowed to pass through a solvent or solution, it takes up solvent vapour with it to get itself saturated with vapours at that temperature. Set A and B are weighed separately before and after passing dry air. Loss in weight of each set, gives 0 8 A Fig6.3 the lowering of vapour pressure. The temperature of air, the solution and the solvent is kept constant. Loss in weight of solution (w,) P, Loss in weight of solvent (w,) e= P*— P, in in weight of anhydrous CaCl, & P° Elevation of Boiling Point (Ebullioscopy) The boiling point of a liquid is the temperature at which its vapour pressure is equal to the atmo- spheric pressure. The vapour of a liquid is lowered when a non-volatile solute is added to it. Hence, the temperature of the solution when its vapour pressure will be equal to atmospheric pressure will be higher than the temperature of the pure solvent. The dif- ference in the boiling point of the solution and the boiling point of the pure solvent is termed elevation of boiling point. T,, T, and T, represent the boil- ing points of pure solvent, solution I and solution IT respectively. The vapour pressure of pure solvent, solution I and solution Il at temperature T, are P,, P, and P, respectively. From ABAD & ACAE, we have 8 or or or AT & AP ‘Atmospheric s Temperature 0 3! 7? Fig 6.4 a You have either reached 2 page thts unevalale fer vowing or reached your ievina tit for his book. a You have either reached 2 page thts unevalale fer vowing or reached your ievina tit for his book. a You have either reached 2 page thts unevalale fer vowing or reached your ievina tit for his book. Theory of dilute Solution 169 = Kyye = 0.120 xy. = moliy260 1000 = Molatity = 1000%0.120 5 srotal a Ans. Vy tames2 OS™ At a certain temperature vapour pressure of pure ether is 646 mm and that of pure acetone is 283 mm, Calculate the mole fraction of each component in the ‘vapour state if the mole fraction of ether in the solu- tion is 0.50.? > Solution From Raoult’ lav. Prout = Paccone * Patter = P°ncetone Xacetone * Petner Xeter = Py= 646 x 0.5 +283 x 0.5 = 464.5 mm of Hg. . _ PRX, _ 6460.5 “ Prog 4645 = 0.695. = PaXu — 28305 Frou 464.5 = 0.305. > Example 6.3 A Decimolar solution of potassium ferrocyanide K, [Fe(CN),] is 80% dissociated at 27°C in a par- ticular medium. Calculate the osmotic pressure of the solution? > Solution Molar Concentration of solution = M 0.1M, K, [Fe(CN),] 4K" + [Fe(CN),J* Initially 1 0 0 After 1-08 08 dissociation 4x08 Total mole after dissociation = 0.2432408=42 42 i (Vant’s Hoff factor) = == 42 iCRT. = 4.2 x 10! x 0.0821 x 300 10.3446 atm. V5 Brampte 64 A millimolar solution of acetic acid demerises to the extent of 60% in benzene at 27°C. Calculate the os- ‘motic pressure of acetic acid in benzene ? > Solution = cose, (0 HON — c—cH. So—H-- 0% * Dimer of acetic acid (CH, COOH — (CH,COOH), CHy Initially 1 0 After 1-06 03 association Total mole after association = 0.4 + 0.3 = 0.7 i (van't Hoff factor 07 Hl icRT 0.7 x 10° x 0.0821 x 300 = 17.241 x 10° atm, Ans, V5 Bampre 6s Calculate the molecular mass of Cellulose acetate if its 0.5% (wt/mole) solution in acetone having sp. gravity 0.9 gm/ml shows an osmotic rise of 23 mm against pure acetone at 27°C.2 07 use > Solution = 23 mm against acetone a 3x09 oon of Hg 10x13.6 (since dg = 13.6 = 0.1522 em of Hg. _ 0.1522 76 =2x 10% atm. = 0,002 atm, a You have either reached 2 page thts unevalale fer vowing or reached your ievina tit for his book. a You have either reached 2 page thts unevalale fer vowing or reached your ievina tit for his book. a You have either reached 2 page thts unevalale fer vowing or reached your ievina tit for his book. Theory of dilute Solution 173 Weserose X1000 Modality = Myo _ 3421000 342x180 =0.555m Weacyose ¥1000 M grecose *Veohtion Ans. Molarity (M 34.2x1000x0.9 3422142 = 0.42 Ans. > Example 6.18 The density of 3 M solution of Na; is 1.25 gm mI. Calculate (a) mole fraction of Na,S,0, (b) I Molalities of Na” and $,0% ions in solution. > Solution Mol. wt of Na,S,0, = 158 Wt of solution = 1000 x 1.25 = 1250 gm. Wt of Na,S,0, = 158 x 3 = 474 gm. ‘Weight of water = 1250-474 = 776 gm. (a) - Mole fraction of Na,S,0, mole of Na,S,03 Mole of solution = 0.065 34+776/18 Mole of Na* Mass of water (b) Molality of Na* x 1000 Ans, 776 ns [Since Na,8,0, = 2Na"+$,0} ] __Mole of $,03 Mole of solution SX1000 3,865 Ans. 776 > Baampte 6-19 A complex is represented by COCI,.+ NH, Its 0.1 molal solution in water shows AT, = 0.558° K, for HO is 1.86 K molality Assuming 100% Ionisation Molality of S,07 of complex having coordination No. = 6. Calculate its formula, > Solution According to question C,, = 0 AT, = 0.558. K-= 1.86 mole 'K using the formula, AT, =i K,m. Where i = Vant’s Hoff factor => 0,558 = 1.86 X01 Xi 0.558 0.186 i= it means the complex ionises to form three ions i.e. total primary & secondary ligands are = 345 =Biex=5. COCI,.SNH, + [CO(NH,), Cl?" 2Cr- Initially 1 0 0 After ionisation 1 — a a 2a Leata+2o0 _ 1420 1 “oT Ife = 100% it means /=3 Hence the complex is [CO(NH,),CIC. [> Baampte 6.20 Cellobiose is sugar obtained by degration of cellulose, 1f200 ml of aq solution containing 1.5 gm cellobiose at 25°C gives rise to an osmotic pressure of 407.2 ‘mm Hg. What is the molecular mass of cellobiose.? > Solution Given x= sone. atm Vootaion = 200 ml. Weoyse = 1-5 gm. T= 298k. 1.5% 1004 ‘The molarity of solution = 1:5%1000 _ 200 using the formula m=CRT=MRT. Where M=Molarity. = 072. 75 0,082 «298 760M 160 x7.5x0.082 x 298 are 407.2 = 342.47 Ans. a You have either reached 2 page thts unevalale fer vowing or reached your ievina tit for his book. a You have either reached 2 page thts unevalale fer vowing or reached your ievina tit for his book. a You have either reached 2 page thts unevalale fer vowing or reached your ievina tit for his book. Theory of dilute Solution 177 42, 43. 44, 45. 46. 41. 48, (b) The lattice energy of barium sulphate is less than the hydration energy (c) The lattice energy has no role to play in solubility (@) The hydration energy of beryllium sulphate is less than its lattice energy ‘The solubility ofa solid ina liquid depends on : (a) Nature of solute (b) Nature of solvent (©) Temperature (@) Allofthese Solubility of deliquescent substances in water is generally (a) High (b) Low (©) Moderate (a) Cannot be said If AZ, is the elevation in boiling point for the electrolyte and AT," of non-electrolyte of the same concentration, then Van't Hoff’s factor i is equal to (a) AT,x AT, (b) Az," AT, @ “hah (@ arare AI molal solution of NaC solution in water freezes at a temperature of -3.72°C. The best explanation for this is (K; for water = 1.86) (a) incomplete dissociation of NaCl (b) Hydrogen bonding (©) increased pressure of the solvent (4) NaCImoleculesarecompletely dissociated Which solvent would show the largest depres- sion in freezing point when one mole of non- volatile solid is dissolved in it. (a) H,0.K,= 1.86 (b) comphor, K,= 40.0 (©) Naphthalene, Kp= 6.8 @) CiHKe= 5.2 ‘Sea water is converted into fresh water based upon the phenomenon of (a) plasmolysis (b) sedimentation (©) reverse osmosis (d) diffusion Solute A is a ternary electrolyte and solute is non electrolyte. If 0.1 M solution of solute B produces an osmotic pressure of 2 P, then 0.05 M solution of 4 at the same temperature will produced an osmotic pressure equal to (a) P (b) LSP (©) 2P (@ 3P 49. The molecular weight of NaCI determined by studying freezing point depression of its 0.5% aqueous solution is 30, The apparent degree of dissociation of NaCl is (a) 0.95 (b) 0.5 © 06 (@ 03 50. PureBenzene freezes at5.4°C. A solute of 0,223 g of phenyl acetic acid (C,H,CH,COOH) in 4.4 g of benzene (K,= 5.12 K kg mol ') freezes at 447°C. From this observation, one can con- clude that (a) phenyl acetic acid exists as such in ben- zene (b) phenyl acetic acid undergoes partial ion- ization in benzene (©) phenyl acetic acid undergoes complete ionization in benzene (d) phenyl acetic acid dimerizes in benzene I IIT WINDOW-III l > El 1. Two liquids 4 and B form an ideal solution. At 300 K the vapour pressure of a solution of 1 mole of A and.x moles of B is 550 mm. If the vapour pressures of pure 4 and B are 400 mm and 600 mm respectively, then x is @ 1 (b) 2 @3 @4 2. A sample of tooth paste weighing 500 g, on analysis, was found to contain 0.2 g of fluo- rine. The concentration of fluorine in ppm is (a) 410° (b) 4x1 (©) 4x10! (d) 2x1 3. 10% aqueous solution of certain substance is isotonic with 5% aqueous solution of fructose (CoH),0,).The molecular mass of the sub- stance is (a) 90 (v) 360 (©) 180 (d) 540 4. Osmotic pressure of 30% solution of glucose is 1.20 bar and that of 3.42% of solution of cane sugar is 2.5 bar. The osmotic pressure of a You have either reached 2 page thts unevalale fer vowing or reached your ievina tit for his book. a You have either reached 2 page thts unevalale fer vowing or reached your ievina tit for his book. a You have either reached 2 page thts unevalale fer vowing or reached your ievina tit for his book. Theory of dilute Solution 181 1 10. (©) Solvent molecules to the total number of molecules in solution (@) Solvent molecules to the total number of ions in solution ICBSE, 1991] 500g tooth paste sample has 0.2 g fluoride ion concentration. What is the concentration of fluoride in terms of ppm level? IAIIMS, 1991] (a) 250 (b) 200 (©) 400 (a) 1000 Molal depression constant depends upon [Pb CET, 1991] (a) Nature of the solute (b) Nature of the solvent (©) Vapour pressure of solution (4) Heat of solution .. Mole fraction of glycerine CH,(OH), in solu- ML. 12. 2B. tion containing 36 gm of water and 46 gm of glycerine is (a) 0.46 (ce) 0.20 (b) 0.40 (@) 0.36 [CPMT], 1991] ‘The molecular mass of sodium chloride as de- termined by osmotic pressure method is : [Haryana CET Sample Paper, 1992} (a) Equal to 58.5 (b) Greater than $8.5 (©) Less than 58.5 (4) None of the information is correct ‘The molarity of pure water is_ [KCET, 1993] (@) 55.5 (b) 50 (©) 100 (d) 18 Which of the following liquid pairs will show positive deviation from Raoult’s Law ? IMPCEE, 1993] (a) Water-hydrochloric acid (b) Water-nitric acid (©) Acetone-chloroform () Benzene-methanol ‘The law which states that the mass of a gas dissolved in given mass of the solvent at any temperature is directly proportional to the pressure of the gas above the solution is : IPMT, 1998] (a) Joule’s law (b) Boyle’s law (©) Henry’s law (d) Charle’s law 14, Increasing the temperature of an aqueous so- lution will cause TIFT, 1993] (a) Decrease in molarity (b) Decrease in molality (©) Decrease in mole fraction (a) Decrease in % (wiw) The vapour pressure of a solution 5g of non electrolyte in 100g of water at a particular temperature is 2985 Nm *. The vapour pres- sure of pure water at this temperature is 3000 Nm_*, The molecular mass of solute is (IIT, 1993] 15. (a) 180 (b) 90 (©) 270 (@) 200 The molal freezing point for water is 1.86° Cim. Therefore the freezing point of 0.1 m NaCl solution in water is expected to be IMLNR, 1994] 16. (a) - 186°C (b) -0.186°C (©) -0.372°C (@) +0.372°C 17. Van"t Hoff factor for 0.1 M ideal solution is IMLNR, 1994] (a) 01 (b) 1 (©) 0.01 (@) None of these 18. At 25°C the highest osmotic pressure is ex- pected by 0.1 M solution is [CBSE, 1994] (a) CaCl, (b) KCI (©) Glucose (@) Urea 19. Which of the following salts will have the same value of Van"t Hoff factor (i) as that of K, [Fe(CN),] ICBSE, 1994] (a) ALS (b) NaCl (c) AKNO,), (d) Na,SO, 20. 12 g of urea is dissolved in 1 litre of water and 68.4g of sucrose is also dissolved in 1 litre of water. The lowering in the vapour pressure of first case is [EAMCET, 1995] (a) equal to second (b) greater than second (©) less than second (@) double the second 21, Isotonic solutions have same [AFMC, 1995] (a) Molar concentration (b) Molality (©) Normality (d) None of these 22. When mango is placed in a aqueous solution of hydrochloric acid, it ICPMT, 1995] a You have either reached 2 page thts unevalale fer vowing or reached your ievina tit for his book. a You have either reached 2 page thts unevalale fer vowing or reached your ievina tit for his book. a You have either reached 2 page thts unevalale fer vowing or reached your ievina tit for his book. Theory of dilute Soluion 185 7. Mark out the incorrect combination of inter molecular forces, (a) KCl in H,0...... (b) CoH in CCl, (©) HClin CH,CN ion-dipole (@) HClin HO hydrogen bonding 8. Camphor CyoHy, which has a freezing point of 174°C has a freezing point depression con- stant at 40°C/m. Choose the correct statement regarding camphor. (a) Camphor has law freezing point depres- sion constant and could be used as a solvent for determination of molecular weight (b) Camphor has high freezing point depres- sion constant and could be used as solvent for determination of molecular weight. (©) High molecular weight solute is preferred with Camphor for precise determination. (@) Low molecular weight solute is preferred with Camphor for precise determination, 9, The total vapour pressure of a binary solvent is given by R = (I10X, + 125%,) mm Hg ‘Where x, and xp are the mole fractions of com- ponents 4 and B respectively. It suggests that (a) the vapour pressure of solution (1r, is less than the pure B component (b) the vapour pressure of solution (1 is more than that of pure component (©) Vapour pressure of pure A component is 110mm Hgand that of pure Bis 125mm Hg (4) The vapour pressure of pure 4 and B are 125 mm Hg and 110 mm Hg respectively. Consider 0.1 M solutions of two solutes X and ¥, The solute X behaves as univalent electro- lyte while the solute Y dimerises in solution. Which of the following statement (s) is / are correct regarding these solutions ? (a) The boiling point of the solution of X' will be higher than that of ¥. (b) The osmotic pressure of solution of will be lower than that of X. (©) The freezing point of the solution of X will be lower than that of ¥, (A) The relative lowering of vapour pressure of both the solution will be the same. ion-dipole dispersion force 10. 11. In the depression of freezing point experi- ‘ment, itis found that (a) The vapour pressure of the solution is less than that of pure solvent. (b) The vapour pressure of the solution is ‘more than that of pure solvent. (©) Only solute molecules solidify at the freezing point. (@) Only solvent molecules solidify at the freezing point, 12. If P® and P, are the vapour pressures of the solvent and its solution respectively and NV, and N, are the mole fractions of the solvent and solute respectively, then (a) P,=P'N, (b) P'-P, (©) P.=PN, (a) (P?= Pi PL= Ny (NAN) 13. 1 Mole benzene (P'yenyeqe = 42 mm) and 2 mole toulene (P°,jen¢ = 36 mm) will have : (a) total vapour pressure 38 mm (b) mole fraction of vapour of benzene above liquid mixture is 7/19 (©) positive deviation from Raoult’s law (a) negative deviation from Raoul’s law. 14, Which is / are correct statement (s) ? (a) When mixture is less volatile, there is positive deviation from Raoult’s law (b) When mixture is more volatile, there is negative deviation from Raoult’s law (©) When mixture is less volatile, there is negative deviation from Raoult’s law (d) When mixture is more volatile, there is positive deviation from Raoul’s law 15. The azeotropic solutions of two miscible liq- uids (a) can be separated by simple distillation (b) may show positive or negative deviation from Raoult’s law (6) are supersaturated solutions (@) behave like a single component and boil at constant temperature Ideal solution is formed when its components, (a) have zero heat of mixing (b) have zero volume change (c) obey Raoult’s law (a) can be converted into gases 16. a You have either reached 2 page thts unevalale fer vowing or reached your ievina tit for his book. a You have either reached 2 page thts unevalale fer vowing or reached your ievina tit for his book. a You have either reached 2 page thts unevalale fer vowing or reached your ievina tit for his book. Theory of dilute Solution 189 B. Vapour pres- 2. Inversely proportional sure 10 van't Hof factor (i) {in the same solvent) C. Freezing 3. Directly proportional to point ‘molecularmassor the sol- ute (inthe same solvent) D. Boiling 4._Directly proportional. to point molecular mass ofthe sole vent (for the same solute) I 1IT WINDOWAX ] > Em 1, A solution of sucrose (molar mass = 342) has been prepared by dissolving 68.4g of sucrose in one Kg of water. K; for water is 1.86 K kg mol and vapour pressure of water at 298 Kis 0.024 atm, 1. The vapour pressure of the solution at 298K will be (a) 0.230 atm (b) 0.233 atm (©) 0.236 atm (4) 0, 0239 atm 2. The osmotic pressure of the solution at 298 K will be (a) 4.29 atm (b) 4.49 atm (©) 4.69 atm (a) 4.89 atm 3. The freezing point of the solution will be (i) —0. 684°C (ii) ~ 0. 342°C (iii) —0.372°C fiv) ~0.186°C 2. According to Raoult’s law (which is applica- ble fora mixture of volatile liquids) the par- tial vapour pressure of a liquid is directly pro- portional to mol fraction of that component. Further assuming ideal behaviour for vapours and applying Dalton’s law we can write dif- ferent equation as follows Py PRXq, Py=PRXp Prog = Pat Pa = PRX A+ PR+(PR-PR)xA Further mol fraction of'a component in vapour phase is equal to the ratio of partial vapour pressure to total vapour pressure of mixture. The vapour pressure of two pure liquids 4 and B which form an ideal solution are 300 to 500 torr respectively at temperature T. A mixture of the vapour of A and B for which the mol fraction of 4 is 0.25 is slowly compressed at temperature T. The total pressure when first drop of conden- sate is formed will be. (a) 428 torr () 400 torr (©) 388 torr (d) 358 torr The pressure when only the last bubble of va- pour remains will be (a) 350 torr (b) 375 torr (©) 525 torr (@) 450 torr ‘The mol fraction of B in the last bubble of va- pour will be (a) 0.16 (b) 0.84 (©) 0.20 (@) 0.80 Properties such as boiling point, freezing point and vapour pressure of a pure solvent change when solute molecules are added to get homogenous solution. These are called colligative properties. Application of colliga- tive properties are very useful in day to day life, One of its example is the use of ethylene glycol and water mixtures as anti-freezing liq- uid in the radiator of automobiles. A solution M is prepared by mixing ethanol and water. The mole fraction of ethanol in the mixture is 0.9. in : Freezing point depression constant of water (K, water = 1. 86 K kg mol Freezing point depression constant of ethanol (K, ethanol) = 2. 0 K kg mol Boiling point elevation constant of water (K,“"") = 0. 52 K kg mol! Boiling point elevation constant of ethanol (Kg ethanol )= 1.2 K kg mol Standard freezing point of water = 273 K Standard freezing point of ethanol Standard boiling point of wate Standard boiling point of ethanol a You have either reached 2 page thts unevalale fer vowing or reached your ievina tit for his book. a You have either reached 2 page thts unevalale fer vowing or reached your ievina tit for his book. a You have either reached 2 page thts unevalale fer vowing or reached your ievina tit for his book. CHAPTER Chemical Kinetics Rate of Reaction ‘The rate of reaction means the speed with which the reaction takes place. This is expressed either in terms of decrease in the concentration ofa reactant per unit Alo AIA] _ =(C,-G)) action A — B. It is clear from the graph that the concentration of re- actant ‘A’ decreases and concentration of product “B? increases with the lapses of time. The rate of reaction is expressed in two ways: { ML .lGa=6 { lope of tangent at ine < a 2 at Bey slope of tangent g atime g 8 8% ttm 6a Tame Fig. 7.1 time or increase in the concentration ofa product per unit time, Or the rate of reaction is the change in the concentration of any one of the reactant or products per unit time, It is expressed by the graph for the re- 1. Average rate of reaction The rate when determined during a certain time in- terval, itis called average rate of reaction a You have either reached 2 page thts unevalale fer vowing or reached your ievina tit for his book. a You have either reached 2 page thts unevalale fer vowing or reached your ievina tit for his book. a You have either reached 2 page thts unevalale fer vowing or reached your ievina tit for his book. Chemical Kinetics 197 Pseudo First Order Reaction Consider the following acid-catalysed reactions: (i) Hydrolysis of ethylacetate. CH, C00 C,H, + H,0 “> cH, COOH +CH,OH (i) Inversion of cane-sugar 0 CypHz 04, H3O > CyHy 04+ CoH Sucrose Glucose Fructose. ‘The above reactions are bimolecular but experiment- ly it is found that order is one. Thus. Rate of (1) reaction o [CH, COO C,H.) and Rate of (II) reaction of [C, Hy, Oy} Thus the reactions in which molecularity is two but order is one because the other component of the re- actants are used as medium and large excess, called as Pseudo Unimolecular Reaction, (Examples of Tst order Reaction 1. Hydrolysis of esters CH, COOC, H, + H,0 “+ cH, COOH + C3 Hs OH. In this reaction acetic acid is one of the products and the amount of which can be found by titrating against standard NaOH solution, Since catalyst acid also neutralise NaOH. Let the data observed for this titration is as follows Time Vol. of NaOH used, , Vad, So V, a Volume of acid catalyst (No CH, COOH yet to formed) V, oc Volume of acid catalyst + Volume of CH, COOH formed. 2. Amount of acetic acid formed @ V,~ V,, Thusx FV, Again V_s: Amount of acid catalyst + Maximum, amount of CH, COOH formed Maximumamountaceticacid produced aV.,—V, 2 aaV-V, K,=2303 logy =—= Viv 2303, ak = [HSB bog. K VV 2, Inversion of Cane Sugar Cp Hyp Oy, + HO 4 C,H,30,+C, Hi Oy Sucrose Glucose "Fructose The hydrolysis of dextro-rotatory sucrose produce a mixture of glucose (dextrorotatory) and fructose (Laevorotatory). The rate is measured by measuring the change in the angle of rotation by a polarimeter. The change produced in rotatory power ime “f° gives a measure of x ie. the quantity of sucrose de- ‘composed in that time. The total change in the rota- tory power produced at the end of the reaction gives the measure of CO. the initial concentration of su- crose If rr; and r,, represent rotations at the start of reaction after time ‘t’ and at the end of reaction re- spectively, then o 1 ct 270" 90° 180° 3. Gaseous Reaction involving First order ‘The gaseous reaction rate is measured in terms of change in pressure a You have either reached 2 page thts unevalale fer vowing or reached your ievina tit for his book. a You have either reached 2 page thts unevalale fer vowing or reached your ievina tit for his book. a You have either reached 2 page thts unevalale fer vowing or reached your ievina tit for his book. Chemical Kinetics 201 atc and a4 =K,[B] eo) From equation (2) by integrating we have [4] = [4], sere) Substituting the value of [4] into (2) a [4], eM K; [BI a lo 4 a = K, [B]=K, [4], e. The left side expression is equal to d[B]e%*' /dt Thus a[B]eX* = K,[A], ar ‘Now on integrating the above equation with B=(and1=0, we get, [8] = [4], (e“Kit _ 6 (3) Similarly “ky mt Ke* [C1=llo [!. Kok, (6) Since for the consecutive reaction. [4] = [4] + (2) + (C] assuming [B]y = [C], = 0 (1 3 "] 0 Time —> Kalk, =10 a : 2 a Bi/ B Time —> KgIk =05 Fig. 7.8 a[B [B) is maximum if +— =0 [B] is dt = KTAT= Ky 1B] nx ABD KerB 4 Kye at 2 a Ke = Ke = (B) Reversible Reaction Let us suppose a reaction ASB. Both forward and backward reaction are Ist order, Let x mole/L of A changes into B then concentration B increases. A «4. B = Initially a b After time‘? a—x b+ Rate = K, (ax) ~K, (6+ x) [ Both occur simultaneously] At equilibrium, net rate is zero, Thus, K, (a~x,) = Ky(b +x,) where e represent Rate of reaction a =K,(a-x)-K, on solving we get Rate = (K, +3) (X,—X) Again K,.= 2 (©) Parallel Reactions or side Reactions A reaction which do not follow only one path but follow different path to give different products and a You have either reached 2 page thts unevalale fer vowing or reached your ievina tit for his book. a You have either reached 2 page thts unevalale fer vowing or reached your ievina tit for his book. a You have either reached 2 page thts unevalale fer vowing or reached your ievina tit for his book. Chemical Kinetics 205 After 30 min from start of the decomposition in a closed vessel the total pressure developed is found to be 250 mm of Hg and on complete decomposition the total pressure is 500 mm, Calculate rate constant of the reaction.? > Solution The given reaction 2N,05(8) — 4NOz(g) +02 (8) Initially Py 0 0 After time‘? P,—2r 4x x Ati=00 0 2P, if. Total pressure (P,) after time ‘1° = P,— 2x + ax +x + 3x And P= 2P,+ ; P, sr : (Py i (250-200) = 50/3 Now apply first order kineties x= 238 gg 1 Pi-2x [since concentration crpartial pressure] 2.303 200 - log i 200 100 .08 x 10> min > Example 7.12 ‘The activation energy of the reaction A + B —> Prod- ucts is 105.73 kJ/mole, At 40°C the products are formed at the rate of 1.33 mole L-' min '. What will be rate of formation of product at 80°C? > Solution For A+B — Product The rate law expression at different temp. are given as At 40°C, ryy = K; [4]! [BY At 80°C, rgg = K; [4}" [BP Ky = an() From Arthenius equation =f. ( 1 K, 23088 (77 K, spew (1 ) ag Ae ee ST = WS” 2303x8314 (313353 = og S2 =r = & ~amttog2 > r= Fy (100) = fe =1007% = (00 x 0.133 mole L"! min”! = 13.3 mole L'! min! P5 Bampre 7.73 The rate constant at 25°C for the reaction of NH} and OH” to form NH,OH (aq. NH,) is 4.0. 10'° M"" 'S '. Calculate the rate constant for proton transfer to NH,, lonisation constant for eq. NH, is 1.8 x 10°? > Solution ‘The reaction, NH} +oH —\> NH,OH d[NH,OH [NH,OH] _ [NH] [OH 1 a 4] [01] o Again, NH, +H,0 —> NH, OH. d NH,OH = K: [NEG IHO] 2) From equation (1) and (2) we get K, [NH] = K, [NH{] [OH] [Nj J[on™ RAK, eK Ky [NH] =4x 10" x 1.8 «10° 2x 10° NU; [or "(NE where Kb=(lonisation constant. ofag. NH, a You have either reached 2 page thts unevalale fer vowing or reached your ievina tit for his book. a You have either reached 2 page thts unevalale fer vowing or reached your ievina tit for his book. a You have either reached 2 page thts unevalale fer vowing or reached your ievina tit for his book. a You have either reached 2 page thts unevalale fer vowing or reached your ievina tit for his book. a You have either reached 2 page thts unevalale fer vowing or reached your ievina tit for his book. a You have either reached 2 page thts unevalale fer vowing or reached your ievina tit for his book. a You have either reached 2 page thts unevalale fer vowing or reached your ievina tit for his book. Chemical Kinetics 213 4. 6. 1. (CH) 0 CH, + Hy + CO Time (sec) 0 400 x ow Total pressure 312 468585. yp (mm) ‘The values of (x, y) missing above are: (a) x= 800, y= 936 (b) x= 620, y = 624 (©) x= 620,y=936 (4) x= 800, y = 624 For the reaction N, + 3H, —> 2NH,, the rate Ais) 2.x 10* Ms". Therefore the rate i a[Ny] . — TNA iis given as: a salves (a) 10*M sec! (b) 10'M sec! (c) 107M sec! (@) 104 sec! M! For the reaction N,(g) + 3H,(g) > 2NH,(g) under certain conditions of temperature and partial pressure of the reactants, the rate of formation of NH, is 0.001 kg h', The rate of conversion of H, under same conditions is: (a) 0.0015 kg h™ (b) 1.76«10kgh™ (c) 0.002 kgh' (d) 0.003 kgh' . The rate constant for a zero order reaction is 2X 10? mol L see '. If the concentration of the reactant after 25 sec is 0.5 M, the initial concentration must have been: (a) 05M (b) 1.25M (c) 12.5M (d) 10M Given that for a reaction of nth order, the inte- grated rate equation is: x-—! yt ves Cand C, tta-1l) Loe cet are the concentration of reactant at time ¢ and initially respectively. The f,,,ad f, are related as (3, is time required for C to become C,/4): @) ty = 4221+ 1] (©) fy4= hal2" = 1) ©) bg = hal" +) @ f= 4,2""'= 1) For a chemical reaction A —> Products, the rate of disappearance of A is given by: ac, KC, -—A = —14*~ At low C,, the reaction dt 1FKCy is of the ..... order with rate constant ..... (AS- sume K;, K, are lesser than 1) (a) L KK (b) LK, (©) I, K/K, (@) UW, Ky/K,+K, 8. The rate equation for an autocatalytic reac- tion; aC, A+R A R+Ris “TS = KOC The rate of disappearance of reactant A is maximum when: (@) Cy=2Cq (0) C=C (©) 2C,=Cy (@) Cy= (CQ) 9. At the point of intersection of the two curves shown, the cone. of B is given by ...... for, A> nB: Ao Cone Time @ > ©) — A+B —> +D+OW (a) Xis threshold energy level (b) Y and Z are energy of activation for for- ward and backward reaction respectively (©) Q is heat of reaction and reaction is exo- thermic (A) All of these Consider the reaction: No.) 2NHy,) The equally relationship between [NH dt u. + 3H > a You have either reached 2 page thts unevalale fer vowing or reached your ievina tit for his book. a You have either reached 2 page thts unevalale fer vowing or reached your ievina tit for his book. a You have either reached 2 page thts unevalale fer vowing or reached your ievina tit for his book. Chemical Kinetics 207 37. If for two reaction E, > E,, and TC, and ‘TC, are temperature coefficient respectively, then which altemative is correct: (a) TC, > TC, (b) TC, Product is rate = K [4]" [B]”. On doubling the concentra- tion of A and halving the concentration of B, the ratio of new rate to the earlier rate of reac- 39, tion will be: (a) nm () 2" 1 O ae (@) mtn 2 40. Ina first order reaction, the concentration of the reactant decreases form 0.8 M to 0.4 Min 15 minute. The time taken for the concentra- tion to change from 0.1 M to 0.025 M is, (a) 60 minute (b) 15 minute (©) 7.5 minute (4) 30 minute 41. ‘The rate equation for the reaction: 2A + BC is rate = K [4] [B]. The correct statement about this is: (a) Kis independent of [4] and [B] (b) fp is constant (c) Unit of K is sec! (a) Rate of formation of C is twice the rate of disappearance of A 42. The energy of activation for forward and backward change for an endothermic reac- tion; X > Y are E,and E, respectively. Which is correct: (a) Ey < Ee () E> Ey (c) E,=E, (@) No relation between them 43. A reaction involving two different reactants can never be: (a) Unimolecular reaction (b) Torder reaction (c) I order reaction (d) Bi-molecular reaction 44. 1,,, can be taken as the time taken for concen- tration of reactant to drop to 3/4 of its initial value. If the rate constant for a first order reac- tion is K, then #,,, can be written as: (a) 0.10. (b) 0.29/K (c) 0.69/K (d) 0.75/K 45. The following mechanism has been proposed for the reaction of NO with Br, to form NOBr; NO, = NOBK, + Bray NOBrip)+ NO, 2NOBrg) Ifthe second step is the rate determining step, the order of the reaction with respect to NOjg) is @ 1 (b) 0 (3 (@2 46. Rate of reaction can be expressed by Arhe- nius equation as; Keach" In this equation £, represents: (a) The energy above which all the colliding molecules will react (b) The energy below which colliding mol- ecules will not react (©) The total energy of the reactant molecules at a temperature T (d) The fraction of molecules with energy greater than the activation energy of the reaction 47, The rate constant, the energy of activation and the Arrhenius parameter of a chemical reac- a You have either reached 2 page thts unevalale fer vowing or reached your ievina tit for his book. a You have either reached 2 page thts unevalale fer vowing or reached your ievina tit for his book. a You have either reached 2 page thts unevalale fer vowing or reached your ievina tit for his book. Chemical Kinetics 221 EuRT EuRT 47. (b) Ae or K=4 48, (d) r= K [N,O.] | order as unit of K = sec fT, ¢ 2.40 x 10° = 3.0 x 10° [N,O,] 2.40 ,0;] = —" =0.8M INO51= 55 9.9 k= 22 og 2.303 800 eS = log 2x10" 50 = 1.38 x 104 sec"! 2303 0.1 . (o) K= I 0.0347 50, (6) K= = log = =0.03 rate = K x [X] = 0.0347 x 0.01 347x104 Mmin' I 1IT WINDOWAV l > 1. Which of the following is/are incorrect re- garding activation energy? (a) Large the activation energy, smaller is the value of rate constant, (b) The higher the £, higher the value of Ky +10 ji (©) At lower temperature, increase in temper- ature causes more change in the value of K than at higher temperature. () A plot made between K and 1/T gives temperature co-efficient, a straight lin of slope ——= 2. A+B C+D; AH =-217KJ/mole Mark out the incorrect statements, regarding the reaction: (a) The rate of disappearance of B increases on increase of the concentration of A. (b) The rate of formation of D increases on increasing temperature, (©) The rate of formation of C increases on increasing temperature, (A) The use of catalyst doesn’t affect the rate of formation of B or C 3. Ageneral reaction A(g)+B(g) > C(g) + DV) proceeds in a container of volume V at temper- ature T, which of the following is/are correct? -alA] at anB at | dnb V dt atc) a 4. AB +m > Am +B For the reaction the time required to complete a definite fraction is inversely proportional to the initial concentration, Which of the fol- lowing graphs express the reaction progress clearly? (a) (b) () @ Reaction progress —> @) ©) 4 \ Reaction progress > Reaction progress —> © @ 5. The acid-catalysed iodination of propanone may be investigated by reacting dilute aque- ous iodide with solutions containing known concentration of propanone and acid. Large excess of propanone is used deliberate- ly sometimes. (a) to buffer the acid concentration (b) to observe the effect of change in concen- tration of /, and H” Reaction progress —> a You have either reached 2 page thts unevalale fer vowing or reached your ievina tit for his book. a You have either reached 2 page thts unevalale fer vowing or reached your ievina tit for his book. a You have either reached 2 page thts unevalale fer vowing or reached your ievina tit for his book. 226 Numerical Chemistry 19. 20. a. 22, 23, 4, ment. The 10 mL of this mixture at STP pos- sess the rate of 10° disintegration per minute. How many milli-curie of radioactive carbon is needed to prepare 60 litre of such a mixture? A solution contains 1 milli-curie of L-phenyl alanine C'* (uniformly labeled) in 2.0 mL so- lution, The activity of labeled sample is given as 150 milli-curie/milli-mole. Calculate: (a) The concentration of sample in the solu- tion in mole/litre. (b) The activity of the solution in terms of counting per minute/mL. at a counting ef- ficiently of 80%. For the following sequential reaction, 4 —K_, B —*5 C, find out the concentration of C at time ¢= I day, given the K; = 1.8 «10° s and K, = 1.1 x 107s and initial molar con- centration of A is 1.8. A solution contains a mixture of isotopes of X™ (t= 14 days) and X? (t,,. = 25 days). Total activity is curie at f= 0. The activity re- duces by 50% in 20 days. Find: (a) The initial activities of X*" and X* (b) The ratio of their initial no. of nuclei, The rate of decomposition for methyl nitrite and ethyl nitrite can be given in terms of rate constant (in see ') K, and K, respectively. The energy of activations for the two reactions are 152.30 KJ mol! and 157.7 KJ mol as well as frequency factors are 10° and 10'* respectively for the decomposition of methyl and ethyl nitrite, Calculate the temperature at which rate constant will be same for the two reactions. A given sample of milk tums sour at room temperature (20°C) in 64 hours. In a refrigera- tor at 3°C, milk can be stored three times as long before it sours. Estimate (a) The activation energy for souring of milk, (b) How long it take milk to sour at 40°C? 22.4 litre lask contains 0.76 mm of ozone at 25°C, Calculate (i) the concentration of oxygen atoms needed. so that the reaction O +O, —+ 20, having rate constant equal to 1.5 *10" litre mol! sec’! can proceed with a rate of 0.15 mol litre" see? (ii) the rate of formation of oxygen under this condition. 25. Two Ist order reactions having same reactant concentration proceed at 25°C at the same rate, The temperature coefficient of the rate of the first reaction is 2 and that of second reae- tions is 3. Find the ratio of the rates of these reactions at 75°.C? Decomposition of H,0, is a first order reac- tion. A solution of HO, labelled as 20 vol- umes was left open due to this some HO, decomposed. To determine the new volume strength after 6 hours, 10 mL. of this solution was diluted to 100 mL 10 mL of this diluted solution was titrated against 25 mL of 0.025 M KMn0, acidified solution. Calculate the rate constant for decomposition of H,0,.” The progress of the reaction A = nB with time is presented in the figure. Determine: (i) the value of n (ii) the equilibrium constant, K and (iii) the initial rate of conversion of A 26. 21. oo : ¥ os 8 oo roe ee Tiethou 28. 4;U" is radioactive and it emits a and particles to form,, Pb™. Calculate the number of «and f particles emitted in this conversion. ‘An ore of 4,U is found to contain. y,U** and ,, Pb in the weight ratio of 1:0.1. The half-life period of y,U°™ is 4.5 <10” years. Caleulate the age of the ore.? a You have either reached 2 page thts unevalale fer vowing or reached your ievina tit for his book. a You have either reached 2 page thts unevalale fer vowing or reached your ievina tit for his book. a You have either reached 2 page thts unevalale fer vowing or reached your ievina tit for his book. Chemical Kinetics 231 (©) Three natural life time (d) Four natural life time 5. Number of natural life times (¢) required for a first — order reaction to achieve 99.9% level of completion is (@) 23 (©) 92 (Passage V The rate law for the decomposition of gas- N,O, is N,O, +2NO, + 40. A reaction (b) 6.9 (@) 0.105 mechanism which has been suggested N,0, == NO, + NO, (fast equilibrium) & NO, +NO,; —> NO, +NO +0, (slow) NO +NO, —> 2NO, (fast) 1. In 20 minutes of 80% of N,O, is decomposed find rate constant (a) 0.08 (b) 0.05 (©) 0.12 (a) 02 2. When graph is plotted between t,, Vs initial concentration then slop is (@) 0 (b) = ©@>o0 @ <0 3. At the equilibrium state NOs, NO, and NO, concentration are 0.2, 0.4 and 0.4 M respec- tively. Find K,? (@) Ou () 02 @1 (a2 If the concentration of N,O, is doubled then rate becomes (a) 2 times (b) 4 times (©) 8 times (@) constant remain same (C Passage VI ‘Chemical reaction occurs only as aresult of col- lisions between the reaction molecules. During this process atoms are rearranged by breaking up old bonds and new bonds are formed. 1. Rate of the reaction (r) B given by the expres- sion (a) r=fxz (yr-Ft z @raZ @rafxe 2. Fraction of the molecules (/) in a gas possess- ing energy greater than Ea(B) given by (a) f= (b) fae Rt (c) EaRT (d) Ea-RT 3. Rate constant K for the reaction is given by the expression (a) ae (b) K= £a-RT (c) K=Z* RT (8) K=f*RT 4. The temperature coefficient for most of the reaction is @1 (b) 15 ©2 @ 25 Passage VII ‘The rate of reaction is defined as the change of concentration of any one of its reactants oF products per unit time, In general, for a reac- tion aA + DBC + dD The instantaneous rate may be expressed as Ld{C}_ 1 d{D} =lad[A]_-1d(B} L ad ob dt edt d dt “) then x, y and z dt (@) Lt (0) 3,2,3 (©) 3,3,2 @) 2,2,3 2. The rate reaction that does not involve gases, is not dependent on: (a) Pressure (b) temperature (©) Concentration (4) Catalyst 3. For a hypothetical reaction 2A + 3C, the re- action rate ‘r’ is given by =1dtA} dt (a) r=—d[Aydt () a You have either reached 2 page thts unevalale fer vowing or reached your ievina tit for his book. a You have either reached 2 page thts unevalale fer vowing or reached your ievina tit for his book. a You have either reached 2 page thts unevalale fer vowing or reached your ievina tit for his book. Solid State 237 AY ¢ Ye . ey ‘ Y | Rectangular plane Diagonal plane Axis of four rain oral Geter, (a) (b) (c) < VN D a5 Axis of there fold Axis of two fold Centre of symmetry symmetry (four) ‘symmetry (six) (d) {e) oe Fig. 8.1 4, Elements of Symmetry The total number of planes, axes, and centre of sym- metry possessed by a crystal are termed as elements of symmetry. A cubic crystal possess a total of 23 elements of symmetry. Plane of Symmetry=(3 +6)=9[Fig8.1 (a)and(b)] Axes of Symmetry = (3 +4 + 6) = 13 [Fig 8.1 (c), (d), and (c)] Centre of Symmetry = 1 [Fig 8.1 (9) ‘The total number of symmetry element = 23 (C Crystal Lattice ‘The regular three dimensional arrangement of the points in a crystal is called space lattice. The space lattice is also called crystal lattice. The location of the points in the space lattice are called lattice point or lattice site, Unit Cell ‘The smallest but complete unit in the space lattice which when repeated over and over again in the three dimensions generates the crystal of the given sub- stance. (CTynes of Unit Cells Unit cell are basically of two types: These are primi- tive and non-primitive. Primitive Unit Cells A unit cell is called primitive unit cell if it has par- ticles (or points) only at the comers. It is also called simple unit cell a You have either reached 2 page thts unevalale fer vowing or reached your ievina tit for his book. a You have either reached 2 page thts unevalale fer vowing or reached your ievina tit for his book. a You have either reached 2 page thts unevalale fer vowing or reached your ievina tit for his book. a You have either reached 2 page thts unevalale fer vowing or reached your ievina tit for his book. a You have either reached 2 page thts unevalale fer vowing or reached your ievina tit for his book. a You have either reached 2 page thts unevalale fer vowing or reached your ievina tit for his book. a You have either reached 2 page thts unevalale fer vowing or reached your ievina tit for his book. Solid State 247 2 3 o 4 6, 5 (a) Fig. 8.21 Closed packed structure (a) Hexagonal closed packing (b) Cubical closed packing 5 then the sites marked 2, 4 and 6 are left unoccupied Let us denote the second layers of spheres by B. ‘The second layer again has two types of vacant sites. Around any atom in the second layer, one set of vacant sites lies just above the vacant sites 2, 4 and 6 of the first layer and the other set lies above the centres of the spheres of the first layer. Thus after the second layer is complete, there are two different ways of placing the spheres in the third layer. If the spheres in the third layer are placed in the vacant sites which are above the centres of the spheres of the first layer, then the third layer repeats the arrangement of the first layer and we have only two types of lay- ers, viz, AB AB....... ete. This is called a hexagonal closets packing. hep (fig 8.20 (a)). On the other hand if the spheres are placed in the vacant sites above 2, 4 and 6, a new arrangement (c) of the spheres in pro- duced, (fig 8.20 (b)). The entire arrangement is now of the type ABC ABC ...... etc, and is reffered to as a cubical closest packing cep ot face centered cubical closest packing, fee. ‘The number of nearest neighbours (Known as the co-ordination number) in each arrangement is twelve; six in the same layer three in the layer above and three in the layer below it. In the fee structure, there are two types of va- cant sites or holes: tetrahedral and octahedral. A tet- rahedral hole is surrounded by four spheres while an octahedral hole is the empty space surrounded by six spheres. These vacant sites can accommodate other Fig. 8.22 (a) Tetrahedral holes in an fee structure Fig. 8.23 (b) Octahedral holes in an fee structure smaller atoms or molecules giving rise to a variety of different structures In an Fee Structure, there are eight tetrahedral holes per unit cell as shown in fig (a). The number of octahedral holes in the unit cell of the fee structure is four as can be deduced from the following consid- eration, In fig (b) each (x) mark represent a vacant site and there are tewelve such vacant sites at the edges of unit cubic lattice. The vacancy at an edge is common to four unit cells and hence the number of such vacant sites per unit cell is 2 3. In addition to these vacancies there is one octahedral hole at the centre of the unit cell. Thus the total number of octa- hedral holes per unit cell is four. In fig. 8.24 Body centered cubical elosed-packed structure, Another less packed arrangement of sphere is the body centered cubic arrangement as shown fig. above in which each sphere has cight nearest neigh- ours; four in the same plane, two above and two a You have either reached 2 page thts unevalale fer vowing or reached your ievina tit for his book. a You have either reached 2 page thts unevalale fer vowing or reached your ievina tit for his book. a You have either reached 2 page thts unevalale fer vowing or reached your ievina tit for his book. 252 Numerical Chemistry Hence No. of atom yin a unit cell 1 1 =8xl46x 024 8 2 and No, of atoms yin the unit cell 216 303 ‘Thus formula of compound 1s Ys OXY > Baampte 8.6 Copper Crystallises in face centred cubic close packed arrangement having unit cell length of 361 pm, What is the radius of copper atom? IAIEEE, 2009] > Solution c KI Fig. 8.29 AC= av? (in fee.) In face diagonal, Ary = a2 wt 4 > Example 8 ‘The edge length of a face centred cubic cell of an ionic substance is 508 pm. If the radius of cation is 110 pm, calculate the radius of anion? IAIEEE, 2010] r > Solution <— Fig. 8.30 Its structure like NaCl Thus 2 reion + 2 Faron ~ edge length Edge Length Tania 2 = Teston 54 — 110 = 144 pm. F> Bampte 88 A metal crystallises into two cubic phases face cen- tred cubic and body centred cubic whose unit lengths are 3.5 and 3.0 A° respectively. Calculate the ratio of densities of fce. and b.c.c. IH1T, 1999] > Solution Density (d) = de ~ Abc 83 = 4%B.0x10°)* _ 59 2xB.5x10%) F> Braampte 89 In face centred crystal lattice edge length of the unit cell is 400 pm. Find the diameter of the great- est sphere which can be fitted into the interstial site without distertion of lattice THT, 2005] > Solution Fig. 8.31 In fcc. structure av? =4r a You have either reached 2 page thts unevalale fer vowing or reached your ievina tit for his book. a You have either reached 2 page thts unevalale fer vowing or reached your ievina tit for his book. 264 Numerical Chemistry Answer the following questions 1. The number of atoms in the HCP unit cell is, @4 (b) 6 © 12 @17 2. The volume of this HCP unit cell is @ 242 (b) 165 © 123 (a) 6473 3. ‘The empty space in this HCP unit (a) 74% (b) 47.6% (©) 32% (@) 26% (D) Subjective Numerical Integer Type 1, The density of mercury is 13.6 grv/ml. Caleu- late approximately the diameter of an atom of ‘mercury assuming that each atom is occupy- ing a cube of edge length equal to the diameter of the mercury atom. IIIT, 1983} 2. Sodium metal crystallises in body centred cu- bic lattice with cell edge ‘a’= 4.29 A. What is INIT, 1994] 3. A metallic element crystallises into a lattice containing a sequence of layer of ABABAB any packing of sphere leaves out void in the lattice. What percentage by volume of this IIIT, 1996] A unit cell of sodium chloride has four for- ‘mula units. The edge length of the unit cell is the radius of sodium atom? lattice in emply space? 4. 0.564 nm. What is the density of NaCl? IIT, 1997] $. Chromium metal erystallises with a body cen- tred cubic lattice. The length of the unit cell edge is found to be 287 pm. Calculate the atomic radius . What would be the density of HIT, 1997] 6. A metal crystallizes into two cubie phases face centred cubie (FCC) and BCC, Whose unit cell length are 3.5 and 3.0 and 3.0 A re- spectively. Calculate the ratio of densities of FCC. and BCC. [IIT 1999] The figure given below show the location of atoms in three crystallographic planes in F.C.G. lattice. Draw the unit cell for the cor- responding structure and identify these planes IIIT, 2000) chromium in gm/cm’, in your diagram. 38 OOO OOO 8. You are given marbles of diameter 10 mm. ‘They are to be placed such that their centres. are lying in a square bound by four lines each of length 40 mm. What will be the arrange- ment of marble in a plane so that maximum, no. of marbles can be placed inside the area? ‘Sketch the diagram and derive expression for the number of molecules per unit area? (LIT, 2003} 9. A compound AB has rock salt type structure. The formula wt. of AB is 6.023 yamu. And the closed A — B distance is 7° nm. Where 7 is an arbitrary number (a) Find the density of lattice (b) If density of lattice is found to be 20 kg m_* then predict the type of defect? (UT, 2004) 10. In f.c.c. crystal lattice, edge length is 400 pm. Find the diameter of greatest sphere which can be fit into interstitial void without distortion of lattice (LIT, 2005] 11. The edge length of unit cell of a metal hav- ing molecular weight 75 gm/mole is 5 A, which crystallizes in cubic lattice. If the den- sity is 2 gm ce then find the radius of metal atom (NA = 6 x 10”) Give the answer in pm. {LT, 2006) Silver (at, Mass- 108 gm mole ') has a density of 10.5 gv cm’, The number of Ag atoms on a surface of area 10"? m’ can be expressed in Scientific notation as y x 10°, The value of x is, (IT, 2010) 12. I ANSWER KEY - IIT WINDOW II l L@ 26 30 40 5@ 6.@) 726) 89.) 10.46) IL (b) 12.6) 13.) 14. (6) 15. (d) 16.(b) 17. (a) 18. (¢) 19. (a) 20. (d) 2.0) 2@ B@ WO 26 26.(d) 27.(b) 28. (a) 29. (d) 30, (¢) 31.) 32.0) 33.0) 34) 35.44) 36.(a) 37.(a) 38. (d) 39. (a) 40. (c) AL@) 420) 48.) 44.4) 45.0) 46.(a)47.(6) 48.06) 49.06) 50.4) CHAPTER Surface Chemistry Introduction The branch of chemistry which deals with the na- ture of surface and changes occuring on the surface is called Surface Chemistry. (Adsorption ‘The phenomenon of higher concentration of molecu- lar species (gases or liquids) on the surface of a solid than in the bulk is called adsorption eg, (i) If silica gel is placed in a vessel containing water vapour, the latter are adsorbed on the former. On the other hand, if anhydrous CaCl, is kept in place of silica gel, absorption take place as the water vapours are uniformly dis- tributed in CaCl, to form hydrated calcium chloride (CaCl,.2H,0) Ammonia gas placed in contact with charcoal gets adsorbed on the charcoal whereas am- monia gas placed in contact with water gets absorbed into water, giving NHOH solution of uniform concentration, Adsorbent and adsorbate ‘The solid substance on the surface of which adsorp- tion occur is known as adsorbent, The subst that get adsorbed on the solid surface due to inter- molecular attraction are called adsorbate. (gas or solute) Adsorbent and adsorbate Fig 9.1 Desorption The process of removal of an adsorbent from the sur- face on which it is adsorbed is called desorption. It is the reverse of adsorption and can be brought by heating or by reducing the pressure, The adsorbent may bea solid or a liquid and the adsorbate may be a gas or a solute in some solution. 268 Numerical Chemistry Difference between Adsorption and Absorption ‘Absorption 1. Itis the phenomenon in which the particles of gas or liquid get uniformly distributed throughout the body of the solid ‘Adsorption’ It is the phenomenon of higher concentration of particles of gas or liquid on the surface than in the bulk of the solid. 2. The concentration is the same throughout the material, There= fore, itis a bulk phenomenon, ‘The concentration on the surface of the adsorbent is different from than in the bulk. Therefore, itis a sur- face phenomenon, 3. Absorption occurs at uniform rate Adsorption is rapid in the begining and its rate slowly decreases. (Sorption When both absorption and adsorption occur together andarenotdistinguishable, thesubstancegetsuniformly distributed into the bulk of the solid but at the same time its concentration is higher at the surface than in the bulk, the phenomenon is called sorption eg. Dyes get adsorbed as well as absorbed in the cotton fibers. Positive and Negative Adsorption Positive adsorption When the concentration of ad- sorbate is more on the surface of adsorbent relative to its concentration in the bulk, it is called positive adsorption, Negative adsorption When the concentration of the adsorbate is less on the surface relative to its eoncen- tration in the bulk, it is called negative adsorption, Factors affective adsorption of gases by solids Almost all solids adsorb gases to some extent. The extent of adsorption of a gas on a solid surface de- pends on the following factors. L. Nature of the gas (or adsorbate) The adsorp- tion depends upon the nature of the gas adsorbed. ‘The physical adsorption is non-specific in nature and. therefore, every gas gets adsorbed on th surface of any solid to a lesser or a greater extent. However, under given condition of temperature and pressure, the easily liquifiable gases such as CO;, HCI, NH3, Cl, ete are adsorbed more than the permanent gases such as H,, Np and O The higher the critical temperature, the more easily a gas is liquified and hence more readily it will be adsorbed Gas__[80, Critical [430 temper- ature Amt ad- sorbed (in mi) NH, 406 CO, 304 co[N, [He 134 | 126 [33 380 | 180 | 48 Ease of liquification decrease from SO, to Hy. How- ever, the chemical adsorption is highly specific in nature and a gas gets adsorbed on the solid only if it forms chemical bonds. 2. Nature ofadsorbent The extent of adsorption of a gas depends upon the nature of adsorbent. 3. Specific area of the absorbent The specific area of the adsorbent is the surface area available adsorp- tion per gram of the adsorbent, The large the surface area of the solid, the greater would be its adsorbing capacity. Therefore, the porous and finely divided forms of adsorbents adsorb large quantities of adsor- bate. However the pores of the adsorbent should be large enough to allow the gas molecules to enter them. 4. Effect of pressure ‘The extent adsorption of a gas per unit mass of adsorbent depends upon the pressure of the gas. 5. Effect of temperature : Adsorption isobar Ad- somption is generally temperature dependent. Mostly adsorption processes are exothermic and therefore, extent of adsorption decreases with increasing tem- perature at constant pressure. However as expected for endothermic adsorption processes, adsorption in- crease with increase in temperature. 6. Activation of adsorbent Activation of adsorbent ‘means the increasing of the adsorbing power of the adsorbent. This can be done by the following methods (i) Metallic adsorbents are activated by mechani- cal rubbing or by subjecting it to some chemi- cal reactions. (ii) To increase the adsorbing power of adsorbent, they are sub-divided into smaller pieces. As a a You have either reached 2 page thts unevalale fer vowing or reached your ievina tit for his book. a You have either reached 2 page thts unevalale fer vowing or reached your ievina tit for his book. a You have either reached 2 page thts unevalale fer vowing or reached your ievina tit for his book. a You have either reached 2 page thts unevalale fer vowing or reached your ievina tit for his book. a You have either reached 2 page thts unevalale fer vowing or reached your ievina tit for his book. a You have either reached 2 page thts unevalale fer vowing or reached your ievina tit for his book. Surface Chemistry 277 Fig 9.8 (a) Fig9.8 (b) (ii) Electrodialysis ‘When the rate of dialysis can be increased by using electrodes immersed in water in called electrodialy- sis fig 9.8 (b) (iii) Ultra filtration The process of removing impurities by passing through graded filter paper (such as whatmann no- 41, 42) is called ultrafiteration (iv) Ultra centrifiguration The process in which colloid or solutions settled down on centrifugation while impurities remains down and separated out Properties of colloidal solution (A) Physical Properties (Nature— in nature. Colloids orsolutions are heterogeneous (i) Filterability — Passed through ordinary filter paper but filtered through whatmann no ~ 41,42 filter paper. (iii) Settlement — Colloid donot settle under grav- ity (iv) Particle sice— ‘The particle size in between 10A° to 2000A°. (%) Diffusibility — They have comparatively low diffusibility. Colloids have almost same vis- cosity to that of pure solvent. (vi) Viscosity — Colloids or solutions have almost same surface tension, (vii) Surface tension — (B) Electrical properties (i) Solutions particles are generally charged and their stability depend on magnitude of charge. They have either + ve or — ve charges. + vely charged colloids — Fe (OH), vely charged colloids smoke, blood, metals. The charge on sols or colloids arises due to fric- tion between phase & medium, due to electron cap- ture on cavity of electrons. RCOON: > RCOO™ + Na MAAVVAcO0- ww Hydrophobic ——-Hydhophile Proteins are made up of condensation of amino acid in which +ve and —ve charge developed on re- spective ions. @ ZN no Zino Zum a, a & wy Gy ‘coon ‘coon ‘coor (ii) Preferential ion adsorption theory According to this theory solution particles possess the tendency for adsorption of common ion present in solution and there by acquiring charges. Addition of KF drop-by-drop to dilute cold solu- tion of AgNO, given rise to +ve sol of Agl. Since Agl particles adsorb Ag” ions present in excess in solution. KI-+ AgNO, — Agl + KNO; Agl+Ag’.....AgliAg™ medium positive solution a You have either reached 2 page thts unevalale fer vowing or reached your ievina tit for his book. a You have either reached 2 page thts unevalale fer vowing or reached your ievina tit for his book. a You have either reached 2 page thts unevalale fer vowing or reached your ievina tit for his book. 286 Numerical Chemistry 44, 45. 46. 47, 48, 49. 50. 51 52, (b) Concentration at which micelle formation starts (©) Concentration of electrolyte added to de- sttoy the micelles (@) Concentration of micelles at room tem- perature In Brownian movement or motion the paths of the particles are’ (a) Linear (b) Zig-Zag (©) Uncertain (@) Curved Blood may be purified by (a) Dialysis (b) Electro osmosis (©) Coagulation (d) Filtration Gold number was given by: (a) Ostwald (b) Zsigmondy (©) William and Chang (d) Langmuir The blue colour of water in the sea is due to (a) absorption of other colours except blue by water molecules scattering of blue light by water molecules refraction of blue light by impurities in sea water (@) reflection of blue sky by sea water Which is a natural colloid? (a) Sodium chloride (b) Urea (©) Cane sugar (d) Blood Anacrosol is a colloidal system of (a) a liquid dispersed in a solid (b) a liquid dispersed in a gas (©) a gas dispersed in a liquid (G) a solid dispersed in a gas Which of the following statements is not true for a lyophobie solution (a) Itcan be easily solvated (b) It carries charges (©) The coagulation of this solution is irre- versible in nature (d) Itis less stable in a solvent Which of the following statements is not true for a lyophilie solution? (a) Itcan be easily solvated (b) It carries no charge (©) The coagulation of this solution is revers- ible in nature (d) Itis not very stable in a solvent Which of the following colloidal system rep- resents a gel? (b) ©) 58. 56. 57. 58. g 61. (a) Solid in liquid (b) Solid in gas (c) Liquid in solid (4) Liquid in gas Which of the following colloidal system rep- resents a solution? (a) Solid in liquid (b) Solid in gas (©) Liquid in solid (d) Liquid in gas Which of the following represents a multimo- lecular colloidal particles? (a) Sol of sulphur (b) Starch (c) Soap (d) Proteins Which of the following represents a maromo- lecular colloidal particles? (a) Sol of gold (b) Cellulose (c) Soap (d) Synthetic detergent Which of the following represents an associ- ated colloids? (a) Sol of gold (b) Starch (c) Proteins (d) Soaps Which of the following ions is most effective in the coagulation of a ferric hydroxide sol? (@) cr (b) Br (c) NO; (a) SO} A colloidal solution can be purified following the method of (a) dialysis (b) peptization (c) mechanical dispersion (@) oxidation Which of the following sols is negatively charged? (a) Arsenius sulphide (b) Aluminium hydroxide (c) Ferric hydroxide (d) Silver iodide in AgNO, solution Which of the following sols is positively charged? (a) Silver iodide in potassium iodide solution (b) Ferric hydroxide (c) Gold (d) Silver Which of the following cations will have maxi- mum flocculation value for the arsenious sul- phide sol? (a) Na” (b) Mg™* (c) Ca** (@ arr a You have either reached 2 page thts unevalale fer vowing or reached your ievina tit for his book. a You have either reached 2 page thts unevalale fer vowing or reached your ievina tit for his book. a You have either reached 2 page thts unevalale fer vowing or reached your ievina tit for his book. I IIT WINDOW-IV l bam Matching Type A » Column T Emulsifier P. Colloidal Q. Blectrolyte Oildag = R. Xerogel —S. Column 1 ‘Tyndal effect Pp Dialysis. Peptisation Coagulation Column t Physiosorption (Chemisorption Activated adsorption Desorption Column 1 ‘Tyndal effect Brownian movement Electrophoresis Hardy Sulze Rules R. s. Pp Q R s. Pp Column It Colloidal solution of graphite Detergent Cellophane Dextrix Column 11 Scattering of light Purification of colloidal sol. Addition of electrolyte Precipitation of colloi- dal sol Column 1 High heat of activation Multimotecular High temp is required Low temp is required Column I Avogadro’s number Sky is blue Coagulation of colloidal Charge on colloidal solution 5. 6. 7. B. pom> A Column t Gold sol Purific n of colloidal solu- tion As,S; Sol Zeta potential Column 1 ‘Suspension Colloid ‘True solution Saturated solution Column 1 Aerosol Foam Emulsion Gel Column 1 Surface tension decreases Viscosity Viscosity decreases Surface tension decreases avzon Surface Chemistry 291 Column It P. Bredig are method Q. —vely charged R._ultracentrfiguration S. Electro kinetic potential Double decomposits react Column IL Setted under gravity ‘Show Tyndal effect ultra filtration possible anon ‘Show Brownian move- ments ‘T. Diffuse rapidly Column 1 Chease Milk Whipped cream ‘Smoke Cloud Columa I Lyophilie sol Lyophilic sol Associated colloid Macromolecular colloid a You have either reached 2 page thts unevalale fer vowing or reached your ievina tit for his book. a You have either reached 2 page thts unevalale fer vowing or reached your ievina tit for his book. a You have either reached 2 page thts unevalale fer vowing or reached your ievina tit for his book. CHAPTER Chemical Equilibrium Chemical reactions are of different types, some (vi) Redox Reactions reactions occur very fast, some occur comparatively SnCl, + FeCl, > SnCl, + FeCl, slow, some reactions are reversible, some are KMnO, + FeSO, +H,SO,—>K,S0,+ MnSO, irreversible and so on. In this chapter we are going + Fe,(SO,);+H,0 to study mainly about reversible reactions. i,e, the (vii) Decomposition reaction in which all the reactions which proceed from both the directions, components are solid. Fe,0,(s) 4+ Fe,0, + FeO ‘Some Irreversible Reactions 6) Oo © (i) Reactions of Metal with Acid to Liberate Hea (“Some Reversible Reactions Ca+ HCI CaCh + Hot Zn + HNO, — Zn(NO,), + Hyt Chemical reaction which takes place in both the (ii) Reaction of amphotexic metal with alkali directions under similar conditions are called Al + NaOH —+ NaAIO, + Hot reversible reaction. These reactions never go on Zn + NaOH — Na,Zn0, +H, completion. If the reaction is gascous it may be Pb + NaOH — Na,PbO, + H;t reversible if occurs in a close container. (iii) Neutralisation reactions of strong acid and (i) Dissociation Reaction strong base CaCO(s) = Ca0(s) + COQ) NaOH + HCl — NaCl + H,0 (ii) Reaction of acid and base in which at least one H,S0, + Ca(OH), — CaSO, + 2H,0 component is weak (iv) Precipitation Reactions NaOH + CH, COOH == CH,COONa+ HO AgNO, (aq)+NaCl(aq)—AgCl | +NaNO,(aq) NH,OH + HCl NH,Cl + H,0. Pb(NO,),(aq) + KI(aq) + PbI, | + KNO,(aq) NH,OH + CH,COOH == CH,COONH, + (v) Combustion Reactions HO cH, +20, 4 co, +2H,0 (iii) Gaseous Reaction occurring in a closed 3 container CH = CH+ 5 0, +200, +H,0 a You have either reached 2 page thts unevalale fer vowing or reached your ievina tit for his book. a You have either reached 2 page thts unevalale fer vowing or reached your ievina tit for his book. a You have either reached 2 page thts unevalale fer vowing or reached your ievina tit for his book. 300 Numerical Chemistry (= Pron y atb-2r According to this principle, if any external factor such as temperature, pressure or concentration is imposed on a reversible chemical reaction which is at equilibrium, then the equilibrium shift in the direction in which the effect of the imposed factors in nullified or reduced to zero. eg, formation of NH, is represented as, 2.4K N(g) +3H(g) == —-2NH(g), AH (a) Effect of temperature ‘The formation of NH, is exothermic in nature, thus if temperature is increased backward reaction starts more rapidly and hence more and more ammonia dissociates into N, and H Hence low temperature is favourable for the formation of NH,. (b) Effect of pressure If all the components are gaseous, by increasing pressure, volume is reduced, it means the reaction proceed in the direction so that no of gaseous mole is less. It is possible when more and more ammonia is formed. Thus for the formation of NH,, favourable condition is high pressure (c) Effect of concentration By increasing concentration of N, and H,, more and ‘more ammonia is formed, and if the concentration of INH, is increased backward reaction starts. Favourable condition for formation of NH, ‘Temperature => 450°C (optimum temperature for Haber’s process) Pressure —+ 20-100 atm. Catalyst — Iron Promoter — Mo. Degree of dissociation from density measurements Degree of dissociation is defined as the fraction of one molecules dissociated, let us consider a reaction (Ajn == nA Initially e 0 Atequilibrium (la) ncor Total mole at equilibrium = o(1-a) + nc let ‘d’ be the observed vapour density at a particular temperature and *D” be the vapour density initially. Thus Da — iu) and da ay [C-a)+nca}-y From (1) and I or a= M = Mol. Mass initially m= observed mol. mass. where equilibrium Consider the physical equilibrium Solid == Liquid —= Vapour Effect of pressure on melting when ice melts its volume decreases and hence in this type of system increasing pressure will reduce the melting point of the system. So high pressure will favour the melting of ice. Effect of pressure on boiling point By increasing pressure on the equilibrium liquid change into vapour, vapour condense and hence lower the vapour pressure. Thus boiling point of the solution (oF liquid) increases on increasing pressure. a You have either reached 2 page thts unevalale fer vowing or reached your ievina tit for his book. a You have either reached 2 page thts unevalale fer vowing or reached your ievina tit for his book. a You have either reached 2 page thts unevalale fer vowing or reached your ievina tit for his book. Chemical Equilibrium 305 > Solution N, +0, == 2NO Initially a (100-a) 0 At equilibrium a-x (100 —a—x)2x Mole % of NO. at equilibrium oe ag a-x+100-a—x+2x =x=0.9 =3 Mole of N,=a-0.9 Mole of O, = 100 —a— 0.9 =99.. Mole of NO. = 1.8 x feo 1 2) sys c 20, a Ee) v Vv (sy? (a-0.9)(99.1-a) 0.2054 2.1x103= = a=T947 % Ans. A vessel at 100K. contains CO, with a pressure of 0.5 atm, some of the CO, is converted into CO an addition of graphite. Calculate the value of K, if total pressure at equilibrium is 0.8 atm, [Roorkee, 1993] > Solution The reaction given is CO,(g) + C(s) == 2CO(g) os 0 0 O5-p 0 2p According to question 0.5 —p + 2p=0.8 => P=0.3 atm, Plco, at eqim. = 0.5 - 0.3 = 0.2 atm Initially At equilibrium Peg ategim > Example 10.16 Solid ammonium carbamate dissociate as NH,COONH,(s) == 2NH,(g) + CO,(g) The total pressure at equilibrium is found to be 0.225 atm. Calculate Kp ? > Solution NH,COONH,(s) == 2NH,(g) + CO,(g) Initially 1 0 0 At Lr 2x x Equilibrium Given 2x+.x= 0.225 0225 _ 9.075 3 _ 0.150 Nits Qog95 Pt 0.075 0.225 * Kp = Pyar = (S3xo225) x 2288 0.225 0.225 0.225 1 = 1.6875 x 10% atm’, Ans, t > Example 10.17 7 gm. sample of IBr in 0,2 litre flask is heated to 227°C to decompose as 21Br(g) == 1,(g) + Bry (2). If partial pressure of Br,(g) is 3 atm, Calculate Ke and Ky, > Solution Bg) — 12) + Brie) Initially a 0 “ 207 Atequilibrium (0.0338-») Yo Total mole at equilibrium =0,0338—x+ 0.5x+0.5x = 0.0338 Py, = 255 ep, 0.0338, According to question, -°=*_p, =3aum (1) eas 0.0338 From Gas law, PV, = »RT Px 0.2 = 0.338 0,082 x 500 = 6,929 atm From (1) a You have either reached 2 page thts unevalale fer vowing or reached your ievina tit for his book. a You have either reached 2 page thts unevalale fer vowing or reached your ievina tit for his book. a You have either reached 2 page thts unevalale fer vowing or reached your ievina tit for his book. Chemical Equilibrium 311 of 2:1 initially. The total P at equilibrium is found to be 1 atm. If the moles of NOC! are y% of Cl, at equilibrium, Kp for the reaction is 26 1B 26 oy) 3 © 312 © 256 1B 9 3 @ 2. © 356 © 52 48. The equilibrium constant is 3.8 x 107 and P'' = 6 for the reaction CO, + 2H,0 —= HCO™, + H,O*. The ratio of [HCO;)[CO,] is (a) 0.38 (b) 3.8 (©) 38x10 (d) 13.4 49. XY, dissociates as XY(g) == XY(g) + Y(g). Intial pressure of XY, is 600 mm Hg. The total pressure at equilibrium is 800 mm Hg. Assuming volume of system to remain constant, the value of Kp is (a) 500 mm Hg, (b) 100 mm Hg, (©) 200 mm Hg. (@) 400 mm Hg. 50. Consider the reaction CaCO,(s) = Ca0(S) + €O,(g) in a closed container at equilibrium. What would be the effect of addition of Ca- CO, on the equilibrium pressure of CO, (@) Increase (b) Decrease (©) Remains unaffected (@) Data is not sufficient to predict I 1IT WINDOWSII l > IES 1. Chemical equilibrium can be seen (a) By the constancy of concentration tem- perature and pressure. (b) By constancy of colour for specific reac tion, (©) By constancy of molarity of each compo- nent. (@) By constancy of Normality of all the com- ponents. 2. If a: is degree of dissociation of N,O, at equilibrium. Which is/are correct? (a) No of mole of N,O, remain at equilibrium is C(I — a). Where C is initial concentra- tion. (b) cecan be given as (©) can be given as d-D DoD (a) No of mole of NO, formed at equilibrium is equal to 2Co, 3. What will be the effect of adding catalyst at constant temperature ? (a) The value of Keq remain constant (b) AH remain constant (©) Ky and K, increases to some extent (a) Keg will change 4, For the phase change reaction in H,O(0) = H,,0(g) the increases in pressure causes. (a) Formation of H,0 (I) (b) Formation of H,0 (e) (©) Increasing boiling point of H,0(0) (a) Decreasing boiling point of HOW) 5. When any component of a chemical reaction is added at equilibrium. (a) Keg, remain same. (b) Keg. changes (c) Partial pressure of components changes (@) Partial pressure of components remain same. i 6. Plot of graph between log K and — is a straight line as shown. T (a) Slope is ~Ea/2.303 R (b) OAis log A. (©) Frequency facter=antilog OA (a) Slop is Ea. wT a You have either reached 2 page thts unevalale fer vowing or reached your ievina tit for his book. a You have either reached 2 page thts unevalale fer vowing or reached your ievina tit for his book. a You have either reached 2 page thts unevalale fer vowing or reached your ievina tit for his book. 318 Numerical Chemistry volume IL and 2L respectively. If Ke for the container A is x. The Ke in the container B is (suppose other factors are the same) © % (a) 4x (a) x (© 2x Passage -IIT For the reaction aA + BB == IL + mM. ‘The equilibrium constant has the following characteristics (i) Ke changes by changing stoichiometric coefficient (ii) Ke is inversed when reaction is reversed. (iii) Ke remain unchanged if any component is added at equilibrium. 7. If Ke for the reaction H, +1, == 2HI is 4. The equilibrium constant for the reaction HI 4 H,+ this a a 1 (a) 4 (by) — a + > (d) 16, ©} oy 8 IfNH, is added at equilibrium in the reaction NH,COONH,(s) == 2NH\(g) + CO,(g) The equilibrium constant is given by (a) PP (b) 2P (c) 4P* (@ 2P° ‘Where P is partial pressure of CO, at equilib- rium, 9. By addition NH, at equilibrium in the reaction NH,COONH (8) == 2NH,(g) + CO(g) ‘Then Partial pressure 1 NH,COONH, changes 2. NH, changes 3 CO, changes 4 NH, and CO, both changes Passage -IV Equilibrium is the state of a chemical reaction which is seen by the constancy of concentra- tion temperature, pressure and sometimes co- Jour. 1t is dynamic in nature and attain from either end of a chemical reaction. At equilib- rium the concentration of each and energy component is fixed and non zero, 10.5 mole H, and 3 mole 1; reacts to form HI by H, +I, == 2HI, the component which react completely is (a) Hy () HI 11 For the reation 2NO(g) + 0, === 2NO,(e) the equilibrium is reached at which (a) Brown colour is completely changes to colourless (b) Brown colour intensity increases and reached a maximum point. (©) Brown colour intensity remain constant but not maximum, (A) Can't say. 12 If equal mole of N, and H, reacts in a closed container to form NH, by a reaction N3 + 3H == 2NH,, At equilibrium (a) Concentration of N, and Hy are equal (b) Concentration of N, and NH,are equal (©) Concentration of Hy and NH,are equal (d) Concentration of any two components ‘may not same. (b) 1, (@) neither H, nor I; Passage -V For the reaction 2NO(g) = N.(g) + O.(8) AH =~-180 KJ mol '. 13. Which of the following facts does not hold good. (a) The Pressure changes at constant temperature do not effect the equilibrium constant. (b) The volume changes at constant tempera- ture don’t effect the equilibrium constant. (©) Dissociation of NO is favoured more at high temperature. (d) Dissociation of NO high temperature. 14 Keofthe reaction when2 mole NO. dissociated tothe extent of $0% at equilibrium is given by rt at 4 8 34 is favoured less at a You have either reached 2 page thts unevalale fer vowing or reached your ievina tit for his book. a You have either reached 2 page thts unevalale fer vowing or reached your ievina tit for his book. a You have either reached 2 page thts unevalale fer vowing or reached your ievina tit for his book. Chemical Equilibrium 325 bam Ranker's Choice 1, For the equilibrium, Sn0,(s) + 2H(2) = 2HO, + Sn(s) at 600K, the equilibrium steam-hydrogen mixture was 45% H, by volume and at 700 K, the equilibrium steam- hydrogen mixture was 24% H, by volume. Calculate the bond enthalpy of © ~ H bond. Given AH, [SnO,s] = ~ 65.8 keal/mol, Ey 104.2 keal/mol and Ey.» 190.2 keal/mol. 2. An equilibrium mixture of 2 moles each of PCI, PCI, and Cl, is maintained in a vessel of volume *V’ litre at temperature *T” K and a to- tal pressure of 3 atm, Cl, is now added maii taining the same pressure and temperature till the volume is doubled. Determine the number of moles of Cl, added. For the reaction , CaCO(s) + CaO(s) + CO,() K, = 1.16 atm at 800°C. If 20.0 g of CaCO, was put into a 10.0L container and heated to 800°C, what percent of the CaCO, would remain unreacted at equilibrium ? 4. 73.8 g of SrCO, was placed in a 10 litre con- tainer at 127°C. After some time, excess of graphite was added in the container, when equilibrium, C(s) + CO,(g) = 2CO(g) is then established. The K,, for StCO\(s) = SrO(8) + CO,(g) is 2 atm at 127°C and the total pressure of gases in the container is 3 atm at 127°C, calculate K,, for the equilibrium, ls) + CO,(2) = 2CO (g) 5, NO and Br, at initial partial pressures of 98.4 and 41.3 torr respectively, were allowed to re- act at 300K. At equilibrium, the total pressure was 110.5 torr. Calculate the value of equilib- rium constant (K,) and the standard free en- ergy change at 300 K for the reaction, 2NO(g) + Br,(g) = 2NOBr(g). 6. Ammonium hydrogen Sulphide dissociates as, follows, NH,HS(s) = H,S(g) + NH(g) If solid NH,HS js placed in an evacuated flask at a certain temperature, it will dissociate unit the total gas pressure is 500 torr. (a) Calculate the value of the equilibrium constant for the dissociation reaction. (b) Additional NH, is 7, 8. introduced into the equilibrium mixture with- out change in temperature until the partial pressure of ammonia is 700 torr. What is the partial pressure of H,S under these conditions ? Whaat is the total pressure in the flask ? Solid ammonium hydrogen Sulphide dissoci- ates as follows, NH,HS(s) = NH,(g) +H,S(e) If solid NH,HS is placed in an evacuated 5 litre vessel at 27°C, it will dissociate until the total gas pressure is 500 torr. (a) Calculate the value of the equilibrium constant for the dissociation reaction at 2. (b) Calculate the minimum weight of solid NHLHS required to establish the equilib- rium, (©) Additional NH, is introduced into the equilibrium mixture without change in temperature until the partial pressure of NH, at new equilibrium becomes 700 tom ‘What is the total pressure in the flask ? Also calculate the minimum weight of NH,HS. required to establish equilibrium under this condition ? When gaseous NO and NO, are mixed in a vessel of V’ litre capacity, following equilibria are readily attained at temperature TK. 2NO,(g) = N,0,(8) ; Ky, ' NO(g) + NOAg) = N0,(8) Inan experiment when NO and NO, are mixed in the ratio of I : 2, the final total pressure was 5.05 atm and the partial pressure of N,O, was 1.7 atm. Calculate (a) The equilibrium partial pressure of NO; and (b) K, for the second equilibrium. A saturated solution of iodine in water contains 0.330g L/L. More than this can dissolve ina KI solution because of the following equilibrium, I,(aq) + Taq) = 1; (eq) A0.1 M KI solution actually dissolves 12.5¢. of iodine/L, most of which is coverted to T;. Assuming that the concentration of I, in all saturated solution is the same, calculate the |, = 6.8 atm a You have either reached 2 page thts unevalale fer vowing or reached your ievina tit for his book. a You have either reached 2 page thts unevalale fer vowing or reached your ievina tit for his book. a You have either reached 2 page thts unevalale fer vowing or reached your ievina tit for his book. 380 Numerical Chemistry L NH, +H" Ho; +H* [Fe(H,0),OH""+H" bases = [Fe(H,0),P* ‘This concept generates a word conjugate ie. the acid when looses a proton, the species formed is called conjugate base and if a base accept a proton the species formed is called conjugate acid, The strength of conjugate species is just opposite to that of either base or acid. Acids 2" Conjugate Base HCl ——> Cl +H* NH, ——— NH,+ H™ HCO; —— CO} +H* BH, ——> BH;+H* C,H,OH > CH,0-+ H* Similarly +H Base H”, Conjugate Acid wu, 8 nny uco; +4 u,co, co? HS co; +H . cH, ———> CHE Hcl + HO > HO + Cr Acid Base IT Acid I Base T t t $ Conjugate acid-base pair-I Conjugate acid-base pair-IT + If acid or bases are strong its corresponding conjugate base or acids respectively are weak and vice-versa. Lewis Concept (Electron Pair Donor and Acceptor System) This concept is said to be modern concept to distin- guish acid and base. According to this concept the species which are electron deficient called as Lewis acid and the species which are electron rich are called as Lewis base. Lewis Acid (a) Neutral species: etc. (b) Ions: Na’, Mg”, AI", K’, ete. BF,, AICI, BCl,, ZnCl, FeCl, Lewis Base (a) Neutral species: PCI, PBr, ete. (b)Ions: Cl, F Br ,F O° ,N* ete. Strength of Acids An acid is said to be stronger ifit release H” ion eas- ily. Hence stronger the bond strength, weaker is the acid and bigger the bond length, stronger is the acid Levelling Effect and Levelling Solvent The apparent strength of an acid is dependent on the solvent in which the particular acid is dissolved. ‘Thus all the acids such as H,SO,, HCl, HCIO, when dissolved in H,O producing H,O™ ion and thus appears to be equally stronger. Thus water behave here as a levelling solvent. The solvent in which an acid is 100% ionised is called levelling solvent. But in most cases glacial acetic acid solution behave as levelling solvent. Solvent are generally different types such as, (a) Protic solvent > H,0, CH,COOH. (b) Aprotic solvent —> C,H,, Toluene, Xylene, NH. (©) Amphiprotic 1,0. Hydracids CH, < NH, < H,0< HF, This trend is supported by the stability of their conjugate bases. CH, a= [Kby, Again, ia [OH ]=Ca= cy = VCR And, PO = Jog, [OH ] = —log,(C-Kb)* =p! log Kb ~ } log 2 pote Fok 4 tog ¢ (Ciiyarolysis of Salt The soluble salt ionises to produce cation and anion and they intract with H” and OH” ions furnished by H,O and hence P" of solution changes and this phenomenon is called “hydrolysis of water”. Different types of hydrolysis are as follow: Hydrolysis of Salt of Strong Acid and Weak Base Salt such as ZnCl, FeCl, SnCl,, PbCl,, ZnSO,, Fe,(SO,);, FeSO,, Zn(NO,),, ete. behave as salt of strong acid and weak base and hence when dissolved in water, its P'' is less than 7 i.c. acidic in nature. Let ‘BA’ is a salt of this category and its hydroly- sis is represented as, BY +A +H,0 = BOH + HA (sveak) (strong) or, BY +H,0 = BOH +H (aq) (Cationic Hydrolysis) Initially c ° o Ateqim. c(h) Ch Gh (where h is degree of hydrolysis) According to law of Mass action, _ (BOHI(H'] _ Ch-Ch _ Ch’ Kh [By] Ci-h) I-h If1—b=1,Kh=Ch’ > h= JRE BOH is a weak base here and hence, wt snd» HOH Thus we get a conclusion on the basis of above expressions, ie a You have either reached 2 page thts unevalale fer vowing or reached your ievina tit for his book. a You have either reached 2 page thts unevalale fer vowing or reached your ievina tit for his book. a You have either reached 2 page thts unevalale fer vowing or reached your ievina tit for his book. lonic Equilibrium 339 ' Colour pie Indicators 7 range acidic NS Basic tral 1. Methyl 31-44 Pink — Pink Yellow orange 2. Methyl 4.262 Pink Yellow Yellow red 3. Phenol — 8.2-10.0 Colour- Colour- Pink phthalein less less 4, Thymol’ 9.2-10.2 Colou- — Pink phthalein less, Acidic Indicator Let us consider an acidic indicator (HIn) which attain equilibrium as Hin = H+ In (Colourles) (Pink) (H"}Un") [HIn} If the solution is acidic, the H’ ion concentration increases and since Kin is constant, it means equilib- rium shift towards left ie solution remain colourless, when some alkali is added, OH” combine with H” and form H,O, i.e equilibrium shift towards right ie pink colour start depeening. That is why Hin appers colourless in acid medium and pink in basic. {In} [Hin] =Kin Pl = PKIn + log. Basic Indicators When Methyl Orange (MeOH) is dissolved in HO, it attain equilibrium as follows: MeOH = Me" + OH Yellow red colourless If solution is acidic, H’ interact with OH and hence equilibrium shift towards right and red colour appears. But in alkaline medium OH” ion which is common ion shift the equilibrium to the left and hence yellow colour appears. In general P" range of indicator lies between PKin'" to PKin’! Case I, If P! = PKin | je UA * Thin 10 = 10% + % Ionisation of indicator would be Un} 0.10[HIn} iter yeatmy * 1° > Octo + [Hin] x 100% wl. =9.1% = 7p 100= 9.1%, In fact, P" = PKin! is the maximum P" up to which the solution has a distinct colour characteris- tics of Hin Case II, If PY {In7] © THIn] % lonisation of indicator is {n7] Kin” 10 10[HIn} Un 1 _ x 100% = Treaty = (Hin) * 100% [in ]+[HIn] 10[HIn] + (Hin) 1000 91%, il Acid Base Titration Curve Acid-base titration are generally four types in which different strength of acid and bases are used, such as, (a) Strong Acid Vs Strong Base Titration ‘When base i.e. OH” ion is added to acid solution P* goes on increasing. At the point of equivalence, H™ ion rapidly reduced and hence P" increases sharply. ‘Thus all the indicators are used which P''-range lies between 4.2 to 10.2. Vol. of alkali added —> a You have either reached 2 page thts unevalale fer vowing or reached your ievina tit for his book. a You have either reached 2 page thts unevalale fer vowing or reached your ievina tit for his book. a You have either reached 2 page thts unevalale fer vowing or reached your ievina tit for his book. 34 Numerical Chemistry > Example 17.11 Calculate [H"] ina solution containing 0.1 MHCOOH and 0.1M HOCN Ka for HCOOH and HOCN are 1.8 10 and 3.3 x 10%, > Solution Both the acids contribute for [H"] due to appreciable dissociation, HCOOH = H*+HCOO. O.1-x xt+y x HOCN = H’+OCN Ol-y xty y rs acon = HcoO" | =18x10 (HCOOH) Kya = SHE 18x10 Similarl: my (H*)ocN-) HON = (HOCN) CA 5 3 c107 Q OL From (1) and (2) we get ZL yg Or = 1.83x From (1) we get (x +1.83x)x = 1.8 x 104 => x= 2.52107 Thus y=4.61x 10° And (W)=x+4=252% 10° +4.61x 10° 1.13 x 10°M. P> Brample 17.12 How much AgBr could dissolve in 1.0L of 0.4 M NH,? Assume that [Ag(NH,),]" is the only complex formed the dissociation constant for Ag(NH,)$ = Ag’ + 2NH; 6.0 x 10° x10 K sisociaion K,, (Ag Br) > Solution Let us suppose solubility of AgBr be x M. Thus [Br ] 1m but [Ag”] #Xm. This is because Ag” react with ‘NH to form [Ag(NH,)3]” Solet [Ag’]=yM. AgBr = Ag (ag) + Br (ag) => K,, = [Ag' [Br = yx =5.0 x 10 Again Ag +2NH, > [Ag(NH;)2]" x 04 0 04-2r x [Ag(NH,),|" == Ag’ +2NH, i 0.4—2e 4 y 04-204 2p =04 re P y.4-2x+2y) Thus, Ky, LAS#IINH SE _ 204-202) [Ag(NH5)2] » =6.0x 10% If x Kain 18 low Kuna + => 1.15 x 10°M 5 Baampte 1113 A sample of AgCI was treated with 5.0ml of 1.5M Na,CO, solution to give Ag,CO,, The remaining so- lution contained 0,0026gm of CI- per litre Calculate the solubility product of AgCI [K,,, (Ag COs) 1 =8.2x 10" [HIT, 1997] > Solution [Na,CO,] = [CO,” ] = 1.5M At equilibrium 0.026 (Cl}=[Nacl] = (cl-[NaCl] = =0,0000732M. AgCI(s) +Na,CO, = Ag,CO,(s) #2NaCl [1.5 — 0.000036] 0.0000732, 15M M [Ag+] =2.34 x 10M. K, of AgCl = [Ag'}ICI] 2.34 x 10x 0,0000732 =171 x 10". a You have either reached 2 page thts unevalale fer vowing or reached your ievina tit for his book. a You have either reached 2 page thts unevalale fer vowing or reached your ievina tit for his book. a You have either reached 2 page thts unevalale fer vowing or reached your ievina tit for his book. 38 _Numerical Chemistry 22. Which one of the following order of acid strength is correct? (a) RCOOH > ROH > HOH > HC=CH (b) RCOOH > HOH > ROH > HC=CH (©) RCOOH > HOH > HC = CH > ROH (@) RCOOH > HC = CH > HOH > ROH At 90°C pure water has H,O" = 10° mol litre ', The value of K,, at 90°C is (@) 10% (b) 10? @) 10 (d) 10% 24. Aweak acid (HA) after treatment with 12ml of 0.1M strong base (BOH) has a PY of 5. At the cend point the volume of same base required is 26,6mil. Thus K, of acid is (a) 28x 10° (b) 82x 10° (©) 28x 107 (@) 8.2.x 10° 25. The ratio of dissociation constant of two weak acids HA and HB is 4, At what molar concentration ratio, the two acid will have 23, same P"? (a) 2 (b) 0.5 ©4 (6) 0.25 26. The P! of a 10! MHCI solution is approximately (a) 10 (b) 7 1 @ 14 27. The solubility product of A;X; is. 1.08 x10 Its solubility in pure water will be (a) 1.0 10°M (b) 1.0 x10°M (c) 1.0x10°M (d@) 1.0 10°M 28. The molar solubility (in mol L) of a sparing- ly soluble salt MX, is ‘s’. The corresponding solubility product is K,,. ‘s" is given in terms. of K,, by the relation ys =(Kse\!"* = Gz) @s (f) Cas-(2 (©) 8(256 Kp)" (d) 8 =(128 Kgp)"* 29. The dissociation constants of two weak acids are K, and K,. The relative strength of the two acids is given by (@) KK, (b) (K,K,)'* (©) (KyK,)°? (@) KK, 30, An aqueous solution of Borax (NasB,07) is (a) Acidic (b) Basie 3. 32. 33. 35. 36. 37. 38. (c) Neutral (d) Cannot be predicted pH of 0.01 M(NH,),SO, and 0.02 M NH,OH buffer (pKa of NH,” = 9.26) is (a) 4.74 + log 2 (b) 4.74 -log 2 (c) 4.74 +log 1 (d) 9.26 —log 1 0.1 mole of CH|NH,(K, = 5 x 10%) is mixed with 0.08 mole of HCI and diluted to one litre. What will be the H” concentration in the solu- tion? (a) 8x 107M (b) 8x 10M (©) 1.6 10"''M (a) 8x 10°M Abuffer solution contains | mole of (NH,),SO, and 1 mole of NH,OH(K, = 10°), The P* of solution will be @ 5 (b) 9 (©) 5.3010 (d) 8.6990 The increasing order of basic strength of CI ,CO3, CH,COO, OH, Fis (a) Cl H,0 > NH, > OH (c) NH} < HF 7 CH,COOH +50 ml of 0.1M NaOH B. 20mlof0.1 gq. PHET MNH,OH + 50 ml of 0.1 M CH,COOH C. S0mlof 01M —r — Phenolphtha- NaOH=50 Jein is suitable ml of 0.1M indicator CH,COOH D. 20mlof0.1M —s, MeOH is si ‘NH,O11+20 mi able indicator of 0.1 MH,SO, I 1IT WINDOW-V l bam Asserition and Reasoning. 2: Assertion — HCI is not acidic in benzene Reason — Benzene does not accept proton Assertion —H,O” is the strongest acid in aque- ous solution. Reason — Water levels the strength of hydro- nium ion . Assertion — CIO, is the weakest base Reason ~ In ClO,, chlorine atom is —SP* hy- bridised 4. Assertion bases Reason ~ A species that accepts a proton nec- essarily should donate a lone pair of electrons 5. Assertion ~ HINO, is not a Bronsted acid in CHCI, Reason — CHC, is an example aprotic solvent 6. Assertion ~ Ethoxide ion acts as a strong base Reason — Ethyl alcohol is a weak acid 7. Assertion — H,P0, is a Lowry Bronsted acid & base All Bronsted bases are Lewis Reason ~ HPO,” is a proton donor as well as proton acceptor 8. Assertion — P" of an aqueous solution of acetic acid remains unchanged on the addition of sodium acetate Reason — Dissociation of acetic acid is suppressed by the addition of sodium acetate due to common ion effect. 9. Assertion — At 90°C the P" of pure water is less than 7. Reason — Ionic product of water increases with increase in temperature. 10, Assertion — The aqueous CH,COONa is alkaline in nature Reason — Acetate ion undergoes anionic hydrolysis. II. Assertion ~ Aqueous solution of ZnSO, is neutral Reason ~ Salt of strong acid and strong base doesn't undergo hydrolysis. solution of I IT WINDOW-VI l Pam Previous Year asked Questions 1. How many moles of Sodium propionate should be added to 1L of an aqueous solution containing 0,020 moles of propionic acid to obtain a buffer solution of P" 4.75? What will be P" of 0.010 moles of HCI are dissolved in the above butler solution. Compare the last ph value with the P" of 0.010 M HCI solution. Dissociation constant of propionic acid Ka at 25°C is 1.34 x10. [HT JEE,(1981)| 2. 20ML of 0.2M sodium hydroxide is added to 50m! of 0.2 M acetic acid to give 70 ml a You have either reached 2 page thts unevalale fer vowing or reached your ievina tit for his book. a You have either reached 2 page thts unevalale fer vowing or reached your ievina tit for his book. a You have either reached 2 page thts unevalale fer vowing or reached your ievina tit for his book. lonie Equilibrium 357 NH,OH. The solution contain C; mole/litre of NH," ions. The dissociation constant of NH,OH is given é (©) Ky= —* © Ky oe The total [NH,] which is determined by volu- metric analysis is @¢, 0) C+ +C, © Cre, @ G+C, . The degree of dissociation of NH,OH is given by @c ) ) % Passage -II Buffer solution resist the change in P" against addition of small quantity of acid or base from outside. Acidic buffer is prepared by mixing weak acid and its corresponding salt with strong acid. A 0.2 M HCOOH solution having [H = 6.4 x10™’mole/litre, and in it HCOONa solution is added to adjust the [CH,COONa] .. The buffer solution P"' is given by (salt) (Acid) (salt) (Acid) (Salt) (Base) Bi (a) P= PKa + log (b) P= PKb + log (c) P= PKb+ log (@) P= PKb + log (Salt) . The ionisation constant of HCOOH is equal to (@) 1.210% (b) 1.2104 (©) 2.110% @ 12x10 ‘The P" of buffer is (@) 438 (b) 5.09 (©) 3.21 (d) 3.99 Passage -IV The indicator theory explain that all the indicator can’t be used in any type of acid- base titration Methyl orange in an acidic indication while phenol phthalein is a basic one, The graph of acid base are as follow: @ 4 (0.2 le 42 1 Vali added —> o 38 le ss 1 Valli added —> © 1 Lf 6 32 1 Valkali added —> a You have either reached 2 page thts unevalale fer vowing or reached your ievina tit for his book. a You have either reached 2 page thts unevalale fer vowing or reached your ievina tit for his book. a You have either reached 2 page thts unevalale fer vowing or reached your ievina tit for his book. Chemical Energetics 363 (k) Heat Energy, work doneanditssign The energy exchanged between a system and surrounding, when their temperature are different is termed as heat energy. When a system transfer heat energy to the surrounding, the temperature of system decrease, while that of surrounding increases. This transfer of heat energy continue till the temperature of both system and surrounding are the same. Sign Conversion Heat Energy Absorbed by the Evolved from system Work done ‘On the system By the W=4ve System Wee Work is the product of force and distance. Thus work is said to be done if the point of application of force moves through a certain distance. Ifa system looses energy, then the work is said to have been done by the system. On the other hand if the system gains energy from the surrounding, then the work is said to have been done on the system. ¢ th Law of Thermodynamics Itisalso known as law of thermal equilibrium. This law tes that the two systems “A” and “B” are in thermal equilibrium with a third system *C” separately, then “C's also in thermal equilibrium with each other. ‘Thermal equilibrium a Thermal Thermal ‘equilibrium ‘equilibrium c (Cirst Law of Thermodynamics It is nothing but the law of conservation of energy to the thermodynamic system. This law states that “the total energy of an isolated system remain con- stant’ or It is impossible to construct a perpetual motion ma- chine that can produce work without taking energy from outside or Whenever a particular type of energy disappear, an equivalent amount of another type must be produced. Mathematical Expression of Ist Law Let a system take ‘g’ amount of heat energy from the surrounding and due to which the volume of gas enclosed ina cylindrical system changes from *V,’to Vy’ whereas its internal energy change is AE. Thus AE =4-PW;-¥,) > AE =q-w Where ‘w” is work done by the system. Case I For a cyclic process involving isothermal process, then, AE = 0. ‘Then gw i.e. total amount of heat energy taken is converted into work. Case I For an adiabatic process q = 0, ie. AE=—w i.e. the system without taking energy from outside do- ing some work. a You have either reached 2 page thts unevalale fer vowing or reached your ievina tit for his book. a You have either reached 2 page thts unevalale fer vowing or reached your ievina tit for his book. a You have either reached 2 page thts unevalale fer vowing or reached your ievina tit for his book. 374 Numerical Chemistry > Solution Istlaw: AU=q+ w= 40-8 = 32 Joule. > Example 12.2 2.8 gm of N, gas at 300 K and 20 atm was allowed to expand isothermally against a constant external pres- sure of 1 atm. Calculate, AU, g and WV for the gas. > Solution n= 78 91, 28 20x V, = 0.1 x 0.0821 x 300 > V, = 0.123 litre, Finally, 1x V, = 0.1 x 0.0821 x 300 = V, = 2.463 litre, reversible, Poo Vi) 1(2.463 - 0.123) =— 2.340 lit. atm, =2.340 x 1.987 x 4.184 0.082 236.95 J Joule. Since AU = q + w’ & AU for isothermal process is zero 0 =g~236.95 = 4g = 236.95 Joule. > Example 12. Calculate the work done when a system raises a col- umn of water of radius 5.0 mm through 10 em, > Solution —Mgh & Mass =v xd =3.14x(0.57x 10 x1 3.14 x (0.5)° x10 1x980x10/2 —3.85 x 10? erg = — 3.85 x 10° Joule. (The centre of mass of column lies half way along its length ‘7’, > Example 12 Calculate the work done when 50 g of iron reacts with HCI in (i) a closed vessel of fixed volume (i) an open beaker at 25°C, > Solution (i) Vessel is of fixed volume, AV =0, (ii) The gas drives back the atm. hence, W == Poy AV, AV=MRTIP ex wy arg nat Pox Fe(s) + 2HCI(aq) > FeCh(aq) + H,(g) n = 20,3929 mole 56 Ww ==0,8929 x 8.314 x 298 J 2212.25 [> Beampre 125 Calculate g, w, AU and AH for the isothermal revers- ible expansion of one mole of an ideal gas from an initial pressure of 1.0 bar to a final pressure of 0.1 bar at a constant temperature of 273 K? [Roorkee, 2000] > Solution For isothermal reversible expansion, Ww =~ 2.303 nRT log 2 ?, 2303 «1 x 8314 «273 log 5227.2. At constant 7, for expansion, A7=0, <. AU And Au =nC)AT > & q =-w=+5227.21 Also when temp is constant, P|, = PV, or PY = Constant. AH = Au+ A(PV) ; AH =0. > Bampre 12.6 Calculate WV and AE for the conversion of 1 mole of HO into | mole of steam at a temp. of 100°C and a pressure of | atm. Latent heat of vaporisation of H,O is 9720 cal/mole. > Solution P = 1 atm=76 x 13.6 x 981 dyne/em? .013 x 10° dyne/em’, V;, =vol. of | mole of H,O at 100°C = 18 ml ‘ol. of 1 mole of steam at 100 a You have either reached 2 page thts unevalale fer vowing or reached your ievina tit for his book. a You have either reached 2 page thts unevalale fer vowing or reached your ievina tit for his book. a You have either reached 2 page thts unevalale fer vowing or reached your ievina tit for his book. Chemical Energetics 379 > Example 12.24 The standard heat of formation of CCL(g); H,O(g) CO,(g) and HCI(g) are ~ 25.5, -57.8, - 94.1, and—22.1 Keal respectively. Calculate AH,y. for the reaction, CCl,(g) + 2H,0(g) > CO,(g) + 4HCIG). > Solution Al fesction ~ AH Products ~ AH Reactant = (AHo, +4 Alia) ~ (AHGq, + 2AH},0) 41.4 kcal “ > Example 12.25 Determine enthalpy change for CyHg(g) + H3(g) > CH¢(g) + CH,(g) at 25°C using heat of combustion value under standard conditions. H, CH, CoH, Cemptite) — 285.8 — 890.0 — 1560 — 393.5 kJ/mole. AH, CHa) =~ 103.8 kJ/mole. (UIT, 92) > Solution H+ 30 31,0 Ai) CH, +20, + CO, + 2H,0 Ai) GH.+ fos 2CO, +3H,0 (iii) jg) + 2 > COy (iv) 3Cig) + 4H, > C3Hy(g) wh}: Operation + —v— iii ii +3 x v4 S xi AH =~55.7 Kl. I IIT WINDOW—II 1 bo Multiple Choice Questions (Ciuc) THERMO CHEMISTRY 1, Equal values of CH, and H, are combusted under identical condition. The ratio of heat evolved in i.e. 6. 8. H,(g) + 20,(g) = H,O(g) AH = 241.8 kT 1 CHB) +25 Ox(8) = 2C0,{) + H,0(8) AH =~ 1300 kd (a) 5.37/1 (b) 1/537 ou (d) None of these Heat of neutralisation of a strong acid in- creased by 5° when 5 ml of a strong base is added to it. If 10 ml of each are mixed, tem- perature should increased by (a) -274kcal/eq —(b) 13.7 kcal/mol (©) - 13.7 keal/eq —_ (d) ~ 13.7 keal/mol The temperature of a 5 ml of strong acid in- creased by 5° when 5 ml of a strong base is added to it, If 10 ml of each are mixed tem- perature should increased by (a) 5° (b) 10° (©) 15° (@) Can not be known The heat of formation of HCI(g) from the re- action F,(g) + Cl(g) = 2HCI(g) : AH =~ 44 keal (a) +44 keal (b) — 44 kal (©) +22 keal (a) ~22 keal Given N,(g) + 2H,(g) = 2NH,(g) AH? = ~22 keal. The standard enthalpy of formation of NH, gas is (a) 11 kcal/mol (b) 11 kcal/mol (©) -22 kealimol ——_(d) 22 kcal/mol If for H(g) = 2H(g) : AH = 104 keal, heat of atomization of hydrogen is (a) 52 keal (b) 104 keal (©) 208 keal (a) None of these Heat of combustion of CHy, C,H, CyH, are ~ 890, -1411, and —1560 kl/mol respectively. Which has the lowest fuel value in ki/g (a) CH, (b) GH, (©) CH, (@) All same Heat of neutralisation of a strong acid and a strong base is equal to AH of (a) H’ +OH-=H,0 (b) HJO+H" = H,0 (©) 211, + 0, =2H,0 (@) CH,COOH + NaOH = CH,COONa +H,0 a You have either reached 2 page thts unevalale fer vowing or reached your ievina tit for his book. a You have either reached 2 page thts unevalale fer vowing or reached your ievina tit for his book. a You have either reached 2 page thts unevalale fer vowing or reached your ievina tit for his book. 386 Numerical Chemistry (©) AH=Oand AS <0 (@) AH <0 and AS<0 99. All the natural process in this universe pro- duce (a) a decrease in entropy of the universe (b) an increase in entropy of the universe (©) no change in entropy (@) some times increase or sometimes de- crease in entropy 100. Ifthe intemal energy of an ideal gas decreases by the same amount as the work done by the system, the process is (a) cyclic (b) isothermal (©) adiabatic (@) isolated 101. One mole of a perfect gas expands isothermal- ly to ten times its original volume. The change in entropy is (a) O1R (b) 2.303 (c) 100R (d) 100.0R 102. In which of the following case AH and AE both are zero. (@) Adiabatic (b) Isobaric (©) Isocalorie (@) Cyclic 103. One applying pressure to the equilibrium, ice = water ‘Which phenomenon will happen (a) More ice will be formed (b) More water will be formed (c) Equilibrium will not be disturbed (4) Water will evaporate I {IT WINDOWS l em Multiple Choice Questions CMe 1, Ina process a system does 140 J of work on the surrounding and only 40 J of heat is added to the system, hence change in internal energy is (@) 1803 (b) 46.92 cal (©) ~ 23.92 cal () — 1005 2, In an isothermal expansion of an ideal gas (@) AU=0 (b) ar=0 (© 9=0 @ w=-4 3. 2 mole of an ideal gas (monoatomic) undergo a reversible process for which PY’= constant. ‘The gas is expanded from initial volume of 1 L to final volume of 3 L starting from initial temperature of 300 K. Find AH for the process (a) -600R (b) ~ 3000 (c) —1000R (d) —2kcal mole"k" Which of the following reaction have same heat of reaction at constant temperature and at constant volume. (a) Hyg) + Cl(g) = 2HCI (g) (b) 2NO(g) = N,(g) + O,(g) (©) Nag) + 3H,(g) = 2NH,(2) (d) Cls) + Og) = CO,(2) 5. For the gaseous reaction; if AH is the change in enthalpy and AU is the internal energy then, (a) Aff is always greater than AU (b) A/1 is always less than AU (©) AH < AU only if the no. of mole of the products is less than that of the reactant (@) AU < AM only if the no. of mole of the reactant is less than that of the product, #30) |, 6. IfAG=AH—TAS and AG = AH+T (“2 4. then variation of emf of a cell £, with temp. 7 is given by AH AG-AH @) ) nF a AS AS AS a) AS © oF © OF 7. Ifthe inversion temperature of a gas is 80°C, then it will produce cooling under Joule- Thomson effect at (a) 298K (b 273K (c) 193K (@) 173K 8. Sign of AG for the melting of ice is negative at (a) 265K (b) 270K (©) 277K (@) 274K 9. Ifa gas absorbs 200 J of heat and expands by 500 cmagainst a constant pressure of 2 x 10° Nm * then change in internal energy is (a) 23.92 cal (b) — 100) (c) +1003 (a) +3005 a You have either reached 2 page thts unevalale fer vowing or reached your ievina tit for his book. a You have either reached 2 page thts unevalale fer vowing or reached your ievina tit for his book. a You have either reached 2 page thts unevalale fer vowing or reached your ievina tit for his book. 394 Numerical Chemistry 17. 18. 19. 20, C(s) > Clg); AH = 172 keal H, — 2H; AH = 104 keal H+ 30s HO (0); AH = ~ 68.0 kcal C(s) +0, > CO,; Ad Heat of combustion of C 94.0 keal Hg = ~ 372.0 kcal Heat of combustion of CH, =~ 530.0 kcal. Estimate the average S — F bond enthalpy in SF,. The standard heats of formation of SF, (g), S(g) and F(g) are: — 1100, 275 and 80 kJ mol‘ respectively. The enthalpy change for the following reac- tions at 25°C are given below 1 1 gih @ + 502 (8) > OH): AH =~ 10.06 keal Ai) H, (g) > 2H(g); AH = 104.18 keal (ii) ©; (g) > 20(g); AH = 118.32 keal (iii) Calculate OH bond energy for hydroxyl radical ‘The enthalpy change involved in the oxida- tion of glucose is ~ 2880 kJ mol !. Twenty five percent of this energy is available for muscu- lar work. If 100 kJ of muscular work is needed to walk one kilometer, what is the maximum distance that a person will be able to walk af- ter eating 120 g of glucose? At300K, the standard enthalpies of formation of C,HsCOOH (s), CO, (g) and H;O (/) are — 408, — 393 and — 286 kJ mol ' respectively. Calculate the heat of combustion of benzoic acid at (i) Constant pressure and (ii) Constant volume. (R= 8.31 Jmol! K'') I (IT WINDOW-VIIE l Passage -I The dissociation reaction of NO, (dimmer) is given as follows. N,0,(g) > 2NO,(g) The Gibb’s free energy at 298 K and | atmo- spheric pressure is plotted with fraction of N,O, dissociation as follows. 72345578 10 1, Calculate AG? when one mole of NO, change into equilibrium mixture with NO,? (a) —0.84 KI (b) 0.84 KI (©) 624k) (@) - 5.4K). 2. When two mole N,O, is dissociated into NO, which is correct? (a) AH =-ve, AS = +ve (b) AH = +ve, AS= +ve (c) AH = -~ve, AS=—ve (d) AH =+ve, AS=~ve. 3. As the NO, start dissociation, the graphs shows that first AG decreases, and then in- creases. This is due to (a) The decrease is due to first association and then dissociation. (b) The decrease shows that allotropic change first and then dissociated. (©) The first decrease because the process un- dergo spontaneous and then non-sponta- neous. (d) None of these. Passage - IT ‘The enthalpy change when one gm-equivalent of an acid is neutralised with one gm-equiva- lent of a base, is called enthalpy of neutralisa- tion. NaOH(aq) + HCK(aq) > NaCl(aq) + H,0. AH =~ 13.7 keal =-57.1K. 4, Calculate the enthalpy of neutralisation for H,SO,(aq) + Ca(OH), -> CaSO,(aq) + 2H,0. AH =—x kJ, (a) -xkI (b) -2xkI (©) -x2kI (@) ackd 5. The enthalpy of neutralisation for the reac~ tion a You have either reached 2 page thts unevalale fer vowing or reached your ievina tit for his book. a You have either reached 2 page thts unevalale fer vowing or reached your ievina tit for his book. a You have either reached 2 page thts unevalale fer vowing or reached your ievina tit for his book. CHAPTER Electrochemistry Electrochemistry is the branch of chemistry which deals with the study of relationship between elect cal energy and chemical energy or the inter conver- sion of one form into another. (flectrolytes ‘The substance which is either in fused state, or in aque- ‘ous state produce, different ions on passing electricity is called electrolytes. Electrolytes are two types: Strong Electrolytes ‘The electrolytes which is completely ionised in fused or aqueous state is called strong electrolytes e.g. NaCl, NaOH, KOH, HCl, H,SO,, HINO, ete. ‘Weak Electrolytes ‘The electrolytes which ionises partially or freely in aqueous or fused state are called as weak electrolytes €q. CH,0OH, NH,OH, PbSO,, BaSO,, ZnS ete. > CHlectrotysis Itis the process in which different ions are discharged at the different electrodes (cathode and anode) by passage of electric current through the electrolytes, When electricity is passed though the electrolytic solution it ionises into cation and anions which accumulate at different electrodes either by loosing or gaining electrons. Cathode The electrode at which reduction occur is called cathode. It is -vely charged during electrolysis. Anode The electrode at which oxidation occurs is called anode, It is +vely charged during electrolysis. For example NaCl fused is electrolysed as follows NaCl (fused) — Na’ + Cl At cathode (— ve) Na’ ion is discharged. Na‘ +e Na(s) Atanode (+ve) CI ion is discharged crscite 2Cl > Cl, (g). a You have either reached 2 page thts unevalale fer vowing or reached your ievina tit for his book. a You have either reached 2 page thts unevalale fer vowing or reached your ievina tit for his book. a You have either reached 2 page thts unevalale fer vowing or reached your ievina tit for his book. K x 1000 va (where M = Molarity) em em’ oh Unit of molar conductivity, is mole In SI unit it is expressed in § cm? mole! Equivalent Conductivity Equivalent conductivity of a solution at a dilution is defined as the conductance of all the ions produced from one gram equivalent of the electrolyte dissolved Iml of a solution, Mathematically, it is denoted by et Neg= KV. where V is the Vol. of solution containing 1gm equivalent of the solute in em* where N= Normality of solution unit of equivalent conductance = ohm | em? or mho em’ (g.eq) =S om’ (g.eq) '. Effect of Concentration on Conductivity * Molar conductivity (A,,) ofthe strong electrolyte increases to a small extent with dilution due to decrease in interionic attractions with dilution. * Molar conductivity of the weak electrolyte also increases with dilution as the degree of dissocia- tion increases on dilution. ‘* Equivalent conductivity of the electrolyte (strong or weak) also increases with dilution and reaches to @ maximum or limiting value which is, termed A... * The conductivity ofall electrolyte increases with temperature. * The conductivity slighty varies with pressure due to change in viscosity of the medium. + The viscosity of dilute solution decreases with inereasing pressure. Eleclrochemishy 403 Ni,SO, Equivalent conductance GH,COOH Concentration —> Fig 13.1 Kohlrausch’s Law This law states that “at infinite dilution, when the dissociation is complete, each ion makes a definite contribution towards molar conductivity of the electrolyte irrespective of the nature of the other ion present” ie. the equivalent conductivity of the clectrolyte at infinite dilution is the sum of the equivalent conductories of the cation and anions i.e. he= Ny +2, where 2, and 2, are the i and anion at infinite dilution respectively. Again ion- ic conduction ean be mathematically expressed as a= kU.and 2,=kU, where U, and U, are the ionic mobilities of anion and cation respectively at infinite dilution and k is a constant equal to 96,500 coulomb. onic conductance of cati Ionic velocity ie, Tonic Mobility = + sential gradient Tonic velocity ~ Potential difference (volt) Mathematical expressions Equivalent conductance (A.,) can be expressed as, 1 1 ARAM ope he where Z' and Z_ are charges on cation and anion a You have either reached 2 page thts unevalale fer vowing or reached your ievina tit for his book. a You have either reached 2 page thts unevalale fer vowing or reached your ievina tit for his book. a You have either reached 2 page thts unevalale fer vowing or reached your ievina tit for his book. 408 Numerical Chemistry [Products] [Reactants] where AG° represent free energy change for the reaction when various reactants and products are at standard state. The free energy change is given by AG = AG? + 2.308 RT logy AG =-nFE,.y. and AG? = - nFE®.y. Substituting the value of AG and AG® in (1) apo [Products] FE. =~ FE +2.303 RT logyg on ot tl se [Reactants] 2303 RT [Products] Egg Bg SOBRE jog, Productsl Ba nF” [Reactants] © . 2.303RT => Beat = Been logioQ [Products] where Q= called as reaction quotient [Reactants] Equation (2) is called Nernst equation if R= 8.314 JK! mole’! and T = 298 K, F = 96,500 C. Equation (2) written as 2.303 x 8.314 x 298 [Products] 2 og — log 96500 F [Reactants] » 0.0591, [Products] > By=E 2 8) ae iReactantsy Electrode Potential for Half Cell Let us consider Daniel cell i.e. Zn(s) + Cu" (aq) —9 Zn” (aq) + Cu (8). Oxidation half reaction The half reactions are written as Zn > Zn** +20 If we apply Nernst equation for this reaction a Enyindt ~ Bbyigtr~ 29574 jog 2 é {Zn} Since (Zn) = unity Ent Eg OE giz] 8) Similarly Reduction half reaction Cu’+ + 2e° > Culs) 0.0591 cy en [Cu] ecu] c= EG Since [Cu] = unity Then, Eesticu= Eroica + ot From equation (4) & (5) Boa = Exnant” * Bovticw log(Cu”’]...(5) = Baya + Brea) 29 tog ZI o 0.0591, [Zn7* 2 Ran Be ee oO Standard Cell Potential and Equilibrium Constant ‘The standard free energy change for a reaction is re~ lated to the standard cell potential by the equation, AG® = —n FE{, AG° = -RT In K Thus “OF E'y =-RT In K. or go Eppa BEL ek nF nF 2S0SRT _ 9.0591 v at 25°C. Thus B%g = 295? h hog K at 25°C a nS, log K = BES “ eek 0.0591 = K =antilog "Ee 0.0591 Heat Energy, Free Energy and Work done ‘The work done by the system or on the surrounding during the chemical change is given by AG =~ nFE,. ‘The work done (IV) is the product of charge and potential Wonax = charge x potential a You have either reached 2 page thts unevalale fer vowing or reached your ievina tit for his book. a You have either reached 2 page thts unevalale fer vowing or reached your ievina tit for his book. a You have either reached 2 page thts unevalale fer vowing or reached your ievina tit for his book. a You have either reached 2 page thts unevalale fer vowing or reached your ievina tit for his book. a You have either reached 2 page thts unevalale fer vowing or reached your ievina tit for his book. a You have either reached 2 page thts unevalale fer vowing or reached your ievina tit for his book. a You have either reached 2 page thts unevalale fer vowing or reached your ievina tit for his book. Electrochemistry 419 (a) 0.05 mole of Cu” ions passed into solution (b) 0.112 litre of Cl, was liberated (c) 0.56 litre O, was liberated (d) 0.1 mole of Cu ions passed into the solution 9. The cell reaction for the given cell is sponta neous if Pt(H,)|H" (1M)||H" (1M) |(H,)Pt, (s) Py (b) P, Y > X, then (a) Y will oxidise X and not Z (b) ¥ will oxidise Z and not X (©) Y will oxidise both X and Z (A) Y will reduce both X and Z 19. The standard reduction potential values of three metallic cations, X, Y and Z are 0.52, -3.03 and -1.18 V respectively. The order of reducing power of the corresponding metals is (a) Y>Z>X (b) XP¥OZ (© Z>Y>x (@) Z>X>¥ 20. The correct order of chemical reactivity with water according to electrochemical serie (a) K> Mg> Zn> Cu (b) Mg>Zn>Cu>K () K>Zn>Mg>Cu (d) Cu>Zn>Mg>K 21. An electric current of ¢ ampere was passed through a solution of an electrolyte for *t’ sec- ond depositing P g of the metal M on the cath- ode. The equivalent weight E of the metal will be __exr ©) FB x96500 xP (0) B= 796500 a You have either reached 2 page thts unevalale fer vowing or reached your ievina tit for his book. a You have either reached 2 page thts unevalale fer vowing or reached your ievina tit for his book. a You have either reached 2 page thts unevalale fer vowing or reached your ievina tit for his book. Electrochemistry 427 _ 70x (300 x10") ee = 2176V 100 x (=10 x 96500) 78. Ans. (c) Ejp=—Ehp 76. Ans. (c) 0.059 fH] Epp = Egypt >” log At ne Big We 71. Ans. (b) Follow text 78. Ans. (d) I IT WINDOW—II l > [RIT 1. A gas ‘Rat 1 atm is bubbled through a solu- tion containing a mixture of | MQ and IMR at 25°C (298 K). If the reduction potential is in the order R > Q > P then (a) Q will oxidise P only (b) Qwill oxidise S only (©) S will oxidise both P & Q (d) S will reduce both P & Q 2. For the strong electrolyte 4 increases slowly with dilution and can be represented by the equation A= ae-acl® Which electrolytes (s) have the same value of the constant ‘A’ (a) NaCl (b) CaCl, (©) ZnCl, (@) AlCl, Which of the following reactions is / are said to be spontaneous? (a) Sn** (aq) + Ni(s) > Ni* (aq) + Sn*"(aq) (b) Ni?" (aq) + Fe + Fe*"(aq) + Ni(s) (c) Ag* (aq) + Cu Cu’ (aq) + Ag (d) Hg* (aq) + Cu > Cu®” + Hg (s) 4. Which of the followings is are correct (a) Li (b) Cell is spontaneous if C, 0 (b) Eu > 0, AG>0 (©) Ey <0, AG°>0 (a) E o> 0 AG"> 0 11. Ifthe 0.5 molar solution of M’ is replaced by a 0.0025 molar M* solution then the magnitude of cell potential would be (a) 35 mV (b) 70 mv. © 140mv (@) 200mv 12, The above cell is a type of (a) Concentration cell with transference (b) Concentration cell without transference (c) Concentration cell with electrolyte (@) Concentration cell without electrolyte Passage V Redox reaction play a vitol role in chemistry and biology. The value of standard redox po- tential (E°red) of two half cell reactions de- cides which way the reaction is expected to proceed. A simple example is a Daniel cell in which Zine goes into solution and copper gets, deposited. Given below are the set of half reaction (acidic medium) along with their E°(w.r. to NHE) values using this data the cor- rect explanations to given questions E°=0.54V FE 1,426 921 Cl, +26 > 2Cr Mn** +e > Mn**— 36V E° = 1.50V Fe" +e" > Fe 3 O, + 4H” +4e" 3 2H,0 23V {01T,2007] 13. Among the following identify the correct statements. (a) CI ion is oxidised by O, (b) Fe** ion is oxidised by iodine (©) I is oxidised by Cl, (d) Mn* is oxidised by chlorine 14. While Fe *” ion is stable Mn” is not stable in acid solution because (a) O, oxidises Mn?” to Mn (b) O, oxidises both Mn** to Mn** and Fe” to Fe™ (c) Fe * oxidises HO into O, (d) Mn* oxidises H,0 into O,. 15. Sodium fusion extract obtained from aniline, on treatment with Iron (I) sulphate and H,SO, in presence of air gives prussian blue precipi- tate. The blue colour is due to the formation of (a) Fe(Fe(CN)]——_(b) FeslFe(CN)l> (©) FeFe(CN)], — (d) FeylFe(CN)}; I IIT WINDOW-VIII l > RRs 1, Find the volume of gases evolved by passing, 0.965 A current for 1 hr through an aqueous solution of CH;COONa at 25°C and 1 atm. 2. A Zn rod weighing 25 g was kept in 100 mL of IM CuSO, solution. After a certain time the molarity of Cu’” in solution was 0.8. What was molarity of SO,?? What was the weight of Zn rod after cleaning? At. Weight of Zn= 65.4, 3. 50 mL of 0.1 M CuSO, solution is electroly- sed using Pt electrode with a current of 0.965 ampere for a period of 1 minute. Assuming that volume of solution does not change dur- ing electrolysis, calculate [Cu], [H'] and {SO,?}] after electrolysis. What will be the concentration of each species if current is passed using Cu electrodes? [Cu*"]=0.094M, [H"]=0.12M[SO,2 ]=0.1M. 4. 19 g fused SnCl, was electroslysed using inert electrodes. 0.1 1g Sn was deposited at cathode. a You have either reached 2 page thts unevalale fer vowing or reached your ievina tit for his book. a You have either reached 2 page thts unevalale fer vowing or reached your ievina tit for his book. a You have either reached 2 page thts unevalale fer vowing or reached your ievina tit for his book. CHAPTER Redox Oxidation and Reduction Oxidation and reduction are explained on the basis of, different manners are discussed as below: (A) On the basis of Hydrogen ‘The removal of hydrogen in any chemical reaction is called oxidation whereas addition of hydrogen is id to be reduction e.g. e209 H 2CH,OH + (oxidation in terms of CH,OH) A and, HC. + H,—> CH,OH (HCHO is reduced) H (B) On the basis of Oxygen ‘The addition of oxygen in any chemical reaction is said to be oxidation while its removal is called redu tion eg, HNO, + 0; - HNO, (HNO, is oxidized) CuO +H, + Cu +H, (Cu0 is reduced) Ag,O —45 2Ag + 1/20, (Ag,0 is reduced) (C) On the b: The addition of electronegative element in chemical reaction is called oxidation while removal of elec- tronegative element is called reduction, e.g. SnCl, + Cl, — SnCl,. [SnCl, is oxidized] Fel of Electronegative Element J) FeCl, + Cl, [FeCl, is reduced] (D) On the basis of Electron In any chemical reaction if loss of electron is ob- served, it is called oxidation and if there is gain of electron is observed is called reduction e.g. = jose | sts (E) On the basis of Oxidation Number Increase in oxidation number is called oxidation while its decrease is called reduction. e.g. 4 3 2 1 0 ot 2 Increase in 0.8 (Oxidation) Decrease in 0.8 (Reduction) a You have either reached 2 page thts unevalale fer vowing or reached your ievina tit for his book. a You have either reached 2 page thts unevalale fer vowing or reached your ievina tit for his book. a You have either reached 2 page thts unevalale fer vowing or reached your ievina tit for his book. 446 Numerical Chemistry 19. The highest oxidation state of s is found in (@) 1,80, (b) 18,0, (©) NaS,0, 20. In order to balance the half reaection €1,0,7 Cr", the no. of e, H” and H,O are added is, (@) 5.2.9 (b) 2.146 © 6.14, @ 14,67 21. The number of electrons required in the re- duction of nitrate ion (NO, ) into Hydrazine (NHNH,) is (a) 8 (b) 7 ©) 5 @)3 BE L mole of MnO,?° in neutral aqueous solution disproportionate to (2) 2/3 mole of Mn,0, and + mole of MnO, (b) + mote of Mn,O, and ; ‘mole of Mn (©) = mole of MnO, and ; mole of MnO, (@) 5 mole of MnO,” and : mole of MnO, 23. Acompound is made up of three elements A,B and C. If oxidation state of A, B and C are +2 +S and -2 respectively the correct formula of compound is (a) ABC, (©) AYBC), In the reaction M*+MnO, > MO, +Mn™* If one mole of MnO, oxidise 1.67 mole of M®" to MO, The value of x is (2 () 4 ©3 @ 5 25, The no. of mole of Cl, required per mole of NaOH in order to form NaC1O, and NaCl is (a) 0.25 (b) 0.5 (©) 0.75 @) 1 26. The no of ©, mole required per mole of NH, in order to form NO is (2 ) 3 4 @) 5 One mole of NH, loses 10 mole of electrons to form a new compound ¥. Assuming that all the nitrogen appears in the new compound. ‘What is the oxidation state of nitrogen in Y. b) AYB,O), (@) A, (BC); 24. 21. (There is no change in the oxidation state of hydrogen. @) 3 (b) 3 +5 (@ +1 28. The oxidation state of nitrogen in NH,NO, are (a) 3,3 (b) +3,43 (© 3+5 (@) 3-5 29. The oxidation state of carbon in CH,CHO is (a) 341 (b) 43.41 (©) +141 (@) 43.43 30. The no. of mole of NHO, required to react per mole of Zn in order to form NH,NO, along with other products is, (@ 35 (b) LS © 10 (@) 25 I IIT WINDOWS l > Oars 1. The oxidation state of S in Na,8,O, is / are (a) Zero (b) +6 (©) +5 @ 4, 2. The correct statement is/are (a) Oxidation state of Cr in CrO, is six. (b) CrO, contain 4 oxygen atoms having oxi- dation state -1 (©) CrO, has two peroxide bonds, (d)_ C10, has butterfly structure, 3. The oxidation state of S in thiosulphate ion is/ are, (a) 45 (b) +6 () -1 (@) -2 4, The oxidation state of M [M(H,0), (NO)/,] “is/are. fa) 42 (b) t4 © -1 @ 8 5. In the reaction KMn0, + H,SO, +H,0, K,SO, + MnSO, +H,0 +0, The correct statement is/are (a) KMn0, is a reducing agent. (b) KMnO, is redued. (©) H,0, is oxidised. (d) HO, is a reducing agent 6, In the Marshall’s acid (H,S,0,) the incorrect statements is/are. the complex a You have either reached 2 page thts unevalale fer vowing or reached your ievina tit for his book. a You have either reached 2 page thts unevalale fer vowing or reached your ievina tit for his book. a You have either reached 2 page thts unevalale fer vowing or reached your ievina tit for his book. 452_ Numerical Chemistry ‘Compound Formula Uses Magnesite MgC, Basic flux. Milk of Magnesia. Ma(OH), Antacid. Magnesia Mgo Basie ux. Magnesium Sulphate MgSO, Purgative. Magnecia Alba (MgCO.),[Mg(OH)3},2H,0 Mica K,0.3A1,0,6Si0,2H,0 Mohr's Salt FeSO,.(NH,),$0,.61,0 Norwegian Salt Paper Ca0.Ca(NO,), Nitrolim CaCN HC Nitrochalk NH.NO,(NH,),CO, Nitre KNO, Pink Salt (NH),SnCl, Philosopher's Wool ZnO Pharaoh Serpent Hg(CNS),, Phosphorite Ca(PO.). Plaster of Paris (€a80,),, H,0 Permutit, Na,AI,Si,0,.XHO Quartz SiO, Quick Silver Hg Quick Lime C20 Red Lead PbO Realgar As,S Rouge Fe,0, Sodamide NaNH, Salt Peter(Indian) - KNO, Sorel's Cement MgCl,.SMgO.XH,0 Spinel Maal. Slacked Lime Ca(OH), Sugar of Lead (CH3COO),Pb Scheele’s Green Cu(AsO,), Soda Lime NaOHCaO) Soda Ash Na,CO, Tetra Ethyl Lead(TEL) (CH)),P> Thermite Fo,0,+Al Thomas Slag €O,PO,);+CaSi0, Tincal Na,B,0,,10H,0 Antacid. Laxative. Insulator. Analytical ragent. Fertlzer. Nitrogeneous fertilizer. Nitrogencous fertilizer. Manufacturing gun powder & medicine, Mordant in dyeing. Antiseptic, Semiconductor. Red paint Super phosphate of lime. Plastering, Making statue, Moulds, Softening water. In watches. ‘Thermometer, For medicine. Preparing slacked lime, Bleaching powder, Dehydrating agent in Lab. Flint glass, Paints, ete Aurvedie medicines, Paints Red paint, Polishing powder. Eliminating agent for alkyl halides. Gun powder, Fertilizer. ‘Cement forties. Extraction of Aluminium. Preparation of NH,CaC, CaCO Lab.reagent Paints Manufacturing of hydrocarbons, Washing powder, Softering of water, Glass, Cement et Antiknocking agent. Rocket fuel, Extraction of Cr, Mn ete. Fertilizer Glazes and enamels. Comd a You have either reached 2 page thts unevalale fer vowing or reached your ievina tit for his book. a You have either reached 2 page thts unevalale fer vowing or reached your ievina tit for his book. a You have either reached 2 page thts unevalale fer vowing or reached your ievina tit for his book. Redox 459 Gases Distinguishing Reagent ‘Remarks N,0&0, Alkaline pyrogallol solution, (0,1Is absorbed by alkaline pyrogallol solution N,O-No reaction, NH, & HCI (a) Litmus Paper. NH,—>Turns moist red litmus blue, HCI-sTurns moist blue litmus red, (b) Nesseler’s reagent NH,+Turns Nesseler’s reagent. HCIANo reaction. H, & 0, Alkaline pyrogallol solution. _©,~»Absorbed in Sol", H,-»Does not abserb. SO, & HS Lead acetate Paper. H,S—Tums lead acetate paper black. $0,9No reaction. HC1& Cl, Starch Iodide Paper. C1, +Turns moist starch iodide paper blue and thin bleached. HCIANo reaction HBr & HI Cone. H,80, HBr—Brown Br, vapour is produced. HI-Violet vapour OH, produced. Cl, & Br(g) Moist Starch Iodide Paper. Cl,~+Turns moist starch iodide paper blue and bleached Br, Tums moist starch paper blue. NH, &NO Nesseler’s reagent NH,—sTurns Nesseler’s reagent brown, NONo reaction. Table 8 Important Pigments in Inorganic Chemistry ‘Name of the Pigment (Chemical Formula Chrome Yellow PbCrO, Zine Yellow Zn(OH), ZnCr0,,H,0 Chrome Red PbCrO,.PbO Red Lead PbO, Gulgret’s Green ¥,0, 21,0 Prussian Blue Feql Fe(CN) hy Sheela’s Green CuHLASO, Thenard’s Blue C00.Al,0, Vedigris / Basie Copper Acetate (CH,COO), Cu.Cu(OH)s Rinnman’s Green Zn.C00. White Zine Oxide Zn0 Lithopone ZnS + BaSO, White Lead 2PbCO,PH(OH), 14, White Basie Zine chromate ZnCr0,.Z(OH),

Vous aimerez peut-être aussi